Sunteți pe pagina 1din 460

Lecture 1

ChE 333
Spring 2000 1
Heat Transfer and Energy
What is Heat?
Heat is Energy in Transit. Recall the First law from Thermodynamics.
U = Q - W
What did we mean by all the terms?
What is U ?
What is Q ?
What is W ?
What is Heat Transfer?
Heat transfer is the transfer of Heat effected by a temperature
difference. However, in contrast to what we may have done in
Thermodynamics, we are concerned with the rate of heat transfer. For that,
we need a different energy balance, one that accounts for the rate of
transfer and production, a general energy balance.
We will derive such a balance in later lectures.
Examples of Heat Transfer
Pasteurizing milk
Energy loss from a house by conduction
through a window
Melting of a polymer
Cooling an automobile engine
Cooling or heating a stream in a chemical
process
Condensing vapor leaving a distillation
tower
Solar heating of the earth
Radiational cooling of a pond
Air-conditioning
Temperature control of the human body
Ablational cooling of a space shuttle on re-
entry

U = Q W
Lecture 1
ChE 333
Spring 2000 2
Modes of Heat Transfer
Conduction
Convection
Radiation
Conduction
Conduction is the thermal equivalent of diffusion. In gases, energy is
exchanged between molecules in collisions. You will recall from
Thermodynamics that a measure of the molecular motion and the energy
exchange is the temperature and that conduction is the transfer of
energy from the more energetic molecules to the less energetic. In the
presence of a temperature gradient, energy is then transferred from the high
temperature ( more energetic) to the lower temperature(less energetic). For
liquids and solids, the example is not as clear. Conduction in liquids the
collisions are more frequent and not as energetic and in solids transfer is
by vibrations, for example, in a crystal lattice.
The constitutive equation describing the phenomenon is Fouriers
law

q = k T
What we mean by the notation is the following:

q = q
i
e
i

T =
T
x
i
e
i
Lecture 1
ChE 333
Spring 2000 3
Conduction in one-dimension
To give you some notion of what I mean, examine a simple one-
dimensional conduction problem.
q
x
is the heat flux in watts/m
2

q
x
= k
dT
dx
k is the thermal conductivity in
units of watts/m-K
If we make an energy balance over a
differential element at steady state,

q
x
x+ x
q
x
x
= 0
or q
x
= constant
Then, we see that

T = ax + b or
dT
dx
=
T
1
T
2
L
It follows that at steady state

q
x
= k
T
1
T
2
L
Convection
Convection in heat transfer is a process which involves the diffusion of
heat and the advection of energy by flow. The process can be described
by Newtons Law of Cooling

q n = h T T
b
where h is the heat transfer coefficient as watts/m
2
-K
Typical Values of Heat Transfer Coefficients
q
T
T
1
T
2
x
L
h (watts/m
2
-K)
Free Convection 5 - 25
Forced Convection
gases 5-250
liquids 50 - 20,000
Phase Change
(boiling or condensation) 2500-100,000
Lecture 1
ChE 333
Spring 2000 4
What is the relation between h expressed in English units and SI units ?
In SI units , h = 10 watts/m
2
-K
In English units, h = 1.7612 BTU/hr-ft
2
-F
Quick estimates for conversion changes, divide watts/m
2
-K by 6 to get
BTU/hr-ft
2
-F
Radiation
Thermal radiation is energy emitted by matter that is at a finite
temperature. Emission occurs not only from solids, but from gases and
liquids. The energy is carried by electromagnetic waves, originating at the
expense of the internal energy of the matter. Conduction and convection
depend on the presence of an intermediary. Radiation does not !
The normal component of heat flux emitted by a surface is given by the
Stefan-Boltzmanns Law

q n = T
s
4
where is the emissivity is the Stefan-Boltzman constant
The emissivity is the ratio of the energy emitted by the real surface
compared to that emitted by an ideal surface (a black body). It is
dimensionless.
= 5.67 x 10
-8
watts/m
2
-K
4
and T is in degrees Kelvin.
The net rate of energy transfer between two surfaces

q
i j
n
j
= F
ij
T
i
4
T
j
4
The view factor, F
ij
, depends on the distance R between and the
orientation of the two surfaces.
We will discuss Radiation in the last several weeks of the course.
Lecture 1
ChE 333
Spring 2000 5
Problem Solving
Your textbook outlines a "stock" procedure for addressing, formulating
and solving problems in Heat Transfer. In my view, it is a sound
technique and one that I would ask that you follow.
It involves several steps, each with a prescribed formula. What folloiws is
the outline.
Problem Statement
The statement of the problem and the results required
Solution
1. Known
Read the problem carefully, then state briefly and concisely what is
know about the problem. This is not a simple restaement of the
problem.
2. Find
Briefly and concisely state the results that are required.
3. Schematic
Draw a picture (schematic) of the system. Identify and label the heat
transfer processes and the relevant boundaries of the system (control
surfaces)
4. Assumptions
List all the relevant assumptions you will use in formulating the
problem.
5. Properties
Compile the property valued for all the calculations to follow and
identify the sources of the data.
6. Analysis
Apply the appropriate conservation laws and appropriate constitutive
equations ( rate laws). Develop the analysis as completely as you
can without substituting numerical values. If it is feasible, make the
equations dimensionless. Finally complete the calculation with
numerical values.
7. Comments
Discuss the results. This should include the key conclusions. Were
the initial assumptions valid ? What effect would there be if they
were not? How sensitive is your solution to the parameters?
Lecture 1
ChE 333
Spring 2000 6
Measurement of Thermal Conductivity
A Design Problem
Statement of the Problem
Measure the conductivity of the metal in a metal rod connected to a
constant temperature sink (an ice bath)
Water flows through a well-mixed reservoir. In enters at T
h1
and leaves at
the same temperature as it is in the water reservoir, T
h2
. The rod, the two
reservoirs are insulated from the surroundings.
Derive a relationship for the thermal conductivity and recommend a flow
rate for the water.
Solution
1. Known
The heat lost by the fluid in the lower reservoir is transferred to the ice
batch through the rod. The temperature is known for the ce bath. The
water flow is measured and known, and we measure the water
temperatures in and out of the reservoir.
2. Find
Derive a relationship for the thermal conductivity and recommend a flow
rate for the water.
3. Schematic
F
T
h1
F
T
h2
T
T
h2
h2
L = 6 in.
D = 0.5 in.
D
Lecture 1
ChE 333
Spring 2000 7
4. Assumptions
There are a number of assumptions in this energy balance. What are they?
5. Properties
Some are given in the schematic, but what should we list ?
Data k (cal/s-cm-
K)
k (Btu/h-ft-
F)
Aluminum 0.45 108
Copper 0.9 216
Steel 0.13 32
Conditions
and Data
T
h1
=
80F
T
c
=
32F
k = 32BTU/ft-
h-F
C
pw
=
1BTU/lb-
F
= 62lb./ft^3
L = 6in.
D = 0.5in.
Lecture 1
ChE 333
Spring 2000 8
6. Analysis
Energy balance on the rod
Heat exchanged from the water = heat transferred across the rod

Q =
w
F
w
C
pw
T
h1
T
h2
=
k
L
D
2
4
T
h2
T
c
We did not state the constitutive relations . What are they?
There are a number of assumptions in this energy balance. What are they?
We can solve for the thermal conductivity , k.. The result gives us an
equation by which we might better design the experiment.

k =
4L
D
2

w
F
w
C
pw
T
h1
T
h2
T
h2
T
c
The dimensions, L and D are fixed as are all the variables save T
h1
and F
w
.
What are the best choices for these variables?
Dimensionless form for the solution.
We can group parameters as , so that

T
h1
T
h2
T
h2
T
c
=
D
2
k
4L
w
F
w
C
pw
=
and the temperature rise can be expressed simply as

T
h1
T
h2
T
h1
T
c
=

1 +
Lecture 1
ChE 333
Spring 2000 9
If we calculate the temperature rise as a function of flow rate we can obtain
the following table.
In an experiment we can measure temperature no more precisely that 0.1
F so that the highest flow rate would be 5 ml/min.
Some experimental questions
How precise can you control and measure flow rates?
How good is the well-stirred assumption?
How precise are the temperature measurements?
Given the precision of measurement, what is the
precision of the estimation of the thermal conductivity?
Flow rate


T
h1
T
h2
T
h1
T
c

T
h1
T
h2
cc/min.
100 0.00670 0.00666 0.3195
75 0.00894 0.00886 0.4251
50 0.01340 0.0132 0.6349
25 0.02681 0.0261 1.2533
10 0.06701 0.0628 3.0151
5 0.13405 0.1182 5.6738
4 0.16756 0.1435 6.8887
3 0.22342 0.1826 8.7656
2 0.33512 0.2510 12.0483
1 0.67025 0.4013 19.2618
Lecture 2 Spring 2000
ChE 333 1
Introduction to Conduction
Fourier's Law
The constitutive equation for conduction, we have see, is Fourier's Law. It
says that the heat flux vector is a linear function of the temperature
gradient, that is :
What we mean by the notation is the following:
Then for each of the components of q, we have the relation:

q = k T

q = q
i
e
i

T =
T
x
i
e
i

q
i
= k
T
x
i
Lecture 2 Spring 2000
ChE 333 2
Thermal Properties of Matter
Thermal conductivity
The conductivity is a material property that is a very strong function
of the state of the material.
The range of values goes from less than 0.01 Watts/m-K for gaseous
CO
2
to over 600 Watts/m-K for Ag metal. The change may be as much
as 5 orders of magnitude
.
Lecture 2 Spring 2000
ChE 333 3
Thermal Conductivity of Gases
Lecture 2 Spring 2000
ChE 333 4
Thermal Conductivity of Solids
The thermal conductivity of solids differ significantly as the next figure shows
.
Lecture 2 Spring 2000
ChE 333 5
Conductive Loss through a Window Pane
Examine the simple one-dimensional
conduction problem as heat flow
through a windowpane. The window
glass thickness, L, is 1/8 in. If this is the
only window in a room 9x12x8 or 864
ft
3
, the area of the window is 2 ft x 3 ft
or 6 ft
2
.
Recall that q
x
is the heat flux and that k
is the thermal conductivity

q
x
= k
dT
dx
The energy at steady state yielded

q
x
= k
T
1
T
2
L
The room is well heated and the temperature is uniform, so the heat low
through the windowpane is

Q = k
T
1
T
2
L
A
If the room temperature is 60 F and the exterior temperature is 20 F, and
k is 0.41 Btu/hr-ft2-F then

Q = 0.41
60 20
0.125 / 12
6 = 9444
Btu
hr
Questions
Is this a large rate?
How can you tell whether it is large or not?
q
T
T
1
T
2
x
L
Lecture 2 Spring 2000
ChE 333 6
Energy balance on the Room
How long does it take for the room temperature to change from 60 F to
45 F?
To make this estimate, we need to solve an energy balance on the room. A
simple analysis yields

d
dt
VC
p
T
1
T
ref
= Q
Recognizing that heat capacity density are essentially constant, the
equation becomes

dT
1
dt
=
kA
VC
p
L
T
1
T
2
Note that


=
VC
p
L
kA
and that has units of time.
At the outset, T
1
= T
10
= 60 F
The solution of the differential equation
representing the energy balance is

T
1
T
2
T
10
T
2
= e

To solve for the time required to get to


46 F, we need all the data in the table.

t

= ln
T
1
T
2
T
10
T
2
= ln
45 20
60 20
= 0.47
It follows that t = 0.47 = 1.75 minutes.
Data
T
2
= 20F
T
1 0
= 60F
T
1
= 45F
k = 0.41 BTU/ft-h-F
A = 6ft^2
V = 864ft^3
L = 0.125 in.
air
= 0.07 lb/ft^3
C
pair
= 0.24 BTU/lb.-F
Lecture 2 Spring 2000
ChE 333 7
Heat Conduction in a Composite Solid
Examine the simple one-dimensional
conduction problem as heat flow
through a thermally insulated
windowpane. Each layer of window
glass thickness, L, is 1/16 in. The
insulation layer of air between the two
panes is also 1/16 in.
Recall that q
x
is the heat flux and that k
is the thermal conductivity

q
x
= k
dT
dx
The energy at steady state yielded

q
x
= k
T
1
T
2
L
The heat flow through the glass is given by
In layer 1


Q = k
1
T
0
T
1

1
A
In layer 2


Q = k
2
T
1
T
2

2
A
and in layer 3 by


Q = k
3
T
2
T
3

3
A
Then we can rewrite the equations in this form

Q

3
k
3
= T
2
T
3
A ; Q

2
k
2
= T
1
T
2
A ; Q

1
k
1
= T
0
T
1
A
If we add the three equations, we obtain

Q
A

1
k
1
+

2
k
2
+

3
k
3
= T
0
T
3
q
T
T
0
T
2
x
L
1
T
1
T
3
L
3
L
2
Lecture 2 Spring 2000
ChE 333 8
We can consider the thickness/conductivity as a resistance so that

Let R
i
=

i
k
i
so then
1
R
=
1
R
1
+
1
R
2
+
1
R
3
The heat flow is then of the following form :

Q =
T
0
T
3
R
A
This is like a problem of current flow in a series circuit.
In the single pane problem discussed in Lecture 1, we noted that the
resistance, /k, was 1/(192(0.41) = 0.254 hr-ft
2
-F/Btu. Recall that for the
problem of cooling the room, was 1.75 minutes. The thermal
conductivity of air is 0.014 Btu/hr-ft-F. so that

1
k
1
=

3
k
3
=
0.0254
2
;

2
k
2
=
1
96 0.014
= 0.744
as a consequence the reciprocal of the overall resistance is 0.744 +
(0.0254) = 0.746.
Then we see that = (1.min) (0.746/0.0254) = 29.37 min
Lecture 2 Spring 2000
ChE 333 9
The Convective Boundary
Condition
Again consider a windowpane, but now
there is a heat transfer limitation at one
boundary described by a boundary
condition.

q
x
= h T
r
T
i
Conduction through the glass is
described by

q
x
= k
T
i
T
a

The flux is constant at any cross-section so that we can write



T
i
T
a
=

k
q
x
; T
r
T
i
=
1
h
q
x
Solving for the temperatures we get

T
i
= T
a
+

k
q
x
+
1
h
q
x
Solving for q
x
, the relation becomes

q
x
=
1

k
+
1
h
T
i
T
a
Which modified shows a correction to
the heat transfer coefficient modulated by
the conduction problem
The dimensionless number in the denominator is the Biot
number, a ratio of the convective heat transfer coefficient to the
equivalent heat transfer coefficient due to conduction.
Tr
T
i
T
a
L

q
x
=
h
h
k
+ 1
T
i
T
a

Bi =
h
k
Lecture 2 Spring 2000
ChE 333 10
Heat Transfer across a Composite Cylindrical Solid.
In the case of heat transfer in a cylinder, there is radial symmetry do that
heat conduction is important only in the radial direction.
The heat flux in the radial direction is
given by Fouriers law.

q
r
= k
dT
dr
The total heat flow through any circular surface is constant

Q = k 2rL
dT
dr
= constant = C
Rearranging we obtain a relation for the temperature gradient

dT
dr
=
C
k 2rL
which upon separation of variables is

dT =
C
k 2L
dr
r
An indefinite integration yields the temperature profile.

T =
C
k 2L
ln r + a
1
The boundary conditions are
at r = R
1
, T = T
1
;
at r = R
2
, T = T
2
q
r2
= q
r2
at r = R
3
, T = T
3
q
r3
= h(T
3
- T
0
)
T
1
T
2
T
3
T
0
R
1
R
2
R
3
Lecture 2 Spring 2000
ChE 333 11
so that

T
1
T
2
=
C
k
1
2L
ln
R
1
R
2
; T
2
T
3
=
C
k
2
2L
ln
R
2
R
1
It follows that

T
1
T
0
=
C
2L
1
k
1
ln
R
2
R
1
+
1
k
2
ln
R
3
R
2
+
1
h
This can be expressed as

Q =
2L
1
k
1
ln
R
2
R
1
+
1
k
2
ln
R
3
R
2
+
1
hR
3
T
1
T
0
Optimal Insulation on a Pipe
Is there an optimal thickness for the exterior insulation? In the context of
the problem just formulated, is there a best value for R
3
?
Note that Q = f(R
3
).
To find an extremum,

dQ
dR
3
= 0 and
d
2
Q
dR
3
2
< 0

Some algebra yields:


hR
3
k
2
= 1
It offers a critical radius for R
3
= k
2
/h beyond which the heat
loss increases.
Lecture 2 Spring 2000
ChE 333 1
Introduction to Conduction
Fourier's Law
The constitutive equation for conduction, we have see, is Fourier's Law. It
says that the heat flux vector is a linear function of the temperature
gradient, that is :
What we mean by the notation is the following:
Then for each of the components of q, we have the relation:

q = k T

q = q
i
e
i

T =
T
x
i
e
i

q
i
= k
T
x
i
Lecture 2 Spring 2000
ChE 333 2
Thermal Properties of Matter
Thermal conductivity
The conductivity is a material property that is a very strong function
of the state of the material.
The range of values goes from less than 0.01 Watts/m-K for gaseous
CO
2
to over 600 Watts/m-K for Ag metal. The change may be as much
as 5 orders of magnitude
.
Lecture 2 Spring 2000
ChE 333 3
Thermal Conductivity of Gases
Lecture 2 Spring 2000
ChE 333 4
Thermal Conductivity of Solids
The thermal conductivity of solids differ significantly as the next figure shows
.
Lecture 2 Spring 2000
ChE 333 5
Conductive Loss through a Window Pane
Examine the simple one-dimensional
conduction problem as heat flow
through a windowpane. The window
glass thickness, L, is 1/8 in. If this is the
only window in a room 9x12x8 or 864
ft
3
, the area of the window is 2 ft x 3 ft
or 6 ft
2
.
Recall that q
x
is the heat flux and that k
is the thermal conductivity

q
x
= k
dT
dx
The energy at steady state yielded

q
x
= k
T
1
T
2
L
The room is well heated and the temperature is uniform, so the heat low
through the windowpane is

Q = k
T
1
T
2
L
A
If the room temperature is 60 F and the exterior temperature is 20 F, and
k is 0.41 Btu/hr-ft2-F then

Q = 0.41
60 20
0.125 / 12
6 = 9444
Btu
hr
Questions
Is this a large rate?
How can you tell whether it is large or not?
q
T
T
1
T
2
x
L
Lecture 2 Spring 2000
ChE 333 6
Energy balance on the Room
How long does it take for the room temperature to change from 60 F to
45 F?
To make this estimate, we need to solve an energy balance on the room. A
simple analysis yields

d
dt
VC
p
T
1
T
ref
= Q
Recognizing that heat capacity density are essentially constant, the
equation becomes

dT
1
dt
=
kA
VC
p
L
T
1
T
2
Note that


=
VC
p
L
kA
and that has units of time.
At the outset, T
1
= T
10
= 60 F
The solution of the differential equation
representing the energy balance is

T
1
T
2
T
10
T
2
= e

To solve for the time required to get to


46 F, we need all the data in the table.

t

= ln
T
1
T
2
T
10
T
2
= ln
45 20
60 20
= 0.47
It follows that t = 0.47 = 1.75 minutes.
Data
T
2
= 20F
T
1 0
= 60F
T
1
= 45F
k = 0.41 BTU/ft-h-F
A = 6ft^2
V = 864ft^3
L = 0.125 in.
air
= 0.07 lb/ft^3
C
pair
= 0.24 BTU/lb.-F
Lecture 2 Spring 2000
ChE 333 7
Heat Conduction in a Composite Solid
Examine the simple one-dimensional
conduction problem as heat flow
through a thermally insulated
windowpane. Each layer of window
glass thickness, L, is 1/16 in. The
insulation layer of air between the two
panes is also 1/16 in.
Recall that q
x
is the heat flux and that k
is the thermal conductivity

q
x
= k
dT
dx
The energy at steady state yielded

q
x
= k
T
1
T
2
L
The heat flow through the glass is given by
In layer 1


Q = k
1
T
0
T
1

1
A
In layer 2


Q = k
2
T
1
T
2

2
A
and in layer 3 by


Q = k
3
T
2
T
3

3
A
Then we can rewrite the equations in this form

Q

3
k
3
= T
2
T
3
A ; Q

2
k
2
= T
1
T
2
A ; Q

1
k
1
= T
0
T
1
A
If we add the three equations, we obtain

Q
A

1
k
1
+

2
k
2
+

3
k
3
= T
0
T
3
q
T
T
0
T
2
x
L
1
T
1
T
3
L
3
L
2
Lecture 2 Spring 2000
ChE 333 8
We can consider the thickness/conductivity as a resistance so that

Let R
i
=

i
k
i
so then
1
R
=
1
R
1
+
1
R
2
+
1
R
3
The heat flow is then of the following form :

Q =
T
0
T
3
R
A
This is like a problem of current flow in a series circuit.
In the single pane problem discussed in Lecture 1, we noted that the
resistance, /k, was 1/(192(0.41) = 0.254 hr-ft
2
-F/Btu. Recall that for the
problem of cooling the room, was 1.75 minutes. The thermal
conductivity of air is 0.014 Btu/hr-ft-F. so that

1
k
1
=

3
k
3
=
0.0254
2
;

2
k
2
=
1
96 0.014
= 0.744
as a consequence the reciprocal of the overall resistance is 0.744 +
(0.0254) = 0.746.
Then we see that = (1.min) (0.746/0.0254) = 29.37 min
Lecture 2 Spring 2000
ChE 333 9
The Convective Boundary
Condition
Again consider a windowpane, but now
there is a heat transfer limitation at one
boundary described by a boundary
condition.

q
x
= h T
r
T
i
Conduction through the glass is
described by

q
x
= k
T
i
T
a

The flux is constant at any cross-section so that we can write



T
i
T
a
=

k
q
x
; T
r
T
i
=
1
h
q
x
Solving for the temperatures we get

T
i
= T
a
+

k
q
x
+
1
h
q
x
Solving for q
x
, the relation becomes

q
x
=
1

k
+
1
h
T
i
T
a
Which modified shows a correction to
the heat transfer coefficient modulated by
the conduction problem
The dimensionless number in the denominator is the Biot
number, a ratio of the convective heat transfer coefficient to the
equivalent heat transfer coefficient due to conduction.
Tr
T
i
T
a
L

q
x
=
h
h
k
+ 1
T
i
T
a

Bi =
h
k
Lecture 2 Spring 2000
ChE 333 10
Heat Transfer across a Composite Cylindrical Solid.
In the case of heat transfer in a cylinder, there is radial symmetry do that
heat conduction is important only in the radial direction.
The heat flux in the radial direction is
given by Fouriers law.

q
r
= k
dT
dr
The total heat flow through any circular surface is constant

Q = k 2rL
dT
dr
= constant = C
Rearranging we obtain a relation for the temperature gradient

dT
dr
=
C
k 2rL
which upon separation of variables is

dT =
C
k 2L
dr
r
An indefinite integration yields the temperature profile.

T =
C
k 2L
ln r + a
1
The boundary conditions are
at r = R
1
, T = T
1
;
at r = R
2
, T = T
2
q
r2
= q
r2
at r = R
3
, T = T
3
q
r3
= h(T
3
- T
0
)
T
1
T
2
T
3
T
0
R
1
R
2
R
3
Lecture 2 Spring 2000
ChE 333 11
so that

T
1
T
2
=
C
k
1
2L
ln
R
1
R
2
; T
2
T
3
=
C
k
2
2L
ln
R
2
R
1
It follows that

T
1
T
0
=
C
2L
1
k
1
ln
R
2
R
1
+
1
k
2
ln
R
3
R
2
+
1
h
This can be expressed as

Q =
2L
1
k
1
ln
R
2
R
1
+
1
k
2
ln
R
3
R
2
+
1
hR
3
T
1
T
0
Optimal Insulation on a Pipe
Is there an optimal thickness for the exterior insulation? In the context of
the problem just formulated, is there a best value for R
3
?
Note that Q = f(R
3
).
To find an extremum,

dQ
dR
3
= 0 and
d
2
Q
dR
3
2
< 0

Some algebra yields:


hR
3
k
2
= 1
It offers a critical radius for R
3
= k
2
/h beyond which the heat
loss increases.
Lecture 2 Spring 2000
ChE 333 1
Introduction to Conduction
Fourier's Law
The constitutive equation for conduction, we have see, is Fourier's Law. It
says that the heat flux vector is a linear function of the temperature
gradient, that is :
What we mean by the notation is the following:
Then for each of the components of q, we have the relation:

q = k T

q = q
i
e
i

T =
T
x
i
e
i

q
i
= k
T
x
i
Lecture 2 Spring 2000
ChE 333 2
Thermal Properties of Matter
Thermal conductivity
The conductivity is a material property that is a very strong function
of the state of the material.
The range of values goes from less than 0.01 Watts/m-K for gaseous
CO
2
to over 600 Watts/m-K for Ag metal. The change may be as much
as 5 orders of magnitude
.
Lecture 2 Spring 2000
ChE 333 3
Thermal Conductivity of Gases
Lecture 2 Spring 2000
ChE 333 4
Thermal Conductivity of Solids
The thermal conductivity of solids differ significantly as the next figure shows
.
Lecture 2 Spring 2000
ChE 333 5
Conductive Loss through a Window Pane
Examine the simple one-dimensional
conduction problem as heat flow
through a windowpane. The window
glass thickness, L, is 1/8 in. If this is the
only window in a room 9x12x8 or 864
ft
3
, the area of the window is 2 ft x 3 ft
or 6 ft
2
.
Recall that q
x
is the heat flux and that k
is the thermal conductivity

q
x
= k
dT
dx
The energy at steady state yielded

q
x
= k
T
1
T
2
L
The room is well heated and the temperature is uniform, so the heat low
through the windowpane is

Q = k
T
1
T
2
L
A
If the room temperature is 60 F and the exterior temperature is 20 F, and
k is 0.41 Btu/hr-ft2-F then

Q = 0.41
60 20
0.125 / 12
6 = 9444
Btu
hr
Questions
Is this a large rate?
How can you tell whether it is large or not?
q
T
T
1
T
2
x
L
Lecture 2 Spring 2000
ChE 333 6
Energy balance on the Room
How long does it take for the room temperature to change from 60 F to
45 F?
To make this estimate, we need to solve an energy balance on the room. A
simple analysis yields

d
dt
VC
p
T
1
T
ref
= Q
Recognizing that heat capacity density are essentially constant, the
equation becomes

dT
1
dt
=
kA
VC
p
L
T
1
T
2
Note that


=
VC
p
L
kA
and that has units of time.
At the outset, T
1
= T
10
= 60 F
The solution of the differential equation
representing the energy balance is

T
1
T
2
T
10
T
2
= e

To solve for the time required to get to


46 F, we need all the data in the table.

t

= ln
T
1
T
2
T
10
T
2
= ln
45 20
60 20
= 0.47
It follows that t = 0.47 = 1.75 minutes.
Data
T
2
= 20F
T
1 0
= 60F
T
1
= 45F
k = 0.41 BTU/ft-h-F
A = 6ft^2
V = 864ft^3
L = 0.125 in.
air
= 0.07 lb/ft^3
C
pair
= 0.24 BTU/lb.-F
Lecture 2 Spring 2000
ChE 333 7
Heat Conduction in a Composite Solid
Examine the simple one-dimensional
conduction problem as heat flow
through a thermally insulated
windowpane. Each layer of window
glass thickness, L, is 1/16 in. The
insulation layer of air between the two
panes is also 1/16 in.
Recall that q
x
is the heat flux and that k
is the thermal conductivity

q
x
= k
dT
dx
The energy at steady state yielded

q
x
= k
T
1
T
2
L
The heat flow through the glass is given by
In layer 1


Q = k
1
T
0
T
1

1
A
In layer 2


Q = k
2
T
1
T
2

2
A
and in layer 3 by


Q = k
3
T
2
T
3

3
A
Then we can rewrite the equations in this form

Q

3
k
3
= T
2
T
3
A ; Q

2
k
2
= T
1
T
2
A ; Q

1
k
1
= T
0
T
1
A
If we add the three equations, we obtain

Q
A

1
k
1
+

2
k
2
+

3
k
3
= T
0
T
3
q
T
T
0
T
2
x
L
1
T
1
T
3
L
3
L
2
Lecture 2 Spring 2000
ChE 333 8
We can consider the thickness/conductivity as a resistance so that

Let R
i
=

i
k
i
so then
1
R
=
1
R
1
+
1
R
2
+
1
R
3
The heat flow is then of the following form :

Q =
T
0
T
3
R
A
This is like a problem of current flow in a series circuit.
In the single pane problem discussed in Lecture 1, we noted that the
resistance, /k, was 1/(192(0.41) = 0.254 hr-ft
2
-F/Btu. Recall that for the
problem of cooling the room, was 1.75 minutes. The thermal
conductivity of air is 0.014 Btu/hr-ft-F. so that

1
k
1
=

3
k
3
=
0.0254
2
;

2
k
2
=
1
96 0.014
= 0.744
as a consequence the reciprocal of the overall resistance is 0.744 +
(0.0254) = 0.746.
Then we see that = (1.min) (0.746/0.0254) = 29.37 min
Lecture 2 Spring 2000
ChE 333 9
The Convective Boundary
Condition
Again consider a windowpane, but now
there is a heat transfer limitation at one
boundary described by a boundary
condition.

q
x
= h T
r
T
i
Conduction through the glass is
described by

q
x
= k
T
i
T
a

The flux is constant at any cross-section so that we can write



T
i
T
a
=

k
q
x
; T
r
T
i
=
1
h
q
x
Solving for the temperatures we get

T
i
= T
a
+

k
q
x
+
1
h
q
x
Solving for q
x
, the relation becomes

q
x
=
1

k
+
1
h
T
i
T
a
Which modified shows a correction to
the heat transfer coefficient modulated by
the conduction problem
The dimensionless number in the denominator is the Biot
number, a ratio of the convective heat transfer coefficient to the
equivalent heat transfer coefficient due to conduction.
Tr
T
i
T
a
L

q
x
=
h
h
k
+ 1
T
i
T
a

Bi =
h
k
Lecture 2 Spring 2000
ChE 333 10
Heat Transfer across a Composite Cylindrical Solid.
In the case of heat transfer in a cylinder, there is radial symmetry do that
heat conduction is important only in the radial direction.
The heat flux in the radial direction is
given by Fouriers law.

q
r
= k
dT
dr
The total heat flow through any circular surface is constant

Q = k 2rL
dT
dr
= constant = C
Rearranging we obtain a relation for the temperature gradient

dT
dr
=
C
k 2rL
which upon separation of variables is

dT =
C
k 2L
dr
r
An indefinite integration yields the temperature profile.

T =
C
k 2L
ln r + a
1
The boundary conditions are
at r = R
1
, T = T
1
;
at r = R
2
, T = T
2
q
r2
= q
r2
at r = R
3
, T = T
3
q
r3
= h(T
3
- T
0
)
T
1
T
2
T
3
T
0
R
1
R
2
R
3
Lecture 2 Spring 2000
ChE 333 11
so that

T
1
T
2
=
C
k
1
2L
ln
R
1
R
2
; T
2
T
3
=
C
k
2
2L
ln
R
2
R
1
It follows that

T
1
T
0
=
C
2L
1
k
1
ln
R
2
R
1
+
1
k
2
ln
R
3
R
2
+
1
h
This can be expressed as

Q =
2L
1
k
1
ln
R
2
R
1
+
1
k
2
ln
R
3
R
2
+
1
hR
3
T
1
T
0
Optimal Insulation on a Pipe
Is there an optimal thickness for the exterior insulation? In the context of
the problem just formulated, is there a best value for R
3
?
Note that Q = f(R
3
).
To find an extremum,

dQ
dR
3
= 0 and
d
2
Q
dR
3
2
< 0

Some algebra yields:


hR
3
k
2
= 1
It offers a critical radius for R
3
= k
2
/h beyond which the heat
loss increases.
Lecture 3 Spring 2000
ChE 333 1
Conduction in a Simple Fin
Finned tubes are common in heat exchangers to afford greater surface area
for heat transfer. How do they enhance heat transfer?
Steady State Energy
Balance
Conductive flux at z

q
z
z
= k
dT
dz
z
Conductive flux at z

q
z
z+ z
= k
dT
dz
z+ z
The convective flux in the x direction

q
x
z
= h T z T
a
Combining the equations, we obtain

k
dT
dz
z
k
dT
dz
z
2Bw h T z T
a
2wz =0
So that when we take the limit as z ---> 0

d
dz
k
dT
dz
=
h
B
T z T
a
The boundary conditions are

at z = 0, T = T
a
and at z = L, q
z
L
= k
dT
dz
L
= 0
L
x
T
w
z
T
a
Lecture 3 Spring 2000
ChE 333 2
Adimensionalization
If we define the following dimensions

=
T T
a
T
w
T
a
; =
z
L
; N =
hL
2
kB
The differential equation becomes

d
2

d
2
= N
2

Using the boundary conditions



At = 0 , = 1 and at = 1 ,
d
d
= 0
The equation becomes

= cosh N + tanh N sinh N
Rate of heat flow into fin through the base

Q = k
dT
dz
z =0
2Bw
Rate of convection from the surface

Q = 2wh T T
a
dz
0
L
Lecture 3 Spring 2000
ChE 333 3
Bounds
If the fins temperature were the same as the base, the rate of heat transfer
from the fin surface would be
The minimum flux through an unfinned surface is

Q
0
= 2wBh T
w
T
a
The maximum heat flow from the fin

Q
max
= 2wLh T
w
T
a
The fractional heat flow through the fin is the efficiency

Q
Q
max
=
2wh T T
a
dz
0
L
2wLh T
w
T
a
Expressed in dimensionless terms the efficiency is

=
Q
Q
max
= d
0
1
For a planar flat fin, the
efficiency is given by


=
tanh N
N
Efficiency Plot
0.1
1
0.1 1 10
Modulus
E
f
f
i
c
i
e
n
c
y
Lecture 3 Spring 2000
ChE 333 4
Conduction with Heat Generation
Consider an insulated electrical wire of radius R
1
covered with a thickness
of electrical insulation so that the outside radius is

R
2
.
Joule heating is given by R
2

S
e
= I
2
R =
I
2
k
e
R
1
The heat flux in the copper wire is given by Fouriers Law

q
r
= k
dT
dr
An energy balance (steady-state) yields

2rq
r
r
2rq
r
r + r
L +S
e
2r dr L = 0
so that the differential equation becomes

d
dr
rq
r
= rS
e
When we invoke Fouriers law, we obtain the conduction equation.

d
dr
r
dT
dr
= r
S
e
k
When the equation is integrated in the wire region, we obtain

T =
S
e
4k
r
2
a ln r + b
Lecture 3 Spring 2000
ChE 333 5
The same can be done in the insulation to give

d
dr
r
dT
1
dr
= 0
The solution is

T
1
= c ln r + d
Now there are four constants to determine four boundary conditions.

T = T
1
and k
dT
dr
= k
1
dT
1
dr
at r = R
1

T = T
1
and k
dT
dr
= k
1
dT
1
dr
at r = R
1

k
1
dT
1
dr
= h T
1
T
a
The final condition is that the temperature T is finite at r = 0.
The results are the following two equations.

T T
a
=
S
e
R
1
2
4k
1
r
R
1
2

2k
k
1
ln
R
1
R
2
+
2k
hR
2

T T
a
=
S
e
R
1
2
2k
1
ln
R
2
r
+
2k
hR
2
Lecture 4 Spring 2000
ChE 333 1
The Energy Balance
Consider a volume enclosing a mass M and bounded by a surface .
At a point x, the density is , the local
velocity is v, and the local Energy
density is U.
The rate of change total energy in is:

d
dt
UdV

The heat flow from the body is



q n dS

The Work done by the body on the surroundings is



vTn dS

+ gvdV

Since for the body



U = Q W
An equivalent form is

d
dt
UdV

+ q n dS

=
vTn dS

+ gvdV

If our control volume is a differential cube, the differential equation


describing the Energy Equation is:

t
U + Uv + q = vT + gv
U
v

v
s
.
n

Lecture 4 Spring 2000


ChE 333 2
The first term is the local rate of energy change
The second is the convective energy flow
The third is the sum of reversible work and dissipation
The last is the work done by the gravitational acceleration.
Other Conservation Laws
Mass

t
+ v = 0
Momentum

t
v + vv T g = 0
Mechanical Energy
This is obtained by taking the inner product of the momentum equation
and the momentum equation to yield

v

t
v + vv T g = 0
The real Energy Equation
The real Energy equation is obtained by subtracting the Mechanical
Energy Balance from the complete Energy Equation, using the mass
balance and recognizing that H = U + PV.

t
H + vH + q =

v W
s
+

=1
S
This is simplified recalling that


H
T
p
= C
p

C
p

t
T + vT + q =

v W
s
+

=1
S
Lecture 4 Spring 2000
ChE 333 3
Applications of the Energy Equation
to Steady State Conduction
The Energy Equation was

C
p

t
T + vT + q =

v W
s
+

=1
S
But for systems at steady state where there is no motion, no shaft work
done, and no chemical reaction
time derivatives vanish
the velocity, v, is zero
the shaft work iz zero, and
the reaction rate is zero..
This means that the energy equation has a very simple form
Recall that Fourier's Law is a relation for the heat flux, q,
so that
In rectangular Cartesian coordinates, the resulting equation becomes the
steady state head conduction equation .

q = 0

q = k T

k T = 0
and it follows for constantk, that

2
T = 0

2
T
x
2
+

2
T
y
2
+

2
T
z
2
= 0
Lecture 4 Spring 2000
ChE 333 4
Boundary Conditions
Types of boundary conditions in heat transfer problems
1. Constant surface temperature
On a surface S, the temperature is constant if
T(x, t) = T
s
2. Constant heat flux
a) At a surface S, the flux is continuous, finite, and constant so
that :
b) At an adiabatic surface S, the flux vanishes:
3. Convective Surface condition
At any surface, the flux leaving one body is equal to the flux leaving
the other, so that

q
i
= k
T
x
i
S
?

q
i
= k
T
x
i
S
?
= 0

k
T'
x
i
S
1
= k
T"
x
i
S
2
Lecture 4 Spring 2000
ChE 333 5
A Simple Steady State Conduction Problem
Consider a rectangular slab of infinite extent in the z-direction
_______ side is length L, the vetrical sides are of length W.
The differential equation for steady heat conduction in 2 dimensions is:
The boundary conditions are:
T
1
T
1
T
1
T
2

2
T
x
2
+

2
T
y
2
= 0

T = T
1
at y = W
T = T
0
at y = 0
T = T
0
at x = 0
T = T
0
at x = L
Lecture 4 Spring 2000
ChE 333 6
If the Temperature. T, and the independent variables , x and y, are made
dimensionless, as
The conduction equation becomes
With the boundary conditions transformed to
The method we use to solve this partial differential equation is "the method
of separation of variables".
Assume that the solution is of the form
We obtain an equation of the form

=
T T
0
T
1
T
0
and =
y
W
and =
x
L

2
+
L
W
2

2
= 0

= 1 at = 1
= 0 at = 0
= 0 at = 0
= 0 at = 1

= F G

2
F G

2
+
L
W
2

2
F G

2
= 0
Lecture 4 Spring 2000
ChE 333 7
We group the terms that depend on each individual independent variable
so that
If we divide by FG and separate variables, we obtain

G
d
2
F
d
2
+
L
W
2
F
d
2
G
d
2
= 0

For simplicity, let
2
=
L
W

1
F
d
2
F
d
2
=
L
W
2
1
G
d
2
G
d
2
= constant =
2
Lecture 4 Spring 2000
ChE 333 8
The result is that we have two ordinary differential equations to solve:
so that
The solutions to the pair are :
The entire solution is of the form
If we recognize that since at = 0, = 0, then B = 0 and the solution
simplifies considerably.

d
2
F
d
2
+
2
F = 0
d
2
G
d
2

2
G = 0

= F G = A sin + B cos
C sinh

+ D cosh


F = A sin + B cos
G = C sinh

+ D cosh


= A' sinh

sin + B' cosh

sin
Lecture 4 Spring 2000
ChE 333 9
The boundary condition at = 0 gives
So that B' must be zero and the simplified solution is
There are two constants left, and A', and two boundary conditions.
The condition at = 1 leads to
And we must note that
Either A' must vanish and the solution is trivial or
This is true if and only if = n where n = 0, 1,2,3,.......
That means that there are a countable infinite number of solutions. To
find the solution we need to add all the possible solutions and determine
the coefficients (constants).

= 0 at = 0 leads to
0 = A' sinh 0 sin + B' cosh 0 sin

= A' sinh

sin

0 = A' sinh

sin

0 = sin

= a
n
sinh
n

sin n

n= 1

Lecture 4 Spring 2000


ChE 333 10
The coefficients may be determined by the last boundary condition.
To determine the coefficient a
n
, we have to recognize the orthogonality
property of sin functions, that is,
To determine the coefficients, we can use the orthogonality properties of
the sine and cosine functions.
We integrate

= 1 at = 1
1 = a
n
sinh
n

sin n

n= 1


sin(n)sin(m)d
0
1
=
0 for m n

2
for m = n

a
n
sinh
n

sin(n)sin(m)d
0
1

n= 1

= (1)sin(m)d
0
1
Lecture 4 Spring 2000
ChE 333 11
Remember that the first sine integral is non-zero if and only if n = m.
Now the equation for a
n
is
or
Finally the solution is

a
n

2
sinh
n

= sin(n)d
0
1
=
cos (n)
0
1
n

a
n
=
2

1 1
n
n sinh
n


=
2 1 1
n
n
sinh
n

sinh
n

sin n

n= 1

ChE 333- Lecture 6 1


Spring 2000
Applications of the Energy Equation
Solids with a Uniform Temperature
Suppose a metal sphere of uniform temperature, T.
Heat is transferred by convection with a heat transfer
coefficient, h. The temperature of the surroundings is
T
a
Therefore the energy balance is

C
p
V
dT
dt
= hA T T
a
(Please note the difference between this equation and 11.1.1 in the text. This is the
correct form)
If the sphere is initially at T
0
, how does one describe the cooling of the
sphere?
The equation is separable so that

dT
T T
a
=
hA
C
p
V
dt
It follows that the solution is

ln
T T
a
T
0
T
a
=
hA
C
p
V
t
Or more explicitly

T T
a
T
0
T
a
= e

hA
C
p
V
t
ChE 333- Lecture 6 2
Spring 2000
Adimensionalization
If I had defined the following:

=
T T
a
T
0
T
a
and =
C
p
V
hA
The solution has a simple expression ... = e
-t/
Measurement of a Convective Heat Transfer Coefficient
Suppose a sphere of radius R in a stagnant gas of infinite extent. The
heat flux from the sphere through the gas is given by Fouriers law

q
r
= k
g
dT
dr
The heat flow through a spherical shell is constant so

r
2
q
r
= r
2
k
g
dT
dr
= C
The boundary conditions are T = T
s
at r = R and T -> T
a
at r ->
The solution becomes

T T
a
T
R
T
a
=
R
r
ChE 333- Lecture 6 3
Spring 2000
We can calculate the heat flux at the surface as

q
r
r = R
= k
g
dT
dr
r = R
= k
g
T
R
T
a
R
We can define a heat transfer coefficient as

h
q
r
T
R
T
a
=
k
g
R
The corresponding Nusselt Number for heat transfer in a stagnant gas is
This represents a lower bound for convective heat
transfer, given that there is no gas flow. We expect
the heat transfer coefficient to be larger.

Nu =
hD
k
g
= 2
ChE 333- Lecture 6 4
Spring 2000
The experiment
The thermal conductivity of air is 0.014 Btu/ft-F. If the sphere was 1
cm. in diameter, then h = 0.014(1/2.54(12)) = 4.2 Btu/hr-ft
2
-F
Note that h = = 4.2 Btu/hr-ft
2
-F = 0.00117 Btu/sec-ft
2
-F
If = 436 lb/ft
3
and C
p
= 0.12 Btu/lb.-F, then

=
0.00117 6
436 0.12 1 / 30
= 0.00403 sec

If the heat transfer coefficient were 5 times larger (Nu = 10) then =
0.02 sec,
If the heat transfer coefficient were 10 times larger (Nu = 100) then =
0.2 sec,
It should be clear that we cannot make a reasonable verification of a
uniform temperature until we solve for the temperature field in the
sphere.
Lecture 8
ChE 333 1
Unsteady State Heat Conduction in a Bounded Solid
How Does a Solid Sphere Cool ?
We examined the cooling a sphere of radius R. Initially the sphere is at a
uniform temperature T
0
. It is cooled by convection to an air stream at
temperature T
a
.
How long does it take to cool to T
a
?
The answer is and was simple ..... an infinitely long time. In a sense
because it is the answer equivalent to the solution of the Archimedean
Paradox.
If I walk half the distance to a wall, how many steps will I have to take to
reach the wall ?
The answer is an infinite number!
However, if I get within a millimeter of the wall, for intents and purposes, I
am there.
So if the temperature is within 1 % of the final temperature, it will have
reached the final temperature.
What temperature am I taking about.....the center line temperature, the
surface temperature, the average temperature ???
We will choose the average temperature, .
The first model we looked at should be valid for small Biot numbers

=
T T
a
T
0
T
a
= e

hA
C
p
V
t
= e
3Bix
Fo
To get this form we had to recognize that for spheres, A/V = 3/R.
If = 0.01, then 3 Bi x
Fo
= ln (0.01), so that x
Fo
= 1.535/Bi
At large Biot numbers, the suitable model was

sph
= 0.608 e
9.87x
Fo
Lecture 8
ChE 333 2
Now for this case the value of x
Fo
it takes to reach = 0.01 is
x
Fo
= 0.415
A plot of the response is shown below
Time for a temperature
drop of 99%
0.1
1
10
100
0.01 0.1 1 10 100
Bi
X
F
o
Some Dimensional Arguments
At large Biot numbers, the dimensionless time is constant, that is, x
Fo
= 0.415, but

x
Fo
=
t
R
2
so that for two spheres one of size R
1
and another R
2,
the ratio of the
cooling times is

t
1
t
2
=
R
2
R
1
2
Lecture 8
ChE 333 3
Heat Transfer in a Semi-Infinite region
The questions we have posed thus far and the solutions have been really
only applicable for Long times. That is, when the temperature field in
the sphere, for example, has developed to the center of the sphere. But
what happens at Short times?
Consider a large planar solid whose extent (y-direction) is very large.
What is the temperature history of the slab if it is suddenly brought into
contact with a fluid at temperature T
a
? The transient conduction equation
is

T
t
=

2
T
y
2
at t = 0, T = T
0
at y = 0 , T = T
a
as y , T = T
0
Lets make the problem dimensionless.
The temperature can be expressed as

=
T T
a
T
0
T
a
so that the problem reposed is

t
=

y
2
= 1 at t = 0
= 0 at y = 0
= 1 as y
Lecture 8
ChE 333 4
Solution
Let = f((y,t)) where = cy
m
t
n
, then we can introduce that into the
differential equation.

t
=
d
d

t
=
d
d

t
=
d
d
cny
m
t
n 1

y
=
d
d

y
=
d
d

y
=
d
d
cmy
m1
t
n

y
2
=
d
d

y
2
+
d
2

d
2

y
2

y
2
=
d
d
cm m 1 y
m 2
t
n
+
d
2

d
2
c
2
m
2
y
2m2
t
2n
putting these derivatives into an equation we get

d
d
cny
m
t
n 1
=
d
d
cmy
m 1
t
n
+
d
d
cm m 1 y
m 2
t
n
+
d
2

d
2
c
2
m
2
y
2m 2
t
2n
We can divide by cy
m
t
n
so that we obtain

d
d
nt
1
=
d
d
my
1
+
d
d
m m 1 y
2
+
d
2

d
2
cm
2
y
2
Grouping we obtain a more compact form

d
d
nt
1
my
1
m m 1 y
2
=
d
2

d
2
cm
2
y
2
Note that things simplify if we pick m = 1. A bit of exercise will show that
the appropriate choice for is

=
y
4t
Lecture 8
ChE 333 5
Solution of the Differential Equation
The equation becomes and ordinary differential equation

2
d
d
+
d
2

d
2
= 0
The boundary conditions are

= 1 for
= 0 for = 0
The solution we have seen is related to an error function defined as

erf (x) =
1

e
t
2
dt
0

This solution affords us the opportunity to talk of an effective penetration


depth,
T
, that is, the distance at which the dimensionless temperature goes
from 0 to 0.99.
The solution for is = erf(), so that for the penetration depth

= 0.99 = erf

T
4t
It follows that

T
4t
= 2
This means that if
T
is less than the thickness of the slab, it behaves as a
semi-infinite region.
Lecture 8
ChE 333 6
Heat Conduction with a Convective Boundary Condition
The boundary condition at the cooling surface can have a major effect on
the process. The problem in this instance is posed as

T
t
=

2
T
y
2
at t = 0, T = T
0
at y = 0 , k
s
T
y
= h T T
a
as y , T = T
0
The problem can again be solved using combination of variables and the
same transformation as above to yield

= erf
y
4t
+ exp y + t erfc
y
4t
+ t

where y =
hy
k
s
and t =
h
k
s
2
t
Lecture 8
ChE 333 7
Surface temperature of a Cooling Sheet
Polyethylene is extruded and coated onto an insulated substrate, moving at
20 cm/sec. The molten polymer is coated at a uniform temperature T
0
of
400F. Cooling is achieved by blowing air at a temperature T
a
of 80F.
Earlier heat transfer studies determined that the heat transfer coefficient, h,
is 0.08 cal/cm-sec-F. The coating thickness B is 0.1 cm.
At what point downstream does the surface temperature, T(0) fall to 144
F ?
Data
T
0
= 400F h = 3.35 kW/m
2
-K B = 0.1 cm.
T
a
= 80F k
s
= 0.33 W/m-K = 1.3 10
-7
m
2
/sec
The Biot number can be estimated as:

Bi =
hB
k
s
=
3350 0.001
0.33
= 10.15
The dimensionless surface temperature ratiois

s
=
T 0 T
a
T
0
T
a
=
144 80
400 80
= 0.2
The Gurney-Lurie Chart 11.4c yields for Bi 10, the ratio of the surface
temperature to the mid-plane temperature
However, since

1
0
= 0.15
,
we can calculate the mid-plane temperature
from the relation for
s
which is

s
=
1
0

1
0
= 0.2
This gives a midplane-temperature of
1
0
= 0.2/0.15 > 1........Nonsense
Whats wrong ???
We did a lot of things wrong.
Lecture 8
ChE 333 8
First of all the solution we used involved only 1 term of an infinite series...

1
= A
1
e

1
2
x
Fo
sin
1
=
1
0
sin
1

We also get into trouble if we use such an equation for a short time
solution. Therefore avoid the charts for small x
Fo
and large Biot numbers.
The short time solution we presented in the last lecture had the form.

= erf
y
4t
+ exp y + t erfc
y
4t
+ t

where y =
hB
k
s
y
B
and t =
hB
k
s
2
t
B
2
Now for this case, y = 0 and we can use figure 11.3.2. We can determine
that the value of t at which = 0.2.
We observe that t
1/2
= 2.65 and consequently t = 7.65
Recall that C
p
= k/ = 2.5 MJ/m
2
-K.
This leads to

t =
hB
k
s
2
t
B
2
= 7.65 =
h
k
s
2
t
We calculate that the time passed is t = 0.52 seconds and since d = Vt, the
distance is d = (20 cm/s) 0.52 sec = 10.4 cm.
Lecture 8
ChE 333 9
An alternative method
We can use the complete Fourier expansion, not just one term.

=
4 sin
n
2
n
+ sin 2
n
cos
n
e

n
2
x
Fo

n =1

n
tan
n
= Bi
The first set of eigenvalues are
n ln
1 1.429
2 4.306
3 7.228
If we calculate the first three terms of the Fourier expansion, we obtain

(1) = 0.178e
2.04x
Fo
+ 0.155e
18.5x
Fo
For = 0.2, by trial and error, we obtain x
Fo
= 0.608. If we calculate the
time, we get 0.52 sec. The same as the short time solution.
This allows us a measure of short time. as for a slab

4
t
B
2
1 or x
Fo

1
16
Lecture 9
ChE 333 1
Heat Transfer in a Slab
Consider a large planar solid whose thickness (y-direction) is L. What is
the temperature history of the slab if it is suddenly brought into contact
with a fluid at temperature T? The transient conduction equation is
Lets make the problem dimensionless.
The temperature can be expressed as
so that the problem reposed is

T
t
=

2
T
y
2
at t = 0,T = T
0
at y = 0 , T = T
1
for t > 0
at y = L , T = T
1

=
T T
1
T
0
T
1
; =
y
L
; =
t
L
2

t
=

y
2
= 1 at = 0
= 0 at = 0
= 0 as = 1
Lecture 9
ChE 333 2
How do we solve the equation ?
Suppose z has the form = Y()G()
The equation is separable in the form
Integrating each of the equations we obtain
The solution for y(,) has the form
We can construct the exact solution using the boundary conditions
It follows that B must be 0 if the condition is true for all > 0

2
=
YG

2
YG

2
= G
dY
d
Y
d
2
G
d
2
= 0

1
Y
dY
d
=
1
G
d
2
G
d
2
=
2

dY
d
=
2
Y ;
d
2
G
d
2
=
2
G

Y() = Ke

and G() = Asin() +Bcos()



(, ) = Asin() +Bcos() e


(0, ) = Asin(0) +Bcos(0) e

= 0
Lecture 9
ChE 333 3
Now the other boundary condition
Now this is true for all > 0 if and only if sin() = 0
but sin() = 0 only where = n where n = 0, 1, 2, .....
This means there are a countable infinity of solutions so that
To obtain the coefficients A
n
, we need to use the initial condition.
To determine the coefficients, we can use the orthogonality properties of
the sine and cosine functions. (See Appendix)

sin(n)sin(m)d
1
1
=
0 for m n
for m= n

(1, ) = Asin() e

= 0

(, ) = e

A
n
sin(n)

n= 1


= 1 at < 0

z(, 0) = A
n
sin(n) = 1

n= 1

Lecture 9
ChE 333 4
We integrate
You might remember that the first sine integral is non-zero if and only if
n = m. Now the equation for A
n
is
The result for the definite integrals follow from what I gave above . It
follows that
We saw earlier that the solution can be described as:

(, 0)sin(m)d
0
1
= sin(m)d
0
1

A
n
sin(n)sin(m)d
0
1

n= 1

= sin(m)d
0
1

A
n
=
sin(n)d
0
1
sin
2
(n)d
0
1
=
sin(x)dx
0
n
sin
2
(x)dx
0
n

A
n
=
4

1
n
n +
1
2
Lecture 9
ChE 333 5
We have already examined how the sum converges. For > 0.2, only
one term suffices to describe the solution. We can look at many different
classes of problems. The general problem for transient heat transfer in a
slab is one posed as
The dimensionless form is:
The solution is of the form

(, ) =
4

1
2n + 1
e
2n +1
2

sin( 2n + 1 )

n = 0


T
t
=

2
T
y
2
at t = 0, T = T
0
at y = 0 , k
T
y
= h T T
1
for t > 0
at y =
L
2
,
T
y
= 0

C
n
=
4 sin
n
2
n
+ sin 2
n
and
n
tan
n
= Bi

(, ) = C
n
e

n
2

sin(
n
2
)

n = 0

t
=

y
2
= 1 at = 0

= Bi at = 0

= 0 as = 1
Lecture 9
ChE 333 6
Again the approximate solution is the one-term solution
This argument is the same for any transient 1-dimensional heat transfer
problems involving cylinders, planes or spheres.
Examples
Infinite Cylinder
The solution is
An approximate one-term solution is
Note that along the center line

(, ) C
1
e

1
2

sin(
1
2
)

(, ) = C
n
e

n
2

J
0
(
n
)

n = 0


C
n
=
2

n
J
1

n
2 J
0
2

n
+ J
1
2

n
and
n
J
1

n
J
0

n
= Bi


=
1

= 1 at = 0 in 0, 1

= Bi at = 1

= 0 at = 0

(, ) = C
1
e

1
2

J
0
(
1
)

0
(0, ) = C
1
e

1
2

Lecture 9
ChE 333 7
So that the simpler representation is

(, ) =
0
J
0
(
1
)
Lecture 9
ChE 333 8
Sphere
The solution is
The Approximate Solution
The center temperature is
So that the temperature can be expressed as


=
1

= 1 at = 0 in 0, 1

= Bi at = 1

= 0 at = 0

C
n
=
4 sin
n

n
cos
n
2
n
+ sin 2
n
and 1
n
cot
n
= Bi

(, ) = C
n
e

n
2

sin(
n
)

n = 0


(, ) = C
1
e

1
2

sin(
1
)

0
(0, ) = C
1
e

1
2


(, ) =
0
sin(
1
)

Lecture 9
ChE 333 9
Short Time Solutions
Consider a large planar solid whose extent (y-direction) is very large.
What is the temperature history of the slab if it is suddenly brought into
contact with a fluid at temperature T
a
? The transient conduction equation
is
Lets make the problem dimensionless.
The temperature can be expressed as

=
T T
a
T
0
T
a
so that the problem reposed is

T
t
=

2
T
y
2
at t = 0, T = T
0
at y = 0 , k
T
y
= h T T
a
T
a
as y , T = T
0

t
=

y
2
= 1 at t = 0

y
= Bi at y = 0
= 1 as y
Lecture 9
ChE 333 10
Solutions
We noted earlier that the equation can be solved by a combination of
variables supposing that = (,t) and we saw that the the appropriate
choice for is

=
y
4t
The solutions for a number of different cases are as follows:
Case 1 Constant Surface Temperature (T = T
s
)
Case 2 Constant Surface Heat Flux (q
s
= q
0
)
Case 3 Surface Convection

= erf
y
4t
+ exp y + t erfc
y
4t
+ t

T T
s
T
i
T
s
= erf
y
t

q
"
s
t =
k T
s
T
i
t

T T
s
=
2 q
"
0
t

k
e

y
2
4t
q
"
0
y
k
erfc
y
4t
Lecture 9
ChE 333 11

where y =
hy
k
s
and t =
h
k
s
2
t
Lecture 9
ChE 333 12
Surface temperature of a Cooling Sheet
Polyethylene is extruded and coated onto an insulated substrate, moving at
20 cm/sec. The molten polymer is coated at a uniform temperature T
0
of
400F. Cooling is achieved by blowing air at a temperature T
a
of 80F.
Earlier heat transfer studies determined that the heat transfer coefficient, h,
is 0.08 cal/cm-sec-F. The coating thickness B is 0.1 cm.
At what point downstream does the surface temperature, T(0) fall to 144
F ?
Data
T
0
= 400F h = 3.35 kW/m
2
-K B = 0.1 cm.
T
a
= 80F k
s
= 0.33 W/m-K = 1.3 10
-7
m
2
/sec
The Biot number can be estimated as:

Bi =
hB
k
s
=
3350 0.001
0.33
= 10.15
The dimensionless surface temperature ratio is

s
=
T 0 T
a
T
0
T
a
=
144 80
400 80
= 0.2
The Gurney-Lurie Chart 11.4c yields for Bi 10, the ratio of the surface
temperature to the mid-plane temperature
However, since

1
0
= 0.15
,
we can calculate the mid-plane temperature
from the relation for
s
which is

s
=
1
0

1
0
= 0.2
This gives a midplane-temperature of
1
0
= 0.2/0.15 > 1........Nonsense
Whats wrong ???
We did a lot of things wrong.
Lecture 9
ChE 333 13
First of all the solution we used involved only 1 term of an infinite series...

1
= A
1
e

1
2
x
Fo
sin
1
=
1
0
sin
1

We also get into trouble if we use such an equation for a short time
solution. Therefore avoid the charts for small x
Fo
and large Biot numbers.
The short time solution we presented in the last lecture had the form.

= erf
y
4t
+ exp y + t erfc
y
4t
+ t

where y =
hB
k
s
y
B
and t =
hB
k
s
2
t
B
2
Now for this case, y = 0 and we can use figure 11.3.2. We can determine
that the value of t at which = 0.2.
We observe that t
1/2
= 2.65 and consequently t = 7.65
Recall that C
p
= k/ = 2.5 MJ/m
2
-K.
This leads to

t =
hB
k
s
2
t
B
2
= 7.65 =
h
k
s
2
t
We calculate that the time passed is t = 0.52 seconds and since d = Vt, the
distance is d = (20 cm/s) 0.52 sec = 10.4 cm.
Lecture 9
ChE 333 14
An alternative method
We can use the complete Fourier expansion, not just one term.

=
4 sin
n
2
n
+ sin 2
n
cos
n
e

n
2
x
Fo

n =1

n
tan
n
= Bi
The first set of eigenvalues are
n

n
1 1.429
2 4.306
3 7.228
If we calculate the first three terms of the Fourier expansion, we obtain

(1) = 0.178e
2.04x
Fo
+ 0.155e
18.5x
Fo
For = 0.2, by trial and error, we obtain x
Fo
= 0.608. If we calculate the
time, we get 0.52 sec. The same as the short time solution.
This allows us a measure of short time. as for a slab

4
t
B
2
1 or x
Fo

1
16
Lecture 10 3/7/00
ChE 333 1
Convective Heat Transfer
Examples
1. Melt Spinning of Polymer fibers
2. Heat transfer in a Condenser
3. Temperature control of a Re-entry vehicle
Fiber spinning
The fiber spinning process presents a unique engineering problem,
primarily due to the effects of shape variations, heat and possibly the
viscoelastic behavior of the materials (polymers for example) typically
used. This becomes evident when the design of the spinneret geometry is
needed to produce a specified fiber size and shape. Determining the proper
die geometry given the desired final fiber shape is further complicated by
the heat and viscoelastic effects. In addition, since the fiber is pulled from
the spinneret, the final dimensions of the fiber are difficult to determine.
The effects of viscous heating and air cooling must be monitored to ensure
that the material does not degrade because of extreme local temperatures,
often difficult to measure because of the small size. The stresses and
deformation of the material must also be predicted to avoid the fiber from
breaking. All these effects complicate the design of the fiber spinning
process.
Lecture 10 3/7/00
ChE 333 2
Definition of a Heat Transfer Coefficient
For heat transfer in conducting systems, we have seen that we can express
the heat flux across a surface S as

q n
s
= k
s
T n
s
We have used a similar representation to develop detailed descriptions of
mass and transfer in a number of situations where the physics or chemistry
is well-understood, e.g., permeation through a membrane, heat transfer to a
sphere. We showed that we could get a description of the macroscopic
transfer across an interface by the use of a Heat Transfer Coefficient.

q n
s
= h T T
b
We do nor always have such a good model or understanding. The are
other equivalent physical situations, e.g., turbulent flow in a pipe. There we
use a measure of the frictional loss in the pipe as a momentum transfer
coefficient. The dimensionless form was the Friction Factor. The
dimensionless mass transfer coefficient is the Nusselt Number.

Nu =
hL
k
Lecture 10 3/7/00
ChE 333 3
Methods of Analysis
1. Detailed Solution of the Conservation Laws
2. Approximate Analysis
3. Dimensional Analysis
4. Empirical Correlation of Data
We have seen several examples of Detailed Solution and we have done
some Approximate Analysis, e.g., mass transfer to or from a flowing film,
heat transfer from a solid sphere. What we did was to transform the exact
problem into a simpler more solvable one using mathematical analysis.
What we do in the next few lectures is examine in greater detail the last
three methods as tools to analyze heat transfer and to design processes.
Approximate Analysis and Film Theory
Film Theory is the simplest and oldest approach in the use of mass transfer
coefficients and in their prediction. The Theory is attributed to Nernst.
Examine the neighborhood of the phase boundary. We assume that the
flow field consists of two regions, a uniform region in the bulk of the fluid
far from the surface and a region in the vicinity of the boundary where
viscosity dominates (since there is no slip at the boundary).
Lecture 10 3/7/00
ChE 333 4
Film Model
The film model presumes that the velocity field is linearized in some sense
near the boundary so that

=
v
x
y
S

U

=
F
s
A
This means that the film has a thickness


U

But recall the definition of the friction factor



f

1
2
U
2
If we introduce that notion into our analysis


U
1
2
fU
2
=
2
fU
Not surprisingly we can relate the fractional layer thickness to the
Reynolds number

L
=
2
fUL
=
2
f Re

f A Re

1
4
Lecture 10 3/7/00
ChE 333 5
Dimensionless Heat Transfer Coefficient
We defined the Heat Transfer Coefficient, h , by

q n
s
= k
s
T n
s
= h T T
b
so that the Nusselt number is given as

Nu
L
=
q n
s
k T T
b
=
hL
k
Observe that

Nu
L
=
hL
k
=
L

*
=
L

*
=
1
2
f Re

*
so that

Nu
L
=
1
2
f Re

*
= g(Pr) Pr
1/ 3
Then we observe a relation rather like the ones we calculated in our more
detailed models.

Nu
L
=
1
2
f ReSc
1/ 3
Lecture 10 3/7/00
ChE 333 6
The Chilton-Colburn Analogy
In the 1930s, based on the Nernst Film Theory, two duPont researchers
proposed an analogy between heat transfer (and we have seen, mass
transfer) and momentum transfer. They defined a dimensionless number
termed a j-factor, j
H
.

j
H
=
Nu
L
RePr
1/ 3
=
f
2
For mass transfer, the relation was

j
D

Sh
L
ReSc
1/ 3
=
f
2
Simple film theory, then, predicts that

Nu
L
=
1
2
f RePr
1/ 3
=
0.079
2
Re
0.25
RePr
1 / 3
or simplified

Nu
L
= 0.04 Re
0.75
Pr
1/ 3
Lecture 10 3/7/00
ChE 333 1
Convective Heat Transfer
Examples
1. Melt Spinning of Polymer fibers
2. Heat transfer in a Condenser
3. Temperature control of a Re-entry vehicle
Fiber spinning
The fiber spinning process presents a unique engineering problem,
primarily due to the effects of shape variations, heat and possibly the
viscoelastic behavior of the materials (polymers for example) typically
used. This becomes evident when the design of the spinneret geometry is
needed to produce a specified fiber size and shape. Determining the proper
die geometry given the desired final fiber shape is further complicated by
the heat and viscoelastic effects. In addition, since the fiber is pulled from
the spinneret, the final dimensions of the fiber are difficult to determine.
The effects of viscous heating and air cooling must be monitored to ensure
that the material does not degrade because of extreme local temperatures,
often difficult to measure because of the small size. The stresses and
deformation of the material must also be predicted to avoid the fiber from
breaking. All these effects complicate the design of the fiber spinning
process.
Lecture 10 3/7/00
ChE 333 2
Definition of a Heat Transfer Coefficient
For heat transfer in conducting systems, we have seen that we can express
the heat flux across a surface S as

q n
s
= k
s
T n
s
We have used a similar representation to develop detailed descriptions of
mass and transfer in a number of situations where the physics or chemistry
is well-understood, e.g., permeation through a membrane, heat transfer to a
sphere. We showed that we could get a description of the macroscopic
transfer across an interface by the use of a Heat Transfer Coefficient.

q n
s
= h T T
b
We do nor always have such a good model or understanding. The are
other equivalent physical situations, e.g., turbulent flow in a pipe. There we
use a measure of the frictional loss in the pipe as a momentum transfer
coefficient. The dimensionless form was the Friction Factor. The
dimensionless mass transfer coefficient is the Nusselt Number.

Nu =
hL
k
Lecture 10 3/7/00
ChE 333 3
Methods of Analysis
1. Detailed Solution of the Conservation Laws
2. Approximate Analysis
3. Dimensional Analysis
4. Empirical Correlation of Data
We have seen several examples of Detailed Solution and we have done
some Approximate Analysis, e.g., mass transfer to or from a flowing film,
heat transfer from a solid sphere. What we did was to transform the exact
problem into a simpler more solvable one using mathematical analysis.
What we do in the next few lectures is examine in greater detail the last
three methods as tools to analyze heat transfer and to design processes.
Approximate Analysis and Film Theory
Film Theory is the simplest and oldest approach in the use of mass transfer
coefficients and in their prediction. The Theory is attributed to Nernst.
Examine the neighborhood of the phase boundary. We assume that the
flow field consists of two regions, a uniform region in the bulk of the fluid
far from the surface and a region in the vicinity of the boundary where
viscosity dominates (since there is no slip at the boundary).
Lecture 10 3/7/00
ChE 333 4
Film Model
The film model presumes that the velocity field is linearized in some sense
near the boundary so that

=
v
x
y
S

U

=
F
s
A
This means that the film has a thickness


U

But recall the definition of the friction factor



f

1
2
U
2
If we introduce that notion into our analysis


U
1
2
fU
2
=
2
fU
Not surprisingly we can relate the fractional layer thickness to the
Reynolds number

L
=
2
fUL
=
2
f Re

f A Re

1
4
Lecture 10 3/7/00
ChE 333 5
Dimensionless Heat Transfer Coefficient
We defined the Heat Transfer Coefficient, h , by

q n
s
= k
s
T n
s
= h T T
b
so that the Nusselt number is given as

Nu
L
=
q n
s
k T T
b
=
hL
k
Observe that

Nu
L
=
hL
k
=
L

*
=
L

*
=
1
2
f Re

*
so that

Nu
L
=
1
2
f Re

*
= g(Pr) Pr
1/ 3
Then we observe a relation rather like the ones we calculated in our more
detailed models.

Nu
L
=
1
2
f ReSc
1/ 3
Lecture 10 3/7/00
ChE 333 6
The Chilton-Colburn Analogy
In the 1930s, based on the Nernst Film Theory, two duPont researchers
proposed an analogy between heat transfer (and we have seen, mass
transfer) and momentum transfer. They defined a dimensionless number
termed a j-factor, j
H
.

j
H
=
Nu
L
RePr
1/ 3
=
f
2
For mass transfer, the relation was

j
D

Sh
L
ReSc
1/ 3
=
f
2
Simple film theory, then, predicts that

Nu
L
=
1
2
f RePr
1/ 3
=
0.079
2
Re
0.25
RePr
1 / 3
or simplified

Nu
L
= 0.04 Re
0.75
Pr
1/ 3
Lecture 11
ChE 333 1
Exact Laminar Boundary Layer Theory
Heat Transfer from a Flat Plate
In a boundary layer, we have to describe the velocity field and the
temperature field
Conservation of Mass

u
x
x
+
u
y
y
= 0
Conservation of x-component of Linear Momentum

u
x
u
x
x
+ u
y
u
x
y
=

2
u
x
y
2
Conservation of y-component of Linear Momentum

P
y
= 0
Conservation of Energy

C
p
u
x
T
x
+ C
p
u
y
T
y
= k

2
T
y
2
The Boundary Conditions

u
x
= U, T = T

at x 0 or y
u
x
= 0, T = T
0
at y = 0 for all x
The Mass balance can be integrated to yield

u
y
=
u
x
x
dy
o
y
Lecture 11
ChE 333 2
If we introduce this relation in both the temperature and velocity equations
we obtain

u
x
u
x
x

u
x
x
dy
o
y
u
x
y
=

2
u
x
y
2

u
x
T
x

u
x
x
dy
o
y
T
y
=
k
C
p

2
T
y
2
Both equations are very similar and can be expressed as

u
x

u
x
x
dy
o
y

y
=

y
2
where this equation represents both conservation of x-momentum and
conservation of energy and
where for the x-momentum equation

=
u
x
U
; =
u
= 1
and where for the energy equation

=
T T
0
T

T
0
; =
T
=

= Pr
Recognize that we have constructed an equation that describes both the
momentum boundary layer and the energy boundary layer. Let us look
for solutions of that are functions of (x,y), that is,
We can define a combination of variables

=
y
2
U
x
1 / 2
Lecture 11
ChE 333 3
The equation for becomes an ordinary differential equation for = f().

2
u
d
?
?
d
dy
=
d
2

d
2
where

= 0 at = 0
= 1 as
If () is defined as

= 2
u
d
0

We can express the Boundary Layer equation as :




d
dy
=
d
2

d
2
This can be integrated twice to yield :

, =
e
d
0

d
0

e
d
0

d
0

Now since is a function of


u
. and (). We have to solve for
u
numerically, but once the function is known then all is known.
We can observe similarity for the temperature and momentum boundary
layers since that
u
. =

for Pr = 1.
Lecture 11
ChE 333 4
Solution for the Momentum Boundary Layer
The Momentum Boundary Layer equation can be expressed as


d
2

d
2
=
d
3

d
3
since

d
2

d
2
= 2
d
u
d

and


d
3

d
3
= 2
d
2

u
d
2
The boundary conditions are

at = 0
d
d
= 0
as
d
d
= 1
One method of solving the equation is to express the solution of the
equation as a power series in .

=

2
2!
+

5
5!
+ 11

8
8!
375

11
11!
+ ....
The equation has this form since the function and the first derivative vanish
at = 0 ( velocities vanish at the boundary ). The parameter must be
determined from the free stream behavior, e.g.

as
d
d
= 1
The value of which satisfies the free stream condition is 1.3282.
Lecture 11
ChE 333 5
Approximate Analytical Solution
For the Boundary Layer equation and from the solution given above, we
can determine that the solution is very near the free stream solution when
= 2.5.. If we define the boundary layer thickness where the value of


=

2
U
x
1/ 2
= 2.5
then we can see that

x
=
Ux

1/ 2
= Re
x
0.5
We can do the equivalent to determine a Thermal boundary layer
thickness from

k
T
y
y=0
= k
T
0
T

T
so that

T
x
= 2
Ux

0.5
d
T
d

From the solution above for

(0), we can determine that (0) = 0.664.


Recall the solution for

, =
e
d
0

d
0

e
d
0

d
0

Now


0, = e
d
0

d
0

1
= e

0.664
3

T
2

2
d
0

1
Lecture 11
ChE 333 6
The linear approximation for is within 10% of the exact solution.
We observe that
T
(0) = 0.68
T
1/3
, then we see

T
x
= 2.94
Ux

0.5
Pr
1/ 3
The corresponding Nusselt number relation is

Nu =
hx
k
= 0.34
Ux

1/ 2
Pr
1/ 3
Correlations for Heat Transfer Coefficients
Turbulent flow inside pipes
The simple film theory gives us a relation for the Chilton-Colburn Analogy
that for turbulent flow
j
H
= f / 2
where the j-factor was defined as j
H
= Nu Re
-1
Pr
-1/3
.
At high Reynolds numbers and Prandtl numbers, the Friend-Metzner
relation is useful.
Flow outside Pipes and Cylinders

Nu = 0.35 + 0.56 Re
0.52

St =
Nu
Re Pr
=
h
C
p
G
=
f
2
f
2
1.2 + 11.8
f
2
f
2
1/ 2
Pr 1 Pr
1 / 3
Lecture 11
ChE 333 1
Exact Laminar Boundary Layer Theory
Heat Transfer from a Flat Plate
In a boundary layer, we have to describe the velocity field and the
temperature field
Conservation of Mass

u
x
x
+
u
y
y
= 0
Conservation of x-component of Linear Momentum

u
x
u
x
x
+ u
y
u
x
y
=

2
u
x
y
2
Conservation of y-component of Linear Momentum

P
y
= 0
Conservation of Energy

C
p
u
x
T
x
+ C
p
u
y
T
y
= k

2
T
y
2
The Boundary Conditions

u
x
= U, T = T

at x 0 or y
u
x
= 0, T = T
0
at y = 0 for all x
The Mass balance can be integrated to yield

u
y
=
u
x
x
dy
o
y
Lecture 11
ChE 333 2
If we introduce this relation in both the temperature and velocity equations
we obtain

u
x
u
x
x

u
x
x
dy
o
y
u
x
y
=

2
u
x
y
2

u
x
T
x

u
x
x
dy
o
y
T
y
=
k
C
p

2
T
y
2
Both equations are very similar and can be expressed as

u
x

u
x
x
dy
o
y

y
=

y
2
where this equation represents both conservation of x-momentum and
conservation of energy and
where for the x-momentum equation

=
u
x
U
; =
u
= 1
and where for the energy equation

=
T T
0
T

T
0
; =
T
=

= Pr
Recognize that we have constructed an equation that describes both the
momentum boundary layer and the energy boundary layer. Let us look
for solutions of that are functions of (x,y), that is,
We can define a combination of variables

=
y
2
U
x
1 / 2
Lecture 11
ChE 333 3
The equation for becomes an ordinary differential equation for = f().

2
u
d
?
?
d
dy
=
d
2

d
2
where

= 0 at = 0
= 1 as
If () is defined as

= 2
u
d
0

We can express the Boundary Layer equation as :




d
dy
=
d
2

d
2
This can be integrated twice to yield :

, =
e
d
0

d
0

e
d
0

d
0

Now since is a function of


u
. and (). We have to solve for
u
numerically, but once the function is known then all is known.
We can observe similarity for the temperature and momentum boundary
layers since that
u
. =

for Pr = 1.
Lecture 11
ChE 333 4
Solution for the Momentum Boundary Layer
The Momentum Boundary Layer equation can be expressed as


d
2

d
2
=
d
3

d
3
since

d
2

d
2
= 2
d
u
d

and


d
3

d
3
= 2
d
2

u
d
2
The boundary conditions are

at = 0
d
d
= 0
as
d
d
= 1
One method of solving the equation is to express the solution of the
equation as a power series in .

=

2
2!
+

5
5!
+ 11

8
8!
375

11
11!
+ ....
The equation has this form since the function and the first derivative vanish
at = 0 ( velocities vanish at the boundary ). The parameter must be
determined from the free stream behavior, e.g.

as
d
d
= 1
The value of which satisfies the free stream condition is 1.3282.
Lecture 11
ChE 333 5
Approximate Analytical Solution
For the Boundary Layer equation and from the solution given above, we
can determine that the solution is very near the free stream solution when
= 2.5.. If we define the boundary layer thickness where the value of


=

2
U
x
1/ 2
= 2.5
then we can see that

x
=
Ux

1/ 2
= Re
x
0.5
We can do the equivalent to determine a Thermal boundary layer
thickness from

k
T
y
y=0
= k
T
0
T

T
so that

T
x
= 2
Ux

0.5
d
T
d

From the solution above for

(0), we can determine that (0) = 0.664.


Recall the solution for

, =
e
d
0

d
0

e
d
0

d
0

Now


0, = e
d
0

d
0

1
= e

0.664
3

T
2

2
d
0

1
Lecture 11
ChE 333 6
The linear approximation for is within 10% of the exact solution.
We observe that
T
(0) = 0.68
T
1/3
, then we see

T
x
= 2.94
Ux

0.5
Pr
1/ 3
The corresponding Nusselt number relation is

Nu =
hx
k
= 0.34
Ux

1/ 2
Pr
1/ 3
Correlations for Heat Transfer Coefficients
Turbulent flow inside pipes
The simple film theory gives us a relation for the Chilton-Colburn Analogy
that for turbulent flow
j
H
= f / 2
where the j-factor was defined as j
H
= Nu Re
-1
Pr
-1/3
.
At high Reynolds numbers and Prandtl numbers, the Friend-Metzner
relation is useful.
Flow outside Pipes and Cylinders

Nu = 0.35 + 0.56 Re
0.52

St =
Nu
Re Pr
=
h
C
p
G
=
f
2
f
2
1.2 + 11.8
f
2
f
2
1/ 2
Pr 1 Pr
1 / 3
Lecture 12
ChE 333 1
Design Problem
Superheater for a Polymer Solution
1
Ethylene-propylene rubber (EPR) is polymerized in a solvent. The
product of the reaction is a 6% (by weight) solution of EPR in
perchloroethylene. The polymer is recovered as "crumbs" from a drum
dryer. Production capacity is limited by the capacity of the dryer. It is
believed that concentrating the feed to the dryer will provide the sufficient
increase in capacity.
Your problem is to specify the design of a superheater that would
heat the solution sufficiently so that upon flashing to atmospheric pressure
the solution is concentrated to at least 12%.
Task 1 Determine all the physical properties to use in this problem
heat capacities, densities, chemical activity as a function of concentration
Task 2 Write a computer program or spreadsheet to calculate the
prescribed pressure and temperature required at the end of the superheater
Task 3 Write a computer program or spreadsheet to calculate the size
of the heat exchanger.
Data
The production of rubber is 300lbs./hr or 0.0379 kg/hr
The feed temperature is 35C
Note: This is an open-ended problem with insufficient
information given for you to solve the problem. You have to find the data
and make a reasonable set of assumptions about the fluid to be used to
heat the rubber.

1
NOTE This is an open ended, somewhat poorly defined, problem.
Lecture 12
ChE 333 2
Heat Transfer Analysis in Pipe Flow
Consider the problem of flow in a long pipe of circular cross-section. The
inside diameter of the pipe is D and is maintained at a constant
temperature T
o
. The fluid flow through the pipe at a flow rate, Q.
The goal is to describe the average temperature as a function of distance in
the pipe.
MODEL
Energy balance
C
p
u
z
T
z
k
1
r

r
r
T
r


_
,
+

2
T
z
2



1
]
1
Momentum balance
0 =
p
z
+ u
1
r

_
,
r
u
z
r

Initial and Boundary conditions
at z = 0 T = T
i
for all r
at r = 0
T
r
=
u
z
r
= 0
at r = R T = T
R
; u
z
= 0
Lecture 12
ChE 333 3
Adimensionalization and Scaling
The convective heat transfer equation
C
p
u
z
T
z
k
1
r

r
r
T
r


_
,
+

2
T
z
2



1
]
1
can be scaled using a set of reference parameters

=
T T
R
T
1
T
R
; =
z
L
; =
r
R
; v =
u
z
v
The equation is

C
p
v v T
1
T
R
L

=
k T
1
T
R
R
2
1

+
R
2
L
2

2
which after some multiplication becomes

v

=
kL
C
p
v R
2
1

+
R
2
L
2

2
It follows that if R/L is amall then only the first term in the Laplacian is
important and the equation can be written ignoring axial heat conduction.
Lecture 12
ChE 333 4
The dimensional form of the equation is:
C
p
u
z
T
z
= k
1
r

_
,
r
T
r
along with the boundary and initial conditions
T = T
1
at z = 0
T = T
R
at r = R
T
r
= 0 at r = 0
We could solve for the temperature profile in detail, but it might be better if we
seek a solution for the average temperature by integrating over the crossection at
position z.

0
R
C
p
u
z
T
z
r dr =

_
,
r
T
r
dr
0
R
If the velocity field is independent of axial position, we can write
C
p

z
u
z
Tr dr k
0
R

r
T
r


1
]
0
R
kR
T
r
R
0
Examine an average temperature <T>, the mixing cup temperature, the
mean temprature of the fluid that leaves cross section at z = z

T =
u
z
T 2r dr
0
R
u
z
2r dr
0
R
=
u
z
T 2r dr
0
R
Q
Lecture 12
ChE 333 5
This last equation can be re-written as

u
z
Tr dr
0
R
=
Q
2
T
The integrated energy equation is :
C
p
d
dz
Q
2
T


1
]
kR
T
r
R
The material balance teaches us that
Q = w = constant
So that we can write:
wC
p
d T
dz
k2R
T
r
R
Recall that we can define a heat transfer coefficient by an expression such
as :
k
T
r
R
h T T
R
[ ]
and the equation for the mixing cup temperature is :
wC
p
d T
dz
Dh T
R
T [ ]
Lecture 12
ChE 333 6
This can be prepared for integration
2
:
d T
R
T ( )
T
R
T

Dh
wC
p
dz
The relation is integrated readily if h is not a function of z
3
T T
R
T
1
T
R
exp
Dhz
wC
p



_
,
exp 4St
z
D


_
,
The definition of the Stanton Number is :
St
h
C
p
U

Nu
RePr

Nu
Pe
where Pe = Re Pr
T
2
T
R
T
1
T
R
exp 4St
L
D


_
,
exp
DLh
wC
p



_
,


2
From here on, ,we drop the brackets on T<>T> for convenience and the experinced players
benefit!!!
3
We can still use the same relation for St = h/C
p
U where

h
1
L
hdz
0
L
Lecture 12
ChE 333 7
Other Ways of Defining and using Heat Transfer Coefficients
Q
H
= hA(T)
Questions
What is Q
H
and what is T
We know that an energy balance contains:
Q
H
= +wC
p
(T
1
T
2
)
We can rewrite as
T +
wC
p
T
1
T
2
( )
DLh
After integrating we find that

DLh
wC
p
ln
T
2
T
R
T
1
T
R
It follows that I can write
Q
H
= hA(T)
if the temperature difference is T
T
T
2
T
1
ln
T
2
T
R
T
1
T
R



_
,

T
1
T
2
ln
T
1
T
R
T
2
T
R



_
,

T
1
T
R
( ) T
2
T
R
( )
ln
T
1
T
R
T
2
T
R



_
,

T
ln
Lecture 13 3/18/00
ChE 333 1
Parallel Plate Heat Exchanger
Parallel Plate Heat Exchangers are use in a number of thermal
processing applications. The characteristics are that the fluids flow in the
narrow gap, between two parallel sheets. The flow is usually laminar.
For our example, assume a fluid flows confined between two parallel
planes. both held at a fixed temperature, T
H
. The fluid enters at T
1
into the
heated section at a mean velocity U. The flow is laminar so that the
velocity profile is given by
u y ( )
3
2
U 1
y
H


_
,
2



1
]
1
The volumetric flow rate per unit width is given by
q
w
= 2UH
Reasonable assumptions include, Steady State, no viscous dissipation,
constant thermal properties, etc.
Lecture 13 3/18/00
ChE 333 2
Fluid in
laminar
flow
x
y = H
y = H
Uniform wall temperature T
H
x
Uniform inlet
temperature T
1
x=0
Temperature profile T(x,y)
y
Figure 12.5.1 Laminar flow between heated parallel planes
The convective energy balance is given by
u y ( )
T
x

2
T
y
2
+

2
T
x
2



_
,

As we saw in the case of a tubular channel, radial conduction is much
larger than axial conduction so the equation can be simplified to
u(y)
T
x

2
T
Dy
2
Boundary Conditions
T = T
1
at x = 0 for all y Initial temperature profile
T
y
0 at y 0 for all x Symmetry at the channel axis
T = T
H
at y H for all x 0 Wall temperature
Lecture 13 3/18/00
ChE 333 3
Non-Dimensional Form of the Equation
We can make the equation dimensionless with these definitions


T T
H
T
1
T
H
; y* =
y
H
; x*
x
UH
2
So that the equation and its boundary conditions become

3
2
1 y*
2
x*
=

y*
2
1 at x* 0 for all y*

y*
0 at y* 0 for all x*
0 at y* t1 for all x* 0
The solution of the equation can be expressed as a series solution

= A
n
exp
2
3

n
2
x* a
nm
y*
n

n= 0

m= 0

The heating rate per unit width of channel is


Q
H
= 2 U H [

C
p
(T
1
T
cm
)]
where T
cm
is the mixing cup average temperature.
Lecture 13 3/18/00
ChE 333 4
The mixing cup average obtained from the temperature profile is

cm

3
2
U 1
y
H
2
x, y dy
0
H
3
2
U 1
y
H
2
dy
0
H
It can, therefore, be written as

cm
G
m
exp
2
m
2
x*
3



_
,

m0

The first three coefficients and eigenvalues are:


m G
m

m
2
0 0.910 2.83
1 0.0533 32.1
2 0.0153 93.4
Everything we want to know about the temperature profile is in the
solution given above.
There is a simple one-term approximation since the second eigenvalues is
so much larger than the first.

cm
0.91 exp 1.89x* ( )
Lecture 13 3/18/00
ChE 333 5
The Local Heat Transfer Coefficient
The local heat transfer coefficient is defined in terms of the heat transferred
to the fluid in a differential distance along the exchanger.
dQ
H
= - UH C
p
dT
cm
= h
ln
(T
cm
- T
H
) dx
Recalling the non-dimensionalization, we can express the local heat
transfer coefficient, h
ln
, as
h
ln

UH

cm



_
,

k
UH
2


_
,
d
cm
dx *


_
,
From the solution for the dimensionless mixing cup temperature, we obtain
4h
ln
H
k
Nu
ln

8
3

m
2
G
m
exp 2
m
2
x */3
( )
m0

G
m
exp 2
m
2
x*/3
( )
m0

Here the Nusselt number is defined with a length scale 4 H, that is the
hydraulic diameter.
D
H
= 4 (cross-sectional are)/(wetted perimeter)
1
The one-term appoximation to
n
provides the limiting value for the
Nusselt number for large x*
Nu
ln

8
3

0
2
7.55
This is generally valid if x* > 0.1

1


Note that D
H
=
4 2HW
2W+ 4H
=
4H
1+
2H
W
Lecture 13 3/18/00
ChE 333 6
We will find shortly that, though the local Nusselt is useful, we will have
recourse to an average value over the length of the heat exchanger
Nu
L

1
x
L
*
Nu
ln
x * ( )
0
x
L
*

dx*
Though it would appear that we would have to return to the detailed
solution for
cm
, the energy balance yields us a simpler form to evaluate.
Nu
L

4
x
L
*
ln
1

cm
x
L
*
( )



_
,

Short Time Solutions
The equations describing the entrance region of the Parallel Plate Heat
Exchanger are identical save for the the midplane condition
u(y)
T
x

2
T
Dy
2
Boundary Conditions
T = T
1
at x = 0 for all y Initial temperature profile

T T
1
as y
Free stream condition
T = T
H
at y H for all x 0 Wall temperature
Lecture 13 3/18/00
ChE 333 7
Approximate Velocity Profile
In the entrance region, the thermal boundary layer is thin compared to the
width of the channel so that the velocity profile can be approximated by a
linear profile and the problem is then posed as

y
T
x

2
T
y
2
Boundary Conditions
T = T
1
at x = 0 for all y Initial temperature profile

T T
1
as y
Free stream condition
T = T
H
at y H for all x 0 Wall temperature
Here from an expansion of the velocity profile near the wall, = 3U/H.
The problem is identical in statement to the falling film problem discussed
in Mass Transfer and the solution is the same. We can use a combination
of variables

= y

9x
1/ 3
Thsolution for is

=
exp
3
d
0

exp
3
d
0

=
exp
3
d
0

4
3
Lecture 13 3/18/00
ChE 333 8
Nusselt Number Relations
The heat flux at the boundary is

q
y
x = k
dT
dy
y=0
= k T
H
T
1

9x*
1 / 3
The heat transferred per unit width is

Q
H
W
= k
dT
dy
y= 0
dx
0
L
=
3k T
H
T
1
2
4
3

9
1/ 3
so that the local heat transfer coefficient can be expressed as

h x =
q
y
T
H
T
1
= k

9x
1 / 3

4
3
The local Nusselt Number relation is

Nu(x) =
4h(x)H
k
=
4
UH
2
3x
1 / 3

4
3
=
3.12
x*
1/ 3
Expressed as an average Nusselt Number the relation becomes

Nu
L
=
4h
L
H
k
= 2.95 Re Pr
H
4L
1/ 3
Lecture 14 3/18/00
ChE 333 1
Correlations for Heat Exchange
Forced Convection Heat Transfer in Laminar Flow in a Tube
There are two measures of the state of a system in Heat Transfer...Reynolds
number and the Graetz number
Graetz Number Reynolds number
(local)

Gz =
D
x
Re Pr

Re =
UD

(average)

Gz =
D
L
Re Pr

Re =
UD

Developing Boundary layer with Fully Established Velocity


Profile
Uniform Wall Temperature
Local Heat Transfer Coefficient

Nu = 1.077 Gz
1/ 3
for Gz >100
Average Heat Transfer Coefficient

Nu = 1.61 Gz
1/ 3
for Gz >1000
Uniform Heat Flux
Local Heat Transfer Coefficient

Nu = 1.302 Gz
1/ 3
for Gz >10
4
Average Heat Transfer Coefficient

Nu = 1.953 Gz
1/ 3
for Gz >10
4
Lecture 14 3/18/00
ChE 333 2
Simultaneously Developing Temperature and Velocity Profiles

Nu = 1.86 Gz
1/ 3

w
0.14
for Gz

w
0.14
2
Fully Developed Laminar Forced Convection Heat Transfer
Constant Wall Temperature
Nu = 3.66
Constant Wall Heat Flux
Nu = 4.36
Overall Heat Transfer

Nu = 3.66
2
+ 1.61
3
Gz
1 / 3
Forced Convection Heat Transfer in Turbulent Pipe Flow
Heat Transfer in Entry Length

h
x
h
=
1
0.113 ln
x
D
+ 0.693
Fully Developed Forced Convection Heat Transfer
Colburn Equation

Nu = 0.023 Re
0.8
Pr
0.4
Dittus-Boelter Equation

Nu = 0.0225 Re
0.795
Pr
0.495
e
0.0225 ln Pr
2
ChE 333 - Lecture 15 1
Heat and Mass Transfer 3/18/00
Some Observations on Natural or Free Convection
Sometimes a single physical process in nature can explain a variety of events. Free convection is
one such process. It functions because heated fluids, due to their lower density, rise and cooled
fluids fall. A heated fluid will rise to the top of a column, radiate heat away and then fall to be re-
heated, rise and so on. Gasses, like our atmosphere, are fluids, too. A packet of fluid can become
trapped in this cycle. When it does, it becomes part of a convection cell.
Convection cells can form at all scales. They can be millimeters across or larger than Earth. They all
work the same way. The convection that you are most likely to have observed is in cumulonimbus
clouds or "thunderheads." These towering vertical clouds can be seen to evolve over a few minutes.
The tops of the clouds have a sort of cauliflower appearance as warm moist air rises through the
center of the cloud. The moisture in the cloud condenses as it cools. The air gives up some of its
heat to the cold high altitude air and begins to fall.
As the air falls along the exterior of the cloud, it returns to warmer low altitudes where it can be
caught up in the rising column of air in the center of the cloud. This fountain-like cell can form
alongside other cells, and a packet can move between cells. Hail forms when water droplets, carried
by the strong updrafts, freeze, fall through the cloud and are caught in the updraft again. An
additional layer of water freezes around the ice ball each time it makes a trip up through the cloud.
Eventually, the hail becomes too heavy to be carried up anymore, so it falls to the ground. Large
hailstones, when cut apart, show multiple layers, indicating the number of vertical trips the stone
made while it was caught in the convection cell.
Convection also occurs on the Sun. A high resolution white light image of the Sun shows a pattern
that looks something like rice grains. Very large convection cells cause this granulation. The bright
center of each cell is the top of a rising column of hot gas. The dark edges of each grain are the
cooled gas beginning its descent to be re-heated. These granules are the size of Earth and larger.
They constantly evolve and change.
Thunderheads and granulation are large-scale examples of convection. Fortunately, there are
examples of convection that fit into a classroom. An excellent example can be seen in hot Japanese
Miso (soybean paste) soup.
The interior of the broth is hot. The surface of the soup is exposed to cool air. Hot packets of fluid
rise out of the interior of the soup to the surface where they give off heat. Now cooled, they fall
down into the bowl to be re-heated. Left alone, the soup will dissipate its heat in this way (and
through conduction with the sides of the bowl) and reach room temperature.
The soybean paste granules and other ingredients will highlight the convection cells vividly. As
students gaze into their soup, they will see the rising and descending columns of fluid. The cells
will evolve and change their positions. Dark bottomed bowls show the effect best. If the soup is
stirred up, students can observe the cells reform. Of course, the demonstration material can be
consumed at the conclusion of the demonstration.
Free onvection acts as described in the examples above where gravity's effects are present (so that
warm, low density fluids can rise and cool, high density fluids can fall). What happens in the
weightlessness of space where up (rise) and down (fall) have no meaning?
ChE 333 - Lecture 15 2
Heat and Mass Transfer 3/18/00
How do We Describe Free convection?
Free convection is driven by density differences. In order to describe the
process, we can express the change in density as a function of temperatuer
in terms of a Taylors Series

T = T +

T
T
T T + ....
The coefficient of volumetric expansion is

=
1

T
T
so that the Taylors series can be expressed more simply.

T = T 1 + T T + ....
Role of buoyancy on the flow field
If there is a gravitational field, then there is a buoyancy force that acts on
the fluid to impart motion to the fluid. The induced motion would be
influenced by the viscosty or viscous drag. Such motion we term
natural convection or free convection.
In a column of fluid of height L, with a cross section A (normal to x-axis),
the body force acting across the ends of the column is BAL. If the body
force were unopposed by drag or viscous forces the momentum of the fluid
would be of order (u)
2
. The buoyancy driven velocity u
b
would be no
larger than

u
b
= O
gL

1 / 2
ChE 333 - Lecture 15 3
Heat and Mass Transfer 3/18/00
One can express the velocity in terms of a Reynolds Number

Re
b
=
u
b
L

=
gL
3

1 / 2
The natural number to describe natural convection is the Grashof Number,
a merasure of the buoyancy forces to inertial forces.

Gr =
gL
3

but recognizing that behaves


= T
leads to a simpler expression.

Gr =
gL
3

2
T
Buoyancy-induced Flow in a Confined Space
Suppose two vertical plates located 2b apart, One palte is heated and
maintained at T
1
The second plate is set at T
2
. Suppose as well that all the
physical properties are independent of temperature. The temperature field
does not change appreciably. Now the energy equation looks like L:

v
y
T
y
+ v
z
T
z
=

2
T
y
2
+

2
T
z
2
The velocity field is one dimensional so that

v 0,v
z
(y) = v
z
0
,v
z
0
(y)
ChE 333 - Lecture 15 4
Heat and Mass Transfer 3/18/00
Then the temperature equation simplifies to

d
2
T
dy
2
= 0 where
T = T
1
at y = b
T = T
2
at y = + b
and the solution can be expressed as

T = T
m

1
2
T
y
b

T = T
1
T
2
; T
m
=
1
2
T
1
+ T
2
The Navier-Stokes equation describing the flow field in the z-direction is

d
2
v
z
d y
2
=
dp
dz
+ g
For this mode, we make the Boussinesq approximation that is all
properties are T-independednt save density.

d
2
v
z
d y
2
=
dp
dz
+ g g T T
Now if we examine the perturbation expansion of v
z
applied to the N-S
eqaution, we observe that since

v
z
= v
z
0
+ v
z
1
The leading term of the perturbation expansion is the solution to

0 = =
dp
dz
+ g
That is, in the zeroth approximation, there is no flow in the z-direction as a result of
the temperature field... the pressure field is given by the hydrostatic law.... the
pressure field is balanced by the gravitational force.
However, in the first approximation, the velocity field is governesd ny the balance of
viscous forces and buoyancy forces.

d
2
v
z
1
d y
2
= g T T
ChE 333 - Lecture 15 5
Heat and Mass Transfer 3/18/00
The conservation equations are actually coupled, but in this first
approximation, we may treat them as uncoupled

d
2
v
z
1
d y
2
= g T
m
T
1
2
T
y
b
The boundary conditions are v
z
= 0 at y = - b and at y = +b
The solution is simple.

v
z
1
=
gb
2
T
12
y
b
3
A
y
b
2

y
b
+ A
where

A = 6
T
m
T
T
However, there should be no net flowhat is :

v
z
dy
b
b
= 0
This demands that A = 0, so

v
z
1
=
gb
2
T
12
y
b
3

y
b
We can make the velocity dimensionless

v
z
1
b

=
1
12
Gr
y
b
3

y
b
Gr is the Grashof number and is a measure of the buoyancy forces to the
viscous forces.
ChE 333 - Lecture 16 1
Heat Transfer 3/18/00
Buoyancy-induced Flow:
Natural Convection in a Unconfined Space
If we examine the flow induced by heat transfer from a single vertical flat
plat, we observe that the flow resembles that of a boundary layer. The
appropriate description begins with the conservation laws.
Mass


v
y
y
+
v
z
z
= 0
Momentum

v
y
v
z
y
+ v
z
v
z
z
=

2
v
z
y
2
+

2
v
z
z
2

p
z
g
Energy

v
y
T
y
+ v
z
T
z
=

2
T
y
2
+

2
T
z
2
The velocity field is such that the induced flow is viewed as a perturbation
of the steady flow.

v v
y
1
,v
z
0
(y) + v
y
1
Now if we examine the perturbation expansion of v
z
applied to the Navier-
Stokes equation, the leading term of the perturbation expansion is the
solution to

0 = =
dp
dz
+ g
That is, in the zeroth approximation, there is no flow in the z-direction as a result of
the temperature field... the pressure field is given by the hydrostatic law.... the
pressure field is balanced by the gravitational force.
ChE 333 - Lecture 16 2
Heat Transfer 3/18/00
Velocity Field
The velocity field is governed by.

v
y
v
z
y
+ v
z
v
z
z
=

2
v
z
y
2
+

2
v
z
z
2

p
z
g g T T
The conservation equations are clearly coupled. The solution mirrors the
results obtained in Boundary Layer theory analysis.
The boundary conditions are
at y = 0 v
z
= v
y
= 0 T = T
0
at y = v
z
= 0 T = T
1
at z = - v
z
= v
y
= 0 T = T
1
The solution is not simple, but involves combination of variables solutions
as in Boundary Layer theory. The problem is best solved by putting the
equations in dimensionless form.

Y =
H
B
1/ 4
; V
y
=
B
3
H
1/ 4
; V
z
=
BH

1/ 2
B = gT
ChE 333 - Lecture 16 3
Heat Transfer 3/18/00
Dimensionless Equations
Without belaboring the procedure in detail, the results in dimensionless
form are.
The differential equations

= 0

Pr
1

+
z

2
+
2

2
+

+
z

2
+
2

2
For sufficiently small , the equations are simpler. It simply means that
changes in the are less important than chasnges in the direction.

= 0

+
z

2

Pr
1

+
z

2
+
The boundary conditions are
at = 0
ez
=
y
= 0 = 1
at =
z
= 0 = 0
at = -
z
=
y
= 0 = 0
1
Heat Transfer Correlations
ChE 333 - Lecture 16 4
Heat Transfer 3/18/00
We can use the solution or at least the form of the solution of the
equations, as we note the definition of the heat flux per unit length.

q' = k
T
y
=0
dz
0
H
=
kTH
Y

=0
d
0
1
It is apparent that the definite integral is a function only of Pr since

= , , Pr
It follows that since the definite intergral is a constant C = C(Pr), that we
can express the flux q as

q' = CkT Gr Pr
1 / 4
where the Grashof number here is

Gr =

2
gH
3
T

2
It is easy to recall the definition of a heat transfer coefficient.

q'
H
= hT
and we obtain for our model

Nu = C Gr Pr
1/ 4
= CRa
1 / 4
ChE 333 - Lecture 16 5
Heat Transfer 3/18/00
An Empirical Correlation
Chu and Churchill developed a more useful quasi-empirical model

Nu
1/ 2
= Nu
1/ 2
+
Ra
L
300
Ra
L
300
1 +
0.5
Pr
0.5
Pr
9 /16
16/ 9
1/ 6
Geometry L Nu
0
Vertical Surface L 0.68
Vertical cylinder L 0.68
Horizontal cylinder D 0.36
Sphere D/2
Inclined disk 9D/11 0.56
ChE 333 - Lecture 16 6
Heat Transfer 3/18/00
An Example
Heat Loss from a Horizontal Pipe
An uninsulated pipe, 5 cm in outside diam., runs horizontally through a
laboratory maintained at 30C.. Air enter the pipe at 80C at a rate of 300
kg/hr. The pipe is 20 meters.. The down stream pressure in the pipe is 10
5
Pa gauge. What is the heat loss from the pipe and what is the exit
temperatrure of the air ?
Assume the principal resistance to heat transfer is on the outside of the
pipe. We assume as well that he external surface is 10C lower than the
inside of the pipe. (Can we get a better estimate ? How?
The physical parameters are
= 1.0 kg/m
3
; = 2 x 10
-5
kg/m-sec ; Pr = 0.7 ; C
p
= 1.005 kJ/kg-K
so that we evaluate the Rayleigh number as
Ra = GrPr = 350000
From the Chu-Churchill equation, we obtain Nu = 10, so that

h =
k
D
Nu = 5.8
watts
m
2
C
In order to calculate the heat loss we need to know the T

T
lm
= T
h
T
c
lm
We can make a first approximation that

T
lm
= T
h
T
c
lm
T
h
T
c
max
This allows us to get a quick estimate of the heat loss and the exit
temperature from the Heat Exchanger Design Equation.

w
air
C
p
T
in
h
T
out
h
= hAT
lm
ChE 333 - Lecture 16 7
Heat Transfer 3/18/00

T
in
h
T
out
h
=
hAT
lm
w
air
C
p
=
5.18 3.14 40
300 / 3600 1005
= 8.3C
That means that T ~ 36C so (T
in
- T
out
) = 7.5 C
ChE 333 - Lecture 17 1
Heat Transfer 3/18/00
Boiling Bubbles, Condensing Drops and Films:
Heat Transfer with a Phase Transition
It was clear in the last examination that many of you had little
understanding of what happens with heat transfer during a phase transition,
that is, in boiling, or in condensation. We saw that heat transfer during
condensation happens at a single temperature for a single component
system. The same is true of boiling. This makes analysis of condensers
and evaporators, and reboilers a bit easier than liquid liquid heat
exchangers. However, it isnt all that simple because there are lot of
pheniomenolgy in these processes.
Boiling
Ltes examine Boiling first. Imagine a horizontal heated flat plate of large
extent on which is a liquid at its boiling point. The temperature of the plate
is controlled.
What does the liquid look like
at very low heating rates ?
at moderate heating rates ?
at high heating rates ?
We measure the heat flux to the fluid as a function of the temperature
difference between the plate and the fluid. The heat flux from the plate at
temperature T
s
to the saturated fluid at T
sat
is described by

q n = h T
b
T
s
= h T
e
The term T
e
is the excess temperature, that is, the temperature difference
between the surface and the boiling liquid.
The Boiling process is characterized by the formation of vapor bubbles
which grow and detach from the surface. The character of the boiling we
observe depends largely on magnitude of the excess temperature.
ChE 333 - Lecture 17 2
Heat Transfer 3/18/00
Modes of Boiling
We observe three principal modes of boiling :
Free ConvectionBoiling
Nucleate Boiling
Film Boiling
There are of course, transitions between each regime. This is clear in the
following Boiling Curve
Free Convection Boiling occurs when T
e
T
e,A
~ 5C
During Free Convection Boiling, bubbles are formed on
isolated spots on the surface and are swept away by the
buoyancy forces ( free convection determines the motion).
Depending on whether the flow is laminar or tubulent, the heat
transfer coefficient varies as T
e
1/4
or T
e
1/3
and therefore the
heat flux qn is proportional to T
e
5/4
or T
e
4/3
.
The magnitude
is much larger than free convection in the absence of a phase
change.
ChE 333 - Lecture 17 3
Heat Transfer 3/18/00
Nucleate Boiling
Nucleate Boiling occurs when T
e,A
T
e
T
e,C
where T
e,C
~ 50C.
Vapor nucleates on the surface and the bubble rise towards the free
surface. Most of the heat exchange is directly from the surface to the fluid
with extensive mixing of the fluid. As T
e
rises, the bubble density
increases until a maximum concentration of bubbles occur as a
consequence of bubble coalescence. This corresponds to a maximum in
the heat flux . This Critical Heat Flux is in excess of 1 MW/m
2
.

q n
critical
~

24

V
H
fg
g
L

V

V
2
1/ 4

L
+
V

L
1/ 2
The Nucleate Boiling regime is characterized by extremely large heat
transfer coefficients, h, often larger than 10 kW/m
2
-K . As T
e
continues
to rise beyond the critical T
e
, the flux will decrease as the coaleschence
increases as the bubble collapse dominates.
Transition Boiling occurs when T
e,C
T
e
T
e,D
where T
e,D
~ 150C
ChE 333 - Lecture 17 4
Heat Transfer 3/18/00
Film Boiling
Film Boiling occurs when T
e
T
e,D
. Heat Transfer in Film

Boiling is
due to both radiation and convection.

h
4 / 3
= h
conv
4/ 3
+ h
rad
h
rad
1 / 3
where the radiative heat transfer coefficient is

h
rad
= 5.67x10
8

T
s
4
T
sat
4
T
e
T
sat
and the convective heat transfer coefficient is described by

h
conv
= 0.62
k
v
3

L

v
g H
v
+ 0.4C
p,v
T
e

v
DT
e
1 / 4
ChE 333 - Lecture 17 1
Heat Transfer 3/18/00
Boiling Bubbles, Condensing Drops and Films:
Heat Transfer with a Phase Transition
It was clear in the last examination that many of you had little
understanding of what happens with heat transfer during a phase transition,
that is, in boiling, or in condensation. We saw that heat transfer during
condensation happens at a single temperature for a single component
system. The same is true of boiling. This makes analysis of condensers
and evaporators, and reboilers a bit easier than liquid liquid heat
exchangers. However, it isnt all that simple because there are lot of
pheniomenolgy in these processes.
Boiling
Ltes examine Boiling first. Imagine a horizontal heated flat plate of large
extent on which is a liquid at its boiling point. The temperature of the plate
is controlled.
What does the liquid look like
at very low heating rates ?
at moderate heating rates ?
at high heating rates ?
We measure the heat flux to the fluid as a function of the temperature
difference between the plate and the fluid. The heat flux from the plate at
temperature T
s
to the saturated fluid at T
sat
is described by

q n = h T
b
T
s
= h T
e
The term T
e
is the excess temperature, that is, the temperature difference
between the surface and the boiling liquid.
The Boiling process is characterized by the formation of vapor bubbles
which grow and detach from the surface. The character of the boiling we
observe depends largely on magnitude of the excess temperature.
ChE 333 - Lecture 17 2
Heat Transfer 3/18/00
Modes of Boiling
We observe three principal modes of boiling :
Free ConvectionBoiling
Nucleate Boiling
Film Boiling
There are of course, transitions between each regime. This is clear in the
following Boiling Curve
Free Convection Boiling occurs when T
e
T
e,A
~ 5C
During Free Convection Boiling, bubbles are formed on
isolated spots on the surface and are swept away by the
buoyancy forces ( free convection determines the motion).
Depending on whether the flow is laminar or tubulent, the heat
transfer coefficient varies as T
e
1/4
or T
e
1/3
and therefore the
heat flux qn is proportional to T
e
5/4
or T
e
4/3
.
The magnitude
is much larger than free convection in the absence of a phase
change.
ChE 333 - Lecture 17 3
Heat Transfer 3/18/00
Nucleate Boiling
Nucleate Boiling occurs when T
e,A
T
e
T
e,C
where T
e,C
~ 50C.
Vapor nucleates on the surface and the bubble rise towards the free
surface. Most of the heat exchange is directly from the surface to the fluid
with extensive mixing of the fluid. As T
e
rises, the bubble density
increases until a maximum concentration of bubbles occur as a
consequence of bubble coalescence. This corresponds to a maximum in
the heat flux . This Critical Heat Flux is in excess of 1 MW/m
2
.

q n
critical
~

24

V
H
fg
g
L

V

V
2
1/ 4

L
+
V

L
1/ 2
The Nucleate Boiling regime is characterized by extremely large heat
transfer coefficients, h, often larger than 10 kW/m
2
-K . As T
e
continues
to rise beyond the critical T
e
, the flux will decrease as the coaleschence
increases as the bubble collapse dominates.
Transition Boiling occurs when T
e,C
T
e
T
e,D
where T
e,D
~ 150C
ChE 333 - Lecture 17 4
Heat Transfer 3/18/00
Film Boiling
Film Boiling occurs when T
e
T
e,D
. Heat Transfer in Film

Boiling is
due to both radiation and convection.

h
4 / 3
= h
conv
4/ 3
+ h
rad
h
rad
1 / 3
where the radiative heat transfer coefficient is

h
rad
= 5.67x10
8

T
s
4
T
sat
4
T
e
T
sat
and the convective heat transfer coefficient is described by

h
conv
= 0.62
k
v
3

L

v
g H
v
+ 0.4C
p,v
T
e

v
DT
e
1 / 4
Lecture 19
ChE 333 1
Heat Exchangers - Introduction
Concentric Pipe Heat Exchange
Energy Balance on Cold Stream (differential)

dQ
C
= wC
p
C
dT
C
= C
C
dT
C
Energy Balance on Hot Stream (differential)

dQ
H
= wC
p
H
dT
H
= C
H
dT
H
Overall Energy Balance (differential)
For an adiabatic heat exchanger, the energy lost to the surroundings is
zero so what is lost by one stream is gathered by the other.

dQ
C
+ dQ
H
= 0
T
c1
T
c2
T
h1
T
h1
Lecture 19
ChE 333 2
Heat Exchange Equation
It follows that the heat exchange from the hot to the cold is expressed in
terms of the temperature difference between the two streams.

dQ
H
= U T
H
T
C
dA
The proportionality constant is the Overall heat transfer coefficient
( discussion later)
Solution of the Energy Balances
The Energy Balance on the two streams provides a delation for the
differential temperature change.

dT
H
=
dQ
H
C
H
and dT
C
=
dQ
C
C
C
However, we should recall that we have an adiabatic heat exchanger so
that

d T =
dQ
H
C
H
1 +
C
H
C
C
Overall Energy balances on each stream
Hot Fluid

Q
H
= C
H
T
H1
T
H2
Cold fluid

Q
C
= C
C
T
C2
T
C1
Overall Energy balance on the Exchanger

Q
C
+ Q
H
= 0
Lecture 19
ChE 333 3
The equation for T can be modified using the overall energy balances to
yield

d T =
dQ
H
C
H
T
2
T
1
T
H1
T
H2
The denominator is the energy lost by the hot stream, so

d T =
dQ
H
Q
H
T
2
T
1
Application of the relation for energy transfer between the two streams
yields

d T =
UdAT
Q
H
T
2
T
1
Integration of the relation is the basis of a design equation for a heat
exchanger.

ln
T
2
T
1
=
UA
Q
H
T
2
T
1
Rearrangement of the equation leads to
The Design Equation for a Heat Exchanger

Q
H
= UA
T
2
T
1
ln
T
2
T
1
= UAT
lm
Lecture 19
ChE 333 4
Design of a Parallel Tube Heat Exchanger
The Exchanger
The Design Equation for a Heat Exchanger

Q
H
= UA
T
2
T
1
ln
T
2
T
1
= UAT
lm
Glycerin-water solution with a Pr = 50 (at 70 C) flows through a set of
parallel tubes that are plumbed between common headers. We must heat
this liquid from 20 C to 60C with a uniform wall temperature of 100 C.
The flow rate, F, is 0.002 m
3
/sec (31.6 gal/sec.).
How many parallel tubes are required ?
How do we select L and D for these tubes ?
Data
The heat capacity, C
p
, is 4.2 kJ/kg-K
The density, , is 1100 kg/m
3
The liquid has a kinematic viscosity, = 10
3
cm
2
/sec.
T
c1
T
c2
T
h1
T
h1
Lecture 19
ChE 333 5
Step 1
Calculate the heat load

Q
c
= FC
p
T
out
T
out

Q
c
= 1100
kg
m
3
0.002
m
3
sec
4.2
kJ
kgK
1K
1C
60 20 C

Q
c
= 369.6
kJ
sec
= 369.6 kWatts
Step 2
Calculate the heat transfer coefficient
If the flow is laminar, likely since glycerin is quite viscous,
and the Re < 2000
the Nusselt number relation for laminar flow can be expressed as

Nu = 3.66
3
+ 1.61
3
Gz
1 / 3
The Graetz number is

Gz = Re Pr
D
L
If the flow is turbulent (Re > 2000), the Nusselt numberr is given by

Nu = 0.023 Re
0.8
Pr
0.4
We do not know the flow per tube and therefore we do not know the Re.
However we dont need to know that. In Lecture 27 we observed for Heat
Transfer in a Tube that
T T
R
T
1
T
R
= exp
Dhz
wC
p





= exp 4St
z
D




Lecture 19
ChE 333 6
The definition of the Stanton Number is :
St =
h
C
p
U
=
Nu
RePr
=
Nu
Pe
Given a Re and Pr, we can calculate the Nu and the Stanton Number, the
latter prviding us with the temperature at length L from the previous
equation. Lets examine several configurations at L/D = 50, 100, 200.
The Excel table below can be used to specify a design chart.
Design Chart
Pr = 50 L/D =50
Re Nu St cm
1 3.7610 7.52E-02 0.0233
3 3.9482 2.63E-02 0.2682
6 4.1996 1.40E-02 0.4966
10 4.4940 8.99E-03 0.6380
20 5.0980 5.10E-03 0.7750
30 5.5852 3.72E-03 0.8301
100 7.7548 1.55E-03 0.9254
200 9.5962 9.60E-04 0.9532
500 12.8779 5.15E-04 0.9746
1000 16.1628 3.23E-04 0.9840
2000 20.3244 2.03E-04 0.9899
5000 100.1133 4.00E-04 0.9802
10000 174.3074 3.49E-04 0.9827
20000 303.4868 3.03E-04 0.9849
30000 419.7714 2.80E-04 0.9861
To obtain the numbers in the spreadsheet, we used the Nusselt number
relation for laminar flow expressed as

Nu = 3.66
3
+ 1.61
3
Gz
1 / 3
Lecture 19
ChE 333 7
and for turbulent flow as

Nu = 0.023 Re
0.8
Pr
0.4
Design Chart
0.0000
0.0000
0.0000
0.0001
0.0010
0.0100
0.1000
1.0000
1 10 100 1000 10000
Reynolds' Number
T
e
m
p
e
r
a
t
u
r
e
L/ D = 50
L/ D = 100
L/ D = 200
Lecture 19
ChE 333 8
Step 3
Calculate the Area required
Base case
D = 2 cm. and L = 100 D = 2 meters
For this case we observe that from the calculations for
cm
Reduced Temperature
Re L/D = 50 L/D = 100 L/D = 200
1 0.0233 0.0006 0.0000
3 0.2682 0.0789 0.0069
8.8 0.5000 0.3966 0.1718
10 0.6380 0.4387 0.2099
12 0.6800 0.4966 0.2682
12.3 0.6854 0.5042 0.2763
24.4 0.8040 0.6836 0.5017
50 0.8805 0.8073 0.6888
60 0.8945 0.8301 0.7254
70 0.9050 0.8473 0.7532
100 0.9254 0.8805 0.8073
200 0.9532 0.9254 0.8805
500 0.9746 0.9596 0.9358
1000 0.9840 0.9746 0.9596
2000 0.9899 0.9840 0.9746
5000 0.9802 0.9913 0.9862
6000 0.9809 0.9923 0.9878
8000 0.9819 0.9936 0.9899
9000 0.9824 0.9941 0.9907
We can observe that the flow rate per tube is given by

F
nt
=
F
n
t
so that the Reynolds number is

Re =
4F
Dn
t
Lecture 19
ChE 333 9
As a consequence we can observe that the total length of tubing is not
dependent on D alone but on othere considerations that might set a
condition for Re, e.g. a pressure drop limitation. Wv find that for this base
case, we find

n
t
L = A =
4F
Re
L
D
We find that
cm
= 0.5
L/D Re n
t
L n
t
50 8.8 14.47 14.46
100 12.3 20.70 10.35
200 24.4 20.87 5.21
Does it make sense?
Lecture 19
ChE 333 10
Maximum Cooling Capacity of an Exchanger of Fixed Area
Water is available for use as a coolant for an oil stream in a double-pipe
heat exchanger.
The flow rate of the water is 500 lb
m
/hr.
The heat exchanger has an area of 15 ft
2
.
The oil heat capacity, C
po
, is 0.5 BTU/lb-F
The overall heat transfer coefficient, U, is 50 BTU/hr-ft
2
-F
The initial temperature of the water, T
w0
, is 100F
The maximum temperature of the water is 210F
The initial temperature of the oil, T
w0
, is 250F
The minimum temperature of the oil, T
w0
, is 140F
Estimate the maximum flow rate of oil that may be cooled assuming a
fixed flow rate of water at 500 lb
m
/hr
There are two possible modes of operation
Co-current flow
Counter-current flow
Let us look at both cases
Co-current flow
Constraints

T
w
< 210 ; T
w
< T
o
; T
o
140
Energy balances
Oil

Q
o
= F
o
C
po
T
o1
T
o2
= F
o
0.5 250 T
o2
Water

Q
w
= F
w
C
pw
T
w1
T
w2

F
o
C
p0
T
o1
T
o2
= 500(1.0)(210 100) = 55,000 BTU / hr
Lecture 19
ChE 333 11
Recall the Design equation

Q
H
= UA
T
2
T
1
ln
T
2
T
1
= UAT
lm
Now the T
lm
is given by

T
lm
=
T
2
T
1
ln
T
2
T
1
=
Q
w
UA
=
55000
(50)(15)
= 73.3
Using the temperatures, we obtain T
0max
= 238.5 F
and from the heat balance for oil, we obtain

F
o
=
C
po
Q
o
T
o1
T
o2
=
0.5 250 238.5
55000
= 9560 lb / h
Counter-current Flow
Constraints

T
w
< 210 ; T
w
< T
o
; T
o
140
Energy balances
Oil

Q
o
= F
o
C
po
T
o1
T
o2
= F
o
0.5 250 T
o2
Water

Q
w
= F
w
C
pw
T
w1
T
w2
Lecture 19
ChE 333 12

F
o
C
p0
T
o1
T
o2
= 500(1.0)(210 100) = 55,000 BTU / hr
Recall the Design equation

Q
H
= UA
T
2
T
1
ln
T
2
T
1
= UAT
lm
Now the T
lm
is given by

T
lm
=
T
2
T
1
ln
T
2
T
1
=
Q
w
UA
=
55000
(50)(15)
= 73.3
Using the temperatures, we obtain T
0max
= 221 F
and from the heat balance for oil, we obtain the oil flow rate as 3800
lbm/hr.
I thought that countercurrent flow was supposed to be more efficient.
What is the problem ?
Lecture 20
ChE 333 1
Design of a Parallel Tube Heat Exchanger
The Exchanger
100F
Water: 70F
5 ft/s
Benzene
180 F
7500 lb/h
The Design Equation for a Heat Exchanger

Q
H
= UA
T
2
T
1
ln
T
2
T
1
= UAT
lm
Problem
Find the Required Length of a Heat Exchanger with Specified Flows:
Turbulent Flow in Both Streams
The design constraints are given in the schematic above. We show this as
a countercurrent configuration, but we will examine the cocurrent case as
well. The benzene flow is specified as a mass flow rate (in pound mass
units), and the water flow is given as a linear velocity. Heat transfer
coefficients are not provided; we will have to calculate them based on our
earlier discussions and the correlations presented in earlier lectures.
The inside tube is specified as "Schedule 401-14 inch steel."
Lecture 20
ChE 333 2
Pipe "schedules" are simply agreed-upon standards for pipe construction
that specify the wall thickness of the pipe. Perrys Handbook specifies the
following dimensions for
the inside pipe :
Schedule 40 1 1/4 pipe
D
o
= 1.66 in. = 0.138 ft. S
c
= D
2
/4 = 0.0104 ft
2
D
i
= 1.38 in = 0.115 ft. (cross-sectional area for flow)
the outside pipe :
Schedule 40 2 pipe
D
i
= 2.07 in = 0.115 ft.
To calculate the heat exchanger area, we must find A
o
= DL. We know
the diameter; what is the length ?
The Design Equation is
Q
h
= U
o
A
o
T
ln
The overall heat transfer coefficient, U
o
, is given by

U
o
=
1
r
o
1
r
o
h
o
+
ln r
o
/ r
i
k
+
1
r
ihi
1
We can write it as:

U
o
A
o
L
=
1
h
o
A
o / L
+
ln r
o
/ r
i
2k
+
1
h
i
A
i
/ L
1
= R
1
To evaluate the parameters of the problem, we need the physical and
thermal properties and conditions for flow in the system
T
b
= 140F
b
= 52.3 lb
m
/ft
3 C
p
= 0.45 BTU/lb-F

k
b
= 0.085 Btu / h ft F

Pr =
C
pb

k
b
= 5

b
= 0.39 cP =
0.39
1000
1
47.88
= 8.1 x 10
6
lb
f
s / ft
2
= 2.6 X 10
4
lb
m
/fts
Lecture 20
ChE 333 3
Internal Film Resistance
The Nusselt number on the inside of the inner pipe is given by the Dittus-
Boelter equation.

Nu =
h
i
D
i
k
b
= 0.023 Re
0.8
Pr
0.3
= 337
so that the film heat transfer coefficient
h
i
= 249 Btu/hft
2
F
The heat transfer area per unit length is

A
i
L
= (0.115) = 0.361 ft
2
/ft
so that the inner film resistance is :

h
i
A
i
L
1
= 249 (0.361)
1
= 0.011 h ft F / Btu
The other tube dimensions are
D
oi
= 0.138 ft and D
io
= 0.172 ft
Calculation of the Water Flow Rate
The hydraulic diameter is

D
eq
= 4
D
2
i,o
D
o,i
2
/ 4
D
i,o
+ D
o,i
= D
i,o
D
o,i
= 0.034 ft
Given the water velocity of 5 ft/s, we can solve for the water flow rate
W
water
= 9300 lb
m
/h
The Overall Energy balance
(wC
p
T)
benz
= (wC
p
T)
water
Solving for the outlet water temperature:
7500 (0.45) (100 180) = 9300 (1) (70 T
out
)
gives the exit temperature as:
T
out
= 99F
Lecture 20
ChE 333 4
External Film Resistance
The physical properties of the water must be estimated in order to
determine the film heat transfer coefficient in the annular shell. The
average water temperature T
b
is calculated as 84.7 F
= 0.8 cp k = 0.34 BTU/h-ft-F = 62.4 lb/ft
3
so that the Reynolds number can be calculated.

Re
VD
eq

=
62.4 lb
m
/ft
3
32.2 lb
m
f/lb
f
s
2
(5 ft / s) 0.034 ft
1.67 x 10
5
lb
f
s / ft
2
= 2 x 10
4
From the Dittus-Boelter equation, the Nusselt number is given as:
Nu = 0.023 Re
0.8
Pr
0.4
= 127
so that the external film coefficient, h
o
, is
h
o
= 1270 Btu/hft
2
F
The external area/length is

A
0
L
= (0.138) = 0.434 ft
2
/ ft
so that the external film resistance is

h
0
A
0
L
1
= 1270 (0.434)
1
= 0.00181hftF / Btu
Conduction Resistance
The last term in the equation for the overall heat transfer coefficient is

ln r
0
/ r
i
2k
= 0.00116 h ft F / Btu
Overall Heat Transfer Coefficient
The overall resistance is
(U)
1
= R = 0.011 + 0.00116 + 0.00181 + = 0.014
benzene wall water
Lecture 20
ChE 333 5
Log-Mean T

T
ln
=
(180 99) (100 70)
ln (81 / 30)
= 51F
Heat Load
Q
h
= wC
p
T
= 7500 (0.45) (180 - 100) = 2.7 x 10
5
Btu/h
Heating Rate/unit Length

Q
h
L
=
UA
L
T
ln
= (R)
1
T
ln
= 3640 Btu / h ft
Given the heat load, we can calculate the length of tubing so that

L =
Q
h
3640
=
2.7 x 10
5
3640
= 74 ft
The case we considered was countercurrent flow, but we noted in an
earlier example that in co-current flow we could be more fluid. Now is the
pipe longer or shorter ?
Lecture 20
ChE 333 6
A Co-current Flow Heat Exchanger
The Design Equation for a Heat Exchanger

Q
H
= UA
T
2
T
1
ln
T
2
T
1
= UAT
lm
The heat loads are identical, the Overall Resistances to heat transfer (UA)
-1
are no different since the film coefficients do not change, but the T
lm
are
different.
Counter current Co-current
T
1
(water) = 99 T
1
(water) = 70
T
1
(benzene) = 180 T
1
(benzene) = 180
T
2
(water) = 70 T
2
(water) = 99
T
2
(benzene) = 100 T
2
(benzene) = 100
T
1
= 81 T
1
= 110
T
2
= 30 T
2
= 1
T
ln
= 51 T
ln
= 23.2
L = 74 L = 163 ft
There are two observations to be made. First that the tube length required
for co-current flow is more than twice as long. Secondly that the approach
temperature for co-current flow becomes diminishingly small.
Lecture 20
ChE 333 7
Questions
Question 1.
To have a single concentric pipe heat exchanger 73 ft. long may be
impractical. Why ?
Question 2.
What are the alternatives and can you make a rapid evaluation of the their
requirements ?
Question 3.
What if we use more tubes, do I need more area ? How do I estimate the
number of tubes and the required area for a single pass heat exchanger.
Question 4.
If we use more tubes, should we specify the tubes to be smaller. Why?
How do we estimate the effect ?
Lecture 20
ChE 333 8
Question 1.
To have a single concentric pipe heat exchanger 73 ft. long may be
impractical. Why ?
Where do I put a 73 ft. piece of pipe ? Can I fold it up? Can I cut it into
shoter pieces and have them in parallel. ?
Question 2.
What are the alternatives and can you make a rapid evaluation of the their
requirements ?
One alternative is to cut the pipe into 12 equal length, place them in a
header and put a shell around the bundle of tubes.
Question 3.
If we use more tubes, do I need more area ? How do I estimate the number
of tubes and the required area for a single pass heat exchanger.
If we use N identical tubes, Re
new
= Re
old
/N since

Re =
UD

=
Q
4D
From the Dittus-Boelter equation we have
Nu = 0.023 Re
0.8
Pr
0.4
= 127
The internal film heat tarnsfer coefficient h
i
~ Q
If the new flow rate is Q/N then h
i
~ (1/ N
new
)
0.8
So that for 12 identical tubes h
inew
= 0.137 h
iold
The overall resistance is now
R = 0.011/(0.137) + 0.00181 + 0.00116 = 0.0833
The required length is L
new
/L
old
= R
new
/ R
old
/ = 0.0833/0.014 = 5.95
so that L
new
is 73(5.95) = 434.2 ft.
Lecture 20
ChE 333 9
Question 4.
If we use more tubes, should we specify the tubes to be smaller. Why?
How do we estimate the effect ?
When we introduced the use of multiple tubes, we decreased the Re to
significantly reduce the internal film resistance. We can then use similar
arguments in decreasing the tube diameter, but we have the following
consequences
1. we reduce the area/length for heat transfer.
2. we increase the Reynolds number and the heat transfer coefficient
3. we increase the pressure drop
4. we make it harder to clean
How do we do evaluate the trade-offs ?
Lecture 21
ChE 333 1
Effectiveness Concept for Heat Exchangers
The Design Equation for a Heat Exchanger

Q
H
= UA
T
2
T
1
ln
T
2
T
1
= UAT
lm
A typical problem in the analysis of a heat exchanger is the Performance
calculation. That is, we are asked , given inlet conditions to evaluate how
the exchanger performs, that is what are the outlet temperatures. With the
equation given above, the solution may be reached only by trial-and-error.
Effectiveness
An alternate approach lies in the notion of exchanger effectiveness, E.

E =
actual heat transfer
maximumpossible heat transfer
Overall Energy Balance
The actual heat transfer is given by the energy balance
(wC
p
T)
hot
= (wC
p
T)
cold
The maximum possible temperature rise is the difference between the
temperatures of the two entering streams (T
hin
- T
cin
). Which fluid
undergoes the maximum temperature rise ? Of course, it is the one with
the least heat capacitance
(wC
p
)
min
(T)
max
= (wC
p
)
max
(T)
min
It follows then that
Q
max
= (wC
p
)
min
(T
hin
- T
cin
)
max
Lecture 21
ChE 333 2
Definitions of Effectiveness
For the Double-Pipe Heat Exchanger there are four possible cases:
Co-Current Counter-Current
Hot Fluid minimum Case 1 Case 3
Cold Fluid minimum Case 2 Case 4
Case 1- Co-Current flow, hot fluid minimal

E =
wC
p
h
T
h1
T
h2
wC
p
h
T
h1
T
c1
=
T
h1
T
h2
T
h1
T
c1
Case 2- Co-Current flow, cold fluid minimal

E =
wC
p
c
T
c2
T
c1
wC
p
c
T
h1
T
c1
=
T
c2
T
c1
T
h1
T
c1
Case 3- Counter-Current flow, hot fluid minimal

E =
wC
p
h
T
h1
T
h2
wC
p
h
T
h1
T
c2
=
T
h1
T
h2
T
h1
T
c2
Case 4- Counter-Current flow, cold fluid minimal

E =
wC
p
c
T
c1
T
c2
wC
p
c
T
h1
T
c2
=
T
c1
T
c2
T
h1
T
c2
Lecture 21
ChE 333 3
Number of Transfer Units
Recall the definition of the ratio of thermal capacitances

R =
WC
p
c
WC
p
h
=
C
c
C
h
=
T
h1
T
h2
T
c2
T
c1
Also we can reexamine the Design Equation and rewrite it in the following
form:

ln
T
h2
T
c2
T
h1
T
c1
=
UA
C
c
1 + R
or

T
h2
T
c2
T
h1
T
c1
= e

UA
C
c
1+R
We need to express this temperature ratio in terms of the effectiveness, E.
A good deal of algebra leads for Case 1 to

E =
1 + e

UA
C
c
1+R
1 + R
For case 2 the equation is the relation is very similar and indeed would be
the same if R were replaced by R
min
= C
min
/C
max
.

E =
1 e

UA
C
min
1+R
min
1 + R
min
Lecture 21
ChE 333 4
For case 3 and case 4, the equation can be expressed as a single relation.

E =
1 e

UA
C
min
1 R
min
1 + R
min
e

UA
C
min
1 R
min
We can define a dimensionless group as the Number of Transfer Units
(NTU)

NTU =
UA
C
min
This whole concept can be extended to all kinds of exchanger
configurations, e.g.,.shell and tube with n tube passes and one shell pass; a
cross-flow exchanger.
Cross-flow Heat Exchangers
Types - Mixed
Un-Mixed
Example Automobile radiator
Cross-flow
Mixed/Unmixed Exchanger
unmixed stream- minimal stream

E = R 1 exp R 1 e
NTU
mixed stream- minimal stream

E = 1 exp R 1 e
RNTU
Lecture 21
ChE 333 5
Unmixed/Unmixed Exchanger
Cross-flow

E = 1 exp R NTU
0.22
exp R NTU
0.78
1
Lecture 22
ChE 333 1
Design of a 1/2 Heat Exchanger
The Device is a 1 Shell Pass and a 2 Tube Pass Exchanger
The Design Equation for a Heat Exchanger

Q
H
= UAF
T
2
T
1
ln
T
2
T
1
= UAFT
lm
F is the correction to the T for a non-ideal flow path. To determine this
for this exchanger , noting that it is at the same time a co-current and a
counter-current exchange, we have to solve some enrgy balances.
Overall Energy Balance
(wC
p
T)
hot
= (wC
p
T)
cold
This leads to a ratio of thermal capacitances

R =
WC
p
C
WC
p
H
=
C
C
C
H
=
T
H1
T
H2
T
C2
T
C1
Lecture 22
ChE 333 2
or it can be written as

R =
WC
p
tube
WC
p
shell
=
C
tube
C
shell
=
T
shell
T
tube
Shell Balance on a first section of tube (cold stream)

C
C
T
C
'
z
T
C
'
z+ z
= U T
C
'
T
H
dA
2
Shell Balance on a second section of tube (cold stream)

C
C
T
z
"
z
T
C
"
z+ z
= U T
H
T
C
" dA
2
Overall Shell Balance on a second section of tube (hot stream)

C
H
T
H
z
T
H
z+ z
= U T
H
T
C
" dA
2
+ U T
H
T
C
' dA
2
The corresponding differential equations.

C
C
U
dT
C
'
dA
=
T
H
T
C
'
2

C
C
U
dT
C
"
dA
=
T
H
T
C
"
2

C
H
dT
H
dA
= U
T
H
T
C
"
2
U
T
H
T
C
'
2
Lecture 22
ChE 333 3
If we normalize the distance term to

dn
UdA
C
C
In this representation the equations are easier to formulate:

dT
C
'
dn
=
T
H
T
C
'
2

dT
C
"
dn
=
T
H
T
C
"
2

1
R
dT
H
dn
=
T
C
"
T
H
2
+
T
C
'
T
H
2
Energy balance from z to L

C
H
T
H
T
Hz
= C
C
T
C
"
T
C
'
This becomes

1
R
T
H
T
Hz
= T
C
"
T
C
'
Lecture 22
ChE 333 4
Eliminate all the T
C
variables, from the equations and the overall energy
balance, we obtain

dT
C
"
dn
=
dT
C
'
dn
+
1
R
dT
H
dn
=
T
H
2
+
T
C
'
+
1
R
T
H
T
Hz
2
and

T
C
'
T
H
2
+
1
R
dT
H
dn
=
T
H
2
+
T
C
'
+
1
R
T
H
T
Hz
2
The final form of the temperature equation is

1
R
d
2
T
H
dn
2
+
dT
H
dn

1
4R
T
H
T
Hz
= 0
Boundary Conditions
T
H
= T
H1
at n = 0
T
H
= T
H2
at n = n
L
where n
L
= UA/C
C
If we set a dimensionless T
H
, we obtain

=
T
H
T
H2
T
H1
T
H2
and the equation

1
R
d
2

dn
2
+
d
dn

1
4R
= 0
Boundary Conditions
=
1
at n = 0
=
2
at n = n
L
Lecture 22
ChE 333 5
The solution requires algebraic gymnastics, but it produces

F =
R
2
+ 1
R 1
ln
1 P
1 PR
ln
2
P
1 R R
2
+ 1
2
P
1 R + R
2
+ 1
where


P =
T
C2
T
C1
T
H1
T
C1
T
hermal design of shell-and-tube
heat exchangers (STHEs) is
done by sophisticated computer
software. However, a good un-
derstanding of the underlying principles
of exchanger design is needed to use this
software effectively.
This article explains the basics of ex-
changer thermal design, covering such
topics as: STHE components; classica-
tion of STHEs according to construction
and according to service; data needed for
thermal design; tubeside design; shellside
design, including tube layout, baffling,
and shellside pressure drop; and mean
temperature difference. The basic equa-
tions for tubeside and shellside heat
transfer and pressure drop are well-
known; here we focus on the application
of these correlations for the optimum de-
sign of heat exchangers. A followup arti-
cle on advanced topics in shell-and-tube
heat exchanger design, such as allocation
of shellside and tubeside uids, use of
multiple shells, overdesign, and fouling,
is scheduled to appear in the next issue.
Components of STHEs
It is essential for the designer to have a
good working knowledge of the mechani-
cal features of STHEs and how they in-
uence thermal design. The principal
components of an STHE are:
shell;
shell cover;
tubes;
channel;
channel cover;
tubesheet;
baffles; and
nozzles.
Other components include tie-rods and
spacers, pass partition plates, impinge-
ment plate, longitudinal baffle, sealing
strips, supports, and foundation.
The Standards of the Tubular Ex-
changer Manufacturers Association
(TEMA) (1) describe these various com-
ponents in detail.
An STHE is divided into three parts:
the front head, the shell, and the rear
head. Figure 1 illustrates the TEMA
nomenclature for the various construction
possibilities. Exchangers are described by
the letter codes for the three sections
for example, a BFL exchanger has a bon-
net cover, a two-pass shell with a longitu-
dinal baffle, and a xed-tubesheet rear
head.
Classication
based on construction
Fixed tubesheet. A xed-tubesheet
heat exchanger (Figure 2) has straight
tubes that are secured at both ends to
tubesheets welded to the shell. The con-
struction may have removable channel
covers (e.g., AEL), bonnet-type channel
covers (e.g., BEM), or integral tubesheets
(e.g., NEN).
The principal advantage of the xed-
tubesheet construction is its low cost be-
cause of its simple construction. In fact,
the xed tubesheet is the least expensive
construction type, as long as no expan-
sion joint is required.
Other advantages are that the tubes can
be cleaned mechanically after removal of
SHELL- AND- TUBE HEAT EXCHANGERS
CHEMICAL ENGINEERING PROGRESS FEBRUARY 1998

Copyright 1997 American Institute of Chemical Engineers. All rights reserved. Copying and downloading permitted with restrictions.
Effectively Design
Shell-and-Tube
Heat Exchangers
Rajiv Mukherjee,
Engi neers Indi a Lt d.
To make the most
of exchanger
design software,
one needs to
understand STHE
classication,
exchanger
components, tube
layout, baffling,
pressure drop, and
mean temperature
difference.
CHEMICAL ENGINEERING PROGRESS FEBRUARY 1998
SHELL- AND- TUBE HEAT EXCHANGERS
s Figure 1. TEMA designations for shell-and-tube heat exchangers.
E
F
G
H
J
K
X
One-Pass Shell
Two-Pass Shell
with Longitudinal Baffle
Split Flow
Double Split Flow
Divided Flow
Cross Flow
Kettle-Type Reboiler
A
B
Removable Channel and Cover
C
N
Bonnet (Integral Cover)
Integral With Tubesheet
Removable Cover
D
Special High-Pressure Closures
T
U
W
U-Tube Bundle
Pull-Through Floating Head
Floating Head with Backing Device
S
P
N
Outside Packed Floating Head
Fixed Tube Sheet
Like "C"Stationary Head
Fixed Tube Sheet
Like "B"Stationary Head
Externally Sealed
Floating Tubesheet
Fixed Tube Sheet
Like "A"Stationary Head
Stationary Head Types Shell Types Rear Head Types
M
L
Channel Integral With Tubesheet
and Removable Cover
the channel cover or bonnet, and that
leakage of the shellside uid is mini-
mized since there are no anged joints.
A disadvantage of this design is
that since the bundle is xed to the
shell and cannot be removed, the out-
sides of the tubes cannot be cleaned
mechanically. Thus, its application is
limited to clean services on the shell-
side. However, if a satisfactory chem-
ical cleaning program can be em-
ployed, xed-tubesheet construction
may be selected for fouling services
on the shellside.
In the event of a large differential
temperature between the tubes and
the shell, the tubesheets will be un-
able to absorb the differential stress,
thereby making it necessary to incor-
porate an expansion joint. This takes
away the advantage of low cost to a
signicant extent.
U-tube. As the name implies, the
tubes of a U-tube heat exchanger
(Figure 3) are bent in the shape of a
U. There is only one tubesheet in a U-
tube heat exchanger. However, the
lower cost for the single tubesheet is
offset by the additional costs incurred
for the bending of the tubes and the
somewhat larger shell diameter (due
to the minimum U-bend radius), mak-
ing the cost of a U-tube heat ex-
changer comparable to that of a xed-
tubesheet exchanger.
The advantage of a U-tube heat
exchanger is that because one end is
free, the bundle can expand or con-
tract in response to stress differen-
tials. In addition, the outsides of the
tubes can be cleaned, as the tube bun-
dle can be removed.
The disadvantage of the U-tube
construction is that the insides of the
tubes cannot be cleaned effectively,
since the U-bends would require ex-
ible-end drill shafts for cleaning.
Thus, U-tube heat exchangers should
not be used for services with a dirty
uid inside tubes.
Floating head. The oating-head
heat exchanger is the most versatile
type of STHE, and also the costliest.
In this design, one tubesheet is xed
relative to the shell, and the other is
free to oat within the shell. This
permits free expansion of the tube
bundle, as well as cleaning of both
the insides and outsides of the tubes.
Thus, oating-head SHTEs can be
used for services where both the
shellside and the tubeside uids are
dirty making this the standard con-
struction type used in dirty services,
such as in petroleum reneries.
There are various types of oat-
ing-head construction. The two most
common are the pull-through with
backing device (TEMA S) and pull-
through (TEMAT) designs.
The TEMA S design (Figure 4) is
the most common conguration in
the chemical process industries (CPI).
The oating-head cover is secured
against the oating tubesheet by bolt-
ing it to an ingenious split backing
ring. This oating-head closure is lo-
cated beyond the end of the shell and
contained by a shell cover of a larger
diameter. To dismantle the heat ex-
changer, the shell cover is removed
rst, then the split backing ring, and
then the oating-head cover, after
which the tube bundle can be re-
moved from the stationary end.
In the TEMA T construction (Fig-
ure 5), the entire tube bundle, includ-
ing the oating-head assembly, can
be removed from the stationary end,
since the shell diameter is larger than
the oating-head ange. The oating-
head cover is bolted directly to the
oating tubesheet so that a split back-
ing ring is not required.
The advantage of this construction
is that the tube bundle may be re-
moved from the shell without remov-
ing either the shell or the oating-
head cover, thus reducing mainte-
nance time. This design is particular-
ly suited to kettle reboilers having a
dirty heating medium where U-tubes
cannot be employed. Due to the en-
larged shell, this construction has the
highest cost of all exchanger types.
FEBRUARY 1998 CHEMICAL ENGINEERING PROGRESS
Support
Bracket
Stationary
Tubesheet
Stationary
Tubesheet
Bonnet
(Stationary
Head)
Bonnet
(Stationary
Head)
Baffles Tie Rods
and Spacers
s Figure 2. Fixed-tubesheet heat exchanger.
Tubeplate Shell Tubes Baffles Header
s Figure 3. U-tube heat exchanger.
There are also two types of packed
oating-head construction outside-
packed stuffing-box (TEMA P) and
outside-packed lantern ring (TEMA
W) (see Figure 1). However, since
they are prone to leakage, their use is
limited to services with shellside u-
ids that are nonhazardous and non-
toxic and that have moderate pres-
sures and temperatures (40 kg/cm
2
and 300C).
Classication
based on service
Basically, a service may be single-
phase (such as the cooling or heating
of a liquid or gas) or two-phase (such
as condensing or vaporizing). Since
there are two sides to an STHE, this
can lead to several combinations of
services.
Broadly, services can be classied
as follows:
single-phase (both shellside and
tubeside);
condensing (one side condens-
ing and the other single-phase);
vaporizing (one side vaporizing
and the other side single-phase); and
condensing/vaporizing (one side
condensing and the other side
vaporizing).
The following nomenclature is
usually used:
Heat exchanger: both sides single-
phase and process streams (that is,
not a utility).
Cooler: one stream a process uid
and the other cooling water or air.
Heater: one stream a process uid
and the other a hot utility, such as
steam or hot oil.
Condenser: one stream a condens-
ing vapor and the other cooling water
or air.
Chiller: one stream a process
fluid being condensed at sub-atmo-
spheric temperatures and the other a
boiling refrigerant or process stream.
Reboiler: one stream a bottoms
stream from a distillation column and
the other a hot utility (steam or hot
oil) or a process stream.
This article will focus specically
on single-phase applications.
Design data
Before discussing actual thermal
design, let us look at the data that
must be furnished by the process li-
censor before design can begin:
1. ow rates of both streams.
2. inlet and outlet temperatures of
both streams.
3. operating pressure of both
streams. This is required for gases,
especially if the gas density is not
furnished; it is not really necessary
for liquids, as their properties do not
vary with pressure.
4. allowable pressure drop for
both streams. This is a very important
parameter for heat exchanger design.
Generally, for liquids, a value of
0.50.7 kg/cm
2
is permitted per shell.
Ahigher pressure drop is usually war-
ranted for viscous liquids, especially
in the tubeside. For gases, the allowed
value is generally 0.050.2 kg/cm
2
,
with 0.1 kg/cm
2
being typical.
5. fouling resistance for both
streams. If this is not furnished, the
designer should adopt values speci-
ed in the TEMA standards or based
on past experience.
6. physical properties of both
streams. These include viscosity,
thermal conductivity, density, and
specic heat, preferably at both inlet
and outlet temperatures. Viscosity
data must be supplied at inlet and
outlet temperatures, especially for
liquids, since the variation with tem-
perature may be considerable and is
irregular (neither linear nor log-log).
7. heat duty. The duty specied
should be consistent for both the
shellside and the tubeside.
8. type of heat exchanger. If not
furnished, the designer can choose
this based upon the characteristics of
the various types of construction de-
scribed earlier. In fact, the designer is
normally in a better position than the
process engineer to do this.
9. line sizes. It is desirable to
match nozzle sizes with line sizes to
avoid expanders or reducers. Howev-
er, sizing criteria for nozzles are usu-
ally more stringent than for lines, es-
pecially for the shellside inlet. Conse-
quently, nozzle sizes must sometimes
be one size (or even more in excep-
tional circumstances) larger than the
corresponding line sizes, especially
for small lines.
10. preferred tube size. Tube size
is designated as O.D. thickness
length. Some plant owners have a
preferred O.D. thickness (usually
based upon inventory considerations),
and the available plot area will deter-
mine the maximum tube length.
Many plant owners prefer to stan-
dardize all three dimensions, again
based upon inventory considerations.
11. maximum shell diameter. This
is based upon tube-bundle removal re-
quirements and is limited by crane ca-
pacities. Such limitations apply only to
exchangers with removable tube bun-
dles, namely U-tube and oating-head.
For xed-tubesheet exchangers, the
only limitation is the manufacturers
fabrication capability and the avail-
ability of components such as dished
ends and anges. Thus, oating-head
heat exchangers are often limited to a
shell I.D. of 1.41.5 m and a tube
length of 6 m or 9 m, whereas xed-
tubesheet heat exchangers can have
shells as large as 3 m and tubes
lengths up to 12 m or more.
12. materials of construction. If
the tubes and shell are made of iden-
tical materials, all components should
be of this material. Thus, only the
shell and tube materials of construc-
tion need to be specied. However, if
the shell and tubes are of different
metallurgy, the materials of all princi-
pal components should be specied
to avoid any ambiguity. The principal
components are shell (and shell
cover), tubes, channel (and channel
cover), tubesheets, and baffles.
Tubesheets may be lined or clad.
13. special considerations. These
include cycling, upset conditions, al-
ternative operating scenarios, and
whether operation is continuous or
intermittent.
Tubeside design
Tubeside calculations are quite
straightforward, since tubeside ow
CHEMICAL ENGINEERING PROGRESS FEBRUARY 1998
SHELL- AND- TUBE HEAT EXCHANGERS
represents a simple case of ow
through a circular conduit. Heat-trans-
fer coefficient and pressure drop both
vary with tubeside velocity, the latter
more strongly so. A good design will
make the best use of the allowable
pressure drop, as this will yield the
highest heat-transfer coefficient.
If all the tubeside uid were to
ow through all the tubes (one tube
pass), it would lead to a certain veloc-
ity. Usually, this velocity is unaccept-
ably low and therefore has to be in-
creased. By incorporating pass parti-
tion plates (with appropriate gasket-
ing) in the channels, the tubeside uid
is made to ow several times through
a fraction of the total number of tubes.
Thus, in a heat exchanger with 200
tubes and two passes, the uid ows
through 100 tubes at a time, and the
velocity will be twice what it would
be if there were only one pass. The
number of tube passes is usually one,
two, four, six, eight, and so on.
Heat-transfer coefficient
The tubeside heat-transfer coeffi-
cient is a function of the Reynolds
number, the Prandtl number, and
the tube diameter. These can be bro-
ken down into the following funda-
mental paramet ers: physi cal
propert i es (namely viscosity, ther-
mal conductivity, and specific heat);
tube diameter; and, very important-
ly, mass velocity.
The variation in liquid viscosity is
quite considerable; so, this physical
property has the most dramatic effect
on heat-transfer coefficient.
The fundamental equation for tur-
bulent heat-transfer inside tubes is:
Nu = 0.027 (Re)
0.8
(Pr)
0.33
(1a)
or
(hD/k) =
0.027 (DG/)
0.8
(c/k)
0.33
(1b)
Rearranging:
h = 0.027(DG/)
0.8
(c/k)
0.33
(k/D) (1c)
Viscosity inuences the heat-trans-
fer coefficient in two opposing ways
as a parameter of the Reynolds
number, and as a parameter of Prandtl
number. Thus, from Eq. 1c:
h ()
0.330.8
(2a)
h ()
0.47
(2b)
In other words, the heat-transfer
coefficient is inversely proportional
to viscosity to the 0.47 power. Simi-
larly, the heat-transfer coefficient is
directly proportional to thermal con-
ductivity to the 0.67 power.
These two facts lead to some inter-
esting generalities about heat transfer.
Ahigh thermal conductivity promotes
a high heat-transfer coefficient. Thus,
cooling water (thermal conductivity
of around 0.55 kcal/hmC) has an
extremely high heat-transfer coeffi-
cient of typically 6,000 kcal/hm
2
C,
followed by hydrocarbon liquids
(thermal conductivity between 0.08
and 0.12 kcal/hmC) at 2501,300
kcal/hm
2
C, and then hydrocarbon
gases (thermal conductivity between
0.02 and 0.03 kcal/hmC) at
50500 kcal/hm
2
C.
Hydrogen is an unusual gas, be-
cause it has an exceptionally high
thermal conductivity (greater than
that of hydrocarbon liquids). Thus,
its heat-transfer coefficient is to-
ward the upper limit of the range
for hydrocarbon liquids.
The range of heat-transfer coeffi-
cients for hydrocarbon liquids is
FEBRUARY 1998 CHEMICAL ENGINEERING PROGRESS
Stationary-Head
Channel
Stationary
Tubesheet Shell
Support
Saddles
Floating
Tubesheet
Floating-Head
Cover
Shell
Cover
Tie Rods
and Spacers
Pass
Partition
Baffles
s Figure 4. Pull-through oating-head exchanger with backing device (TEMA S).
Shell
Weir
Support
Saddle Baffles
Support
Saddle
Floating
Tubesheet
Floating-Head
Cover
Shell
Cover
Stationary-Head
Channel
Tie Rods
and Spacers
Pass
Partition
s Figure 5. Pull-through oating-head exchanger (TEMA T).
rather large due to the large variation
in their viscosity, from less than 0.1
cP for ethylene and propylene to more
than 1,000 cP or more for bitumen.
The large variation in the heat-transfer
coefficients of hydrocarbon gases is
attributable to the large variation in
operating pressure. As operating pres-
sure rises, gas density increases. Pres-
sure drop is directly proportional to
the square of mass velocity and in-
versely proportional to density. There-
fore, for the same pressure drop, a
higher mass velocity can be main-
tained when the density is higher. This
larger mass velocity translates into a
higher heat-transfer coefficient.
Pressure drop
Mass velocity strongly inuences
the heat-transfer coefficient. For tur-
bulent ow, the tubeside heat-transfer
coefficient varies to the 0.8 power of
tubeside mass velocity, whereas tube-
side pressure drop varies to the square
of mass velocity. Thus, with increas-
ing mass velocity, pressure drop in-
creases more rapidly than does the
heat-transfer coefficient. Consequent-
ly, there will be an optimum mass ve-
locity above which it will be wasteful
to increase mass velocity further.
Furthermore, very high velocities
lead to erosion. However, the pres-
sure drop limitation usually becomes
controlling long before erosive veloc-
ities are attained. The minimum rec-
ommended liquid velocity inside
tubes is 1.0 m/s, while the maximum
is 2.53.0 m/s.
Pressure drop is proportional to
the square of velocity and the total
length of travel. Thus, when the num-
ber of tube passes is increased for a
given number of tubes and a given
tubeside ow rate, the pressure drop
rises to the cube of this increase. In
actual practice, the rise is somewhat
less because of lower friction factors
at higher Reynolds numbers, so the
exponent should be approximately
2.8 instead of 3.
Tubeside pressure drop rises steeply
with an increase in the number of tube
passes. Consequently, it often happens
that for a given number of tubes and
two passes, the pressure drop is much
lower than the allowable value, but
with four passes it exceeds the allow-
able pressure drop. If in such circum-
stances a standard tube has to be em-
ployed, the designer may be forced to
accept a rather low velocity. However,
if the tube diameter and length may be
varied, the allowable pressure drop can
be better utilized and a higher tubeside
velocity realized.
The following tube diameters are
usually used in the CPI: w, , e, ,
1, 1, and 1 in. Of these, in. and
1 in. are the most popular. Tubes
smaller than in. O.D. should not be
used for fouling services. The use of
small-diameter tubes, such as in.,
is warranted only for small heat ex-
changers with heat-transfer areas less
than 2030 m
2
.
It is important to realize that the
total pressure drop for a given stream
must be met. The distribution of pres-
sure drop in the various heat exchang-
ers for a given stream in a particular
circuit may be varied to obtain good
heat transfer in all the heat exchang-
ers. Consider a hot liquid stream ow-
ing through several preheat exchang-
ers. Normally, a pressure drop of 0.7
kg/cm
2
per shell is permitted for liq-
uid streams. If there are ve such pre-
heat exchangers, a total pressure drop
of 3.5 kg/cm
2
for the circuit would be
permitted. If the pressure drop
through two of these exchangers turns
out to be only 0.8 kg/cm
2
, the balance
of 2.7 kg/cm
2
would be available for
the other three.
Example 1:
Optimizing tubeside design
Consider the heat exchanger ser-
vice specied in Table 1. A TEMA
Type AES exchanger (split-ring pull-
through oating-head construction)
was to be employed. Tubes were to
be either 25 mm O.D. (preferred) or
20 mm O.D., 2 mm thick, and 9 m
long (but could be shorter).
A rst design was produced using
25-mm-O.D. 9-m tubes (Case A in
Table 2). The tubeside pressure drop
was only 0.17 kg/cm
2
even though
0.7 kg/cm
2
was permitted. Further,
the tubeside heat-transfer resistance
was 27.71% of the total, which meant
that if the allowable pressure drop
were better utilized, the heat-transfer
area would decrease. However, when
the number of tube passes was in-
creased from two to four (keeping the
shell diameter the same and decreas-
ing the number of tubes from 500 to
480 due to the extra pass-partition
lanes), the tubeside pressure drop in-
creased to 1.06 kg/cm
2
, which was
unacceptable. (The shellside design
was satisfactory, with the allowable
pressure drop quite well utilized.)
CHEMICAL ENGINEERING PROGRESS FEBRUARY 1998
SHELL- AND- TUBE HEAT EXCHANGERS
Shellside Tubeside
Fluid Crude oil Heavy gas oil circulating reux
Flow rate, kg/h 399,831 277,200
Temperature in/out, C 227 / 249 302 / 275
Operating pressure, kg/cm
2
(abs.) 28.3 13.0
Allowable pressure drop, kg/cm
2
1.2 0.7
Fouling resistance, hm
2
C/kcal 0.0007 0.0006
Heat duty, MM kcal/h 5.4945 5.4945
Viscosity in/out, cP 0.664 / 0.563 0.32 / 0.389
Design pressure, kg/cm
2
(gage) 44.0 17.0
Line size, mm (nominal) 300 300
Material of construction Carbon steel Tubes: Type 410 stainless steel
Other: 5CrMo
Table 1. Heat exchanger service for Example 1.
Since the overdesign in the four-
pass conguration was 28.1%, an at-
tempt was made to reduce the tube-
side pressure drop by decreasing the
tube length. When the tube length
was reduced to 7.5 m, the overdesign
was 5.72%, but the tubeside pressure
drop was 0.91 kg/cm
2
, which was
still higher than that permitted.
Next, a design with 20-mm-O.D.
tubes was attempted (Case B in Table
2). The shell diameter and heat-trans-
fer surface decreased considerably,
from 925 mm to 780 mm, and from
343 m
2
to 300 m
2
, respectively. The
tubeside velocity (2.17 m/s vs. 1.36
m/s earlier), pressure drop (0.51
kg/cm
2
vs. 0.17 kg/cm
2
), and heat-
transfer coefficient (1,976 vs. 1,285
kcal/hm
2
C) were all much higher.
The overall heat-transfer coefficient
for this design was 398 kcal/hm
2
C
vs. 356 for Case A.
Stepwise calculations
for viscous liquids
When the variation in tubeside vis-
cosity is pronounced, a single-point
calculation for the tubeside heat-
transfer coefficient and pressure drop
will give unrealistic results. This is
particularly true in cases where a
combination of turbulent (or transi-
tion) ow and laminar ow exist,
since the thermal performance is very
different in these two regimes.
In such cases, it will be necessary
to perform the calculations stepwise
or zone-wise. The number of steps or
zones will be determined by the vari-
ation in the tubeside viscosity and
thus the Reynolds number.
Example 2:
Stepwise calculations
The principal process parameters
for a kettle-type steam generator in a
renery are shown in Table 3. The
viscosity of the heavy vacuum gas oil
varies from 1.6 cP at the inlet to 6.36
cP at the outlet.
A design was produced without
performing the calculations stepwise
that is, on the basis of a single av-
erage temperature and corresponding
physical properties. Details of this de-
sign are shown in Table 4.
Performing the tubeside calcula-
tions stepwise, in ten equal heat duty
steps, revealed that the original ex-
changer was undersurfaced. The rele-
vant performance parameters for the
single-point and stepwise calculations
are compared in Table 5.
The main reason for the difference
was the variation in Reynolds num-
ber, from 9,813 in the rst zone to
2,851 in the last zone. In addition,
the mean temperature difference
(MTD) decreased drastically, from
138.47C in the rst zone to a mere
FEBRUARY 1998 CHEMICAL ENGINEERING PROGRESS
Case A Case B
Shell I.D., mm 925 780
Tube O.D. Number of tubes 25 500 2 20 540 2
Number of tube passes
Heat-transfer area, m
2
343 300
Tube pitch Tube layout angle 32 90 26 90
Baffle type Single-segmental Single-segmental
Baffle spacing, mm 450 400
Baffle cut, percent of diameter 25 30
Velocity, m/s
Shellside 1.15 1.52
Tubeside 1.36 2.17
Heat-transfer coefficient, kcal/hm
2
C
Shellside 2,065 2,511
Tubeside 1,285 1,976
Pressure drop, kg/cm
2
Shellside 0.86 1.2
Tubeside 0.17 0.51
Resistance, %
Shellside lm 17.24 15.84
Tubeside lm 27.71 21.14
Fouling 50.35 57.66
Metal wall 4.69 4.87
Overdesign 8.29 4.87
Table 2. Details of two designs for Example 1.
Shellside Tubeside
Fluid Boiler feedwater, Steam Heavy vacuum gas oil
Flow rate, kg/h 23,100 (fully vaporized) 129,085
Temperature in/out, C 154 / 154 299 / 165
Allowable pressure drop, kg/cm
2
Negligible 1.4
Fouling resistance, hm
2
C/kcal 0.0002 0.0006
Viscosity in/out, cP 0.176 / 0.176 1.6 / 6.36
Design pressure, kg/cm
2
(gage) 6.5 21.3
Heat duty, kcal/h 11,242,000 11,242,000
Table 3. Process parameters for Example 2.
17.04C in the last. Thus, while the
initial zones (the hot end) had both a
high heat-transfer coefficient and a
high MTD, these decreased progres-
sively toward the outlet (cold) end of
the exchanger. Consequently, while
the rst zone required a length of
only 2.325 m, the last zone required
a length of 44.967 m, even though
the heat duties were the same. The
tubeside pressure drop was only
marginally higher by the stepwise
method, because the tubeside is en-
tirely in the transition regime (Re be-
tween 2,851 and 9,813).
Shellside design
The shellside calculations are far
more complex than those for the
tubeside. This is mainly because on
the shellside there is not just one ow
stream but one principal cross-ow
stream and four leakage or bypass
streams. There are various shellside
ow arrangements, as well as various
tube layout patterns and baffling de-
signs, which together determine the
shellside stream analysis.
Shell conguration
TEMA denes various shell pat-
terns based on the ow of the shell-
side uid through the shell: E, F, G,
H, J, K, and X (see Figure 1).
In a TEMAE single-pass shell, the
shellside uid enters the shell at one
end and leaves from the other end.
This is the most common shell type
more heat exchangers are built to
this conguration than all other con-
gurations combined.
A TEMA F two-pass shell has a
longitudinal baffle that divides the
shell into two passes. The shellside
uid enters at one end, traverses the
entire length of the exchanger
through one-half the shell cross-sec-
tional area, turns around and ows
through the second pass, then nally
leaves at the end of the second pass.
The longitudinal baffle stops well
short of the tubesheet, so that the
uid can ow into the second pass.
The F shell is used for tempera-
ture-cross situations that is, where
the cold stream leaves at a tempera-
ture higher than the outlet tempera-
ture of the hot stream. If a two-pass
(F) shell has only two tube passes,
this becomes a true countercurrent ar-
rangement where a large temperature
cross can be achieved.
A TEMA J shell is a divided-ow
shell wherein the shellside uid en-
ters the shell at the center and divides
into two halves, one owing to the
left and the other to the right and
leaving separately. They are then
combined into a single stream. This is
identied as a J 12 shell. Alterna-
tively, the stream may be split into
two halves that enter the shell at the
two ends, ow toward the center, and
leave as a single stream, which is
identied as a J 21 shell.
A TEMA G shell is a split-ow
shell (see Figure 1). This construction
is usually employed for horizontal
thermosyphon reboilers. There is only
a central support plate and no baffles.
A G shell cannot be used for heat ex-
changers with tube lengths greater
than 3 m, since this would exceed the
limit on maximum unsupported tube
length specied by TEMA typical-
ly 1.5 m, though it varies with tube
O.D., thickness, and material.
When a larger tube length is need-
ed, a TEMA H shell (see Figure 1) is
used. An H shell is basically two G
shells placed side-by-side, so that
there are two full support plates. This
is described as a double-split cong-
uration, as the ow is split twice and
recombined twice. This construction,
too, is invariably employed for hori-
zontal thermosyphon reboilers. The
advantage of G and H shells is that
the pressure drop is drastically less
and there are no cross baffles.
A TEMA X shell (see Figure 1) is
a pure cross-flow shell where the
shellside fluid enters at the top (or
bottom) of the shell, flows across the
tubes, and exits from the opposite
side of the shell. The flow may be
introduced through multiple nozzles
CHEMICAL ENGINEERING PROGRESS FEBRUARY 1998
SHELL- AND- TUBE HEAT EXCHANGERS
Number of kettles 2 (in parallel)
Kettle/port I.D., mm 1,825 / 1,225
Tubes per kettle 790 tubes
Type 316 stainless steel
25 mm O.D. 2 mm thick 9 m long
Number of tube passes 12
Tube pitch 32 mm square (90)
Baffling Full support plates only
Connections, mm (nominal) Shellside: inlet 75, outlet 3 200
Tubeside: 150
Heat-transfer area, m
2
1,104 (2 552)
Table 4. Design produced for Example 2
without stepwise calculations.
Single-point Stepwise
Calculations Calculations
Tubeside heat-transfer coefficient, kcal/hm
2
C 347.9 229.2
Overall heat-transfer coefficient, kcal/hm
2
C 244.7 179.3
Tubeside pressure drop, kg/cm
2
1.28 1.35
Overdesign, % 24.03 9.11
Table 5. Performance parameters for Example 2 using
single-point and stepwise calculations.
located strategically along the length
of the shell in order to achieve a bet-
ter distribution. The pressure drop
will be extremely low in fact,
there is hardly any pressure drop in
the shell, and what pressure drop
there is, is virtually all in the noz-
zles. Thus, this configuration is em-
ployed for cooling or condensing va-
pors at low pressure, particularly
vacuum. Full support plates can be
located if needed for structural in-
tegrity; they do not interfere with the
shellside flow because they are par-
allel to the flow direction.
A TEMA K shell (see Figure 1) is
a special cross-ow shell employed
for kettle reboilers (thus the K). It
has an integral vapor-disengagement
space embodied in an enlarged shell.
Here, too, full support plates can be
employed as required.
Tube layout patterns
There are four tube layout pat-
terns, as shown in Figure 6: triangular
(30), rotated triangular (60), square
(90), and rotated square (45).
A triangular (or rotated triangular)
pattern will accommodate more tubes
than a square (or rotated square) pat-
tern. Furthermore, a triangular pat-
tern produces high turbulence and
therefore a high heat-transfer coeffi-
cient. However, at the typical tube
pitch of 1.25 times the tube O.D., it
does not permit mechanical cleaning
of tubes, since access lanes are not
available. Consequently, a triangular
layout is limited to clean shellside
services. For services that require
mechanical cleaning on the shellside,
square patterns must be used. Chemi-
cal cleaning does not require access
lanes, so a triangular layout may be
used for dirty shellside services pro-
vided chemical cleaning is suitable
and effective.
A rotated triangular pattern sel-
dom offers any advantages over a
triangular pattern, and its use is
consequently not very popular.
For dirty shellside services, a
square layout is typically employed.
However, since this is an in-line
pattern, it produces lower turbu-
lence. Thus, when the shellside
Reynolds number is low (< 2,000),
it is usually advantageous to em-
ploy a rotated square pattern be-
cause this produces much higher
turbulence, which results in a high-
er efficiency of conversion of pres-
sure drop to heat transfer.
As noted earlier, fixed-tubesheet
construction is usually employed for
clean services on the shellside, U-
tube construction for clean services
on the tubeside, and floating-head
construction for dirty services on
both the shellside and tubeside. (For
clean services on both shellside and
tubeside, either fixed-tubesheet or
U-tube construction may be used, al-
though U-tube is preferable since it
permits differential expansion be-
tween the shell and the tubes.)
Hence, a triangular tube pattern may
be used for fixed-tubesheet exchang-
ers and a square (or rotated square)
pattern for floating-head exchangers.
For U-tube exchangers, a triangular
pattern may be used provided the
shellside stream is clean and a
square (or rotated square) pattern if
it is dirty.
Tube pitch
Tube pitch is dened as the shortest
distance between two adjacent tubes.
For a triangular pattern, TEMA
species a minimum tube pitch of
1.25 times the tube O.D. Thus, a 25-
mm tube pitch is usually employed
for 20-mm O.D. tubes.
For square patterns, TEMA addi-
tionally recommends a minimum
cleaning lane of in. (or 6 mm) be-
tween adjacent tubes. Thus, the mini-
mum tube pitch for square patterns is
either 1.25 times the tube O.D. or the
tube O.D. plus 6 mm, whichever is
larger. For example, 20-mm tubes
should be laid on a 26-mm (20 mm +
6 mm) square pitch, but 25-mm tubes
should be laid on a 31.25-mm (25
mm 1.25) square pitch.
Designers prefer to employ the
minimum recommended tube pitch,
because it leads to the smallest shell
diameter for a given number of tubes.
However, in exceptional circum-
stances, the tube pitch may be in-
creased to a higher value, for exam-
ple, to reduce shellside pressure drop.
This is particularly true in the case of
a cross-ow shell.
Baffling
Type of baffles. Baffles are used to
support tubes, enable a desirable ve-
locity to be maintained for the shell-
side uid, and prevent failure of tubes
due to ow-induced vibration. There
are two types of baffles: plate and rod.
Plate baffles may be single-segmental,
double-segmental, or triple-segmen-
tal, as shown in Figure 7.
Baffle spacing. Baffle spacing is
the centerline-to-centerline distance
between adjacent baffles. It is the
most vital parameter in STHE design.
The TEMA standards specify the
minimum baffle spacing as one-fth
of the shell inside diameter or 2 in.,
whichever is greater. Closer spacing
will result in poor bundle penetration
by the shellside uid and difficulty in
mechanically cleaning the outsides of
the tubes. Furthermore, a low baffle
spacing results in a poor stream dis-
tribution as will be explained later.
FEBRUARY 1998 CHEMICAL ENGINEERING PROGRESS
Square
(90)
Rotated
Square
(45)
Triangular
(30 )
Rotated
Triangular
(60)
s Figure 6. Tube layout patterns.
The maximum baffle spacing is
the shell inside diameter. Higher baf-
e spacing will lead to predominantly
longitudinal ow, which is less effi-
cient than cross-ow, and large un-
supported tube spans, which will
make the exchanger prone to tube
failure due to ow-induced vibration.
Optimum baffle spacing. For tur-
bulent ow on the shellside (Re >
1,000), the heat-transfer coefficient
varies to the 0.60.7 power of veloci-
ty; however, pressure drop varies to
the 1.72.0 power. For laminar ow
(Re < 100), the exponents are 0.33 for
the heat-transfer coefficient and 1.0
for pressure drop. Thus, as baffle
spacing is reduced, pressure drop in-
creases at a much faster rate than
does the heat-transfer coefficient.
This means that there will be an
optimum ratio of baffle spacing to
shell inside diameter that will result
in the highest efficiency of conver-
sion of pressure drop to heat transfer.
This optimum ratio is normally be-
tween 0.3 and 0.6.
Baffle cut. As shown in Figure 8,
baffle cut is the height of the segment
that is cut in each baffle to permit the
shellside uid to ow across the baffle.
This is expressed as a percentage of
the shell inside diameter. Although
this, too, is an important parameter for
STHE design, its effect is less pro-
found than that of baffle spacing.
Baffle cut can vary between 15%
and 45% of the shell inside diameter.
Both very small and very large
baffle cuts are detrimental to effi-
cient heat transfer on the shellside
due to large deviation from an ideal
situation, as illustrated in Figure 9. It
is strongly recommended that only
baffle cuts between 20% and 35% be
employed. Reducing baffle cut
below 20% to increase the shellside
heat-transfer coefficient or increas-
ing the baffle cut beyond 35% to de-
crease the shellside pressure drop
usually lead to poor designs. Other
aspects of tube bundle geometry
should be changed instead to achieve
those goals. For example, double-
segmental baffles or a divided-flow
shell, or even a cross-flow shell,
may be used to reduce the shellside
pressure drop.
For single-phase uids on the
shellside, a horizontal baffle cut (Fig-
ure 10) is recommended, because this
minimizes accumulation of deposits
at the bottom of the shell and also
prevents stratication. However, in
the case of a two-pass shell (TEMA
F), a vertical cut is preferred for ease
of fabrication and bundle assembly.
Baffling is discussed in greater de-
tail in (2) and (3).
Equalize cross-ow
and window velocities
Flow across tubes is referred to as
cross-ow, whereas ow through the
window area (that is, through the baffle
cut area) is referred to as window ow.
The window velocity and the
cross-ow velocity should be as close
as possible preferably within 20%
CHEMICAL ENGINEERING PROGRESS FEBRUARY 1998
SHELL- AND- TUBE HEAT EXCHANGERS
Baffle
Baffle
Cut
s Figure 8. Baffle cut.
s Figure 7. Types of baffles.
Double Segmental
Baffles
Triple Segmental
Baffles
Single Segmental
Baffles
Rod Baffle No-Tubes-in-Window Segmental Baffles
of each other. If they differ by more
than that, repeated acceleration and
deceleration take place along the
length of the tube bundle, resulting in
inefficient conversion of pressure
drop to heat transfer.
Shellside stream analysis
On the shellside, there is not just
one stream, but a main cross-ow
stream and four leakage or bypass
streams, as illustrated in Figure 11.
Tinker (4) proposed calling these
streams the main cross-ow stream
(B), a tube-to-baffle-hole leakage
stream (A), a bundle bypass stream
(C), a pass-partition bypass stream
(F), and a baffle-to-shell leakage
stream (E).
While the B (main cross-flow)
stream is highly effective for heat
transfer, the other streams are not as
effective. The A stream is fairly effi-
cient, because the shellside fluid is
in contact with the tubes. Similarly,
the C stream is in contact with the
peripheral tubes around the bundle,
and the F stream is in contact with
the tubes along the pass-partition
lanes. Consequently, these streams
also experience heat transfer, al-
though at a lower efficiency than the
B stream. However, since the E
stream flows along the shell wall,
where there are no tubes, it encoun-
ters no heat transfer at all.
The fractions of the total ow rep-
resented by these ve streams can be
determined for a particular set of ex-
changer geometry and shellside ow
conditions by any sophisticated heat-
exchanger thermal design software.
Essentially, the ve streams are in
parallel and ow along paths of vary-
ing hydraulic resistances. Thus, the
ow fractions will be such that the
pressure drop of each stream is iden-
tical, since all the streams begin and
end at the inlet and outlet nozzles.
Subsequently, based upon the effi-
ciency of each of these streams, the
overall shellside stream efficiency
and thus the shellside heat-transfer
coefficient is established.
Since the ow fractions depend
strongly upon the path resistances,
varying any of the following con-
struction parameters will affect
stream analysis and thereby the shell-
side performance of an exchanger:
baffle spacing and baffle cut;
tube layout angle and tube
pitch;
number of lanes in the ow di-
rection and lane width;
clearance between the tube and
the baffle hole;
clearance between the shell I.D.
and the baffle; and
location of sealing strips and
sealing rods.
FEBRUARY 1998 CHEMICAL ENGINEERING PROGRESS
Horizontal Cut
Vertical Cut
s Figure 10. Baffle cut orientation.
B
B
B
B
F
E
A
B
E
A
C
B
C
A
C
C
s Figure 11. Shellside ow distribution.
s Figure 9. Effect of small and large baffle cuts.
Main Flow
Eddies
Baffle
Shell
Diameter
a. Small Baffle Cut
Eddies
Main
Flow
b. Large Baffle Cut
Baffle
c. Ideal Baffle Cut and Baffle Spacing
Baffle
CHEMICAL ENGINEERING PROGRESS FEBRUARY 1998
SHELL- AND- TUBE HEAT EXCHANGERS
Using a very low baffle spacing
tends to increase the leakage and by-
pass streams. This is because all ve
shellside streams are in parallel and,
therefore, have the same pressure
drop. The leakage path dimensions
are xed. Consequently, when baffle
spacing is decreased, the resistance of
the main cross-ow path and thereby
its pressure drop increases. Since the
pressure drops of all ve streams must
be equal, the leakage and bypass
streams increase until the pressure
drops of all the streams balance out.
The net result is a rise in the pressure
drop without a corresponding increase
in the heat-transfer coefficient.
The shellside uid viscosity also
affects stream analysis profoundly. In
addition to inuencing the shellside
heat transfer and pressure drop per-
formance, the stream analysis also
affects the mean temperature differ-
ence (MTD) of the exchanger. This
will be discussed in detail later. First,
though, lets look at an example that
demonstrates how to optimize baffle
design when there is no signicant
temperature prole distortion.
Example 3:
Optimizing baffle design
Consider the heat exchanger ser-
vice specied in Table 6. Since there
are two independent variables baf-
e spacing and baffle cut we will
rst keep the baffle cut constant at
25% and vary the baffle spacing
(Table 7). Later, the baffle spacing
will be kept constant and the baffle
cut varied (Table 8). In real practice,
both parameters should be varied si-
multaneously, but keeping one pa-
rameter constant and varying the
other will more vividly demonstrate
the inuence of each parameter.
The rst design developed is des-
ignated Design A in Table 7. Here,
the baffle cut is 25% and the baffle
spacing is 300 mm. In Designs B and
C, the baffle spacing was changed to
350 mm and 400 mm, respectively.
There is no temperature prole dis-
tortion problem with these designs.
Notice that as the baffle spacing is
increased from 300 mm to 400 mm,
the main cross-ow, bundle bypass,
and pass-partition bypass streams in-
crease progressively, whereas the
tube-to-baffle-hole leakage and baf-
e-to-shell leakage streams decrease
progressively. The overall heat-trans-
fer efficiency of the shellside stream
increases progressively. Neverthe-
less, since the shellside velocity and
the Reynolds number decrease, both
the shellside heat-transfer coefficient
and the shellside pressure drop de-
crease, but the former at a much
lower rate than the latter. Since the
allowable shellside pressure drop is
1.0 kg/cm
2
, Design A is ruled out, as
its shellside pressure drop far ex-
Shellside Tubeside
Fluid Crude oil Heavy gas oil circulating reux
Flow rate, kg/h 367,647 105,682
Temperature in/out, C 209 / 226 319 / 269
Heat duty, MM kcal/h 4.0 4.0
Density in/out, kg/m
3
730 / 715 655 / 700
Viscosity in/out, cP 0.52 / 0.46 0.27 / 0.37
Specic heat in/out, kcal/kgC 0.63 / 0.65 0.78 / 0.73
Thermal conductivity in/out, kcal/hmC 0.087 / 0.085 0.073 / 0.0795
Allowable pressure drop, kg/cm
2
1.0 0.7
Fouling resistance, hm
2
C/kcal 0.0006 0.0006
Design pressure, kg/cm
2
(gage) 36.6 14.0
Design temperature, C 250 340
Line size, mm (nominal) 300 150
Material of construction Carbon steel 5CrMo
Table 6. Process parameters for Example 3.
Design A Design B Design C
Baffle spacing, mm 300 350 400
Tube-to-baffle-hole leakage (A), fraction 0.157 0.141 0.13
Main cross-ow stream (B), fraction 0.542 0.563 0.577
Bundle bypass stream (C), fraction 0.113 0.116 0.119
Baffle-to-shell leakage stream (E), fraction 0.12 0.109 0.1
Pass-partition bypass stream (F), fraction 0.069 0.072 0.075
Overall shellside heat-transfer efficiency, % 71.3 73.4 74.9
Shellside velocity, m/s
Cross-ow 2.5 2.15 1.87
Window ow 2.34 2.34 2.34
Shellside pressure drop, kg/cm
2
1.34 1.03 0.79
Heat-transfer coefficient, kcal/hm
2
C
Shellside 2,578 2,498 2,372
Tubeside 1,402 1,402 1,402
Overall 401.8 399.8 396.5
Overdesign, % 7.58 7.08 6.21
Table 7. Effects of varying baffle spacing for a constant 25%
baffle cut for Example 3.
ceeds this limit. Designs B and C are
both acceptable. The overdesign
varies marginally. Thus, it would be
prudent to adopt Design C, since it
has a lower pressure drop and a bet-
ter stream analysis.
Now consider the effect of varying
the baffle cut while keeping the baffle
spacing constant at 400 mm, as
shown in Table 8. As the baffle cut is
progressively increased from 25% in
Design D to 36% in Design G, the
following changes are observed:
the main cross-ow stream (B)
fraction increases appreciably;
the tube-to-baffle-hole (A), baf-
e-to-shell (E), and pass-partition (F)
stream fractions decrease steadily;
the bundle bypass (C) stream
fraction remains steady;
the overall heat-transfer effi-
ciency of the shellside stream rst de-
creases and then increases; and
as the window velocity decreas-
es, the shellside heat-transfer coeffi-
cient falls; the pressure drop also de-
creases, but not as fast as the heat-
transfer coefficient.
These observations are reected in
the overdesign values. Design E ap-
pears to be the best choice, since De-
sign D cannot be accepted because of
the excessive shellside pressure drop.
Reducing P
by modifying baffle design
Single-pass shell and single-seg-
mental baffles. The rst baffle alter-
native is the single-segmental baffle
in a single-pass (TEMA E) shell.
However, in many situations, the
shellside pressure drop is too high
with single-segmental baffles in a sin-
gle-pass shell, even after increasing
the baffle spacing and baffle cut to the
highest values recommended. Such a
situation may arise when handling a
very high shellside ow rate or when
the shellside uid is a low-pressure
gas. In these cases, the next alterna-
tive that should be considered is the
double-segmental baffle (Figure 7).
Single-pass shell and double-seg-
mental baffles. By changing the baf-
ing from single-segmental to double-
segmental at the same spacing in an
otherwise identical heat exchanger,
the cross-ow velocity is reduced ap-
proximately to half, because the shell-
side ow is divided into two parallel
streams. This greatly reduces the
cross-ow pressure drop. However,
the window velocity and therefore the
window pressure drop cannot be re-
duced appreciably (assuming that the
maximum recommended baffle cut
was already tried with single-segmen-
tal baffles before switching to double-
segmental baffles). Nevertheless,
since cross-ow pressure drop is in-
variably much greater than window
pressure drop, there is an appreciable
reduction in the total pressure drop.
There is also a decrease in the shell-
side heat-transfer coefficient, but this
is considerably less than the reduction
in the pressure drop. The use of dou-
ble-segmental baffles is covered in
depth in (3).
Divided-ow shell and single-seg-
mental baffles. If the allowable shell-
side pressure drop cannot be satised
even with double-segmental baffles at
a relatively large spacing, a divided-
ow shell (TEMA J) with single-seg-
mental baffles (Figure 1) should be in-
vestigated next. Since pressure drop is
proportional to the square of the veloc-
ity and to the length of travel, a divid-
ed-ow shell will have approximately
FEBRUARY 1998 CHEMICAL ENGINEERING PROGRESS
Design D Design E Design F Design G Design H
Baffle cut, percent of diameter 25 30 33 36 20
Tube-to-baffle-hole leakage (A), fraction 0.13 0.106 0.093 0.08 0.159
Main cross-ow stream (B), fraction 0.577 0.612 0.643 0.674 0.54
Bundle bypass stream (C), fraction 0.119 0.122 0.118 0.117 0.126
Baffle-to-shell leakage stream (E), fraction 0.1 0.091 0.085 0.078 0.114
Pass-partition bypass stream (F), fraction 0.075 0.069 0.062 0.052 0.061
Overall shellside heat-transfer efficiency, % 74.9 73.0 75.7 78.6 72.7
Shellside velocity, m/s
Cross-ow 1.87 1.87 1.87 1.87 1.87
Window ow 2.34 1.86 1.65 1.48 3.09
Shellside pressure drop, kg/cm
2
0.79 0.69 0.65 0.6 0.98
Heat-transfer coefficient, kcal/hm
2
C
Shellside 2,372 2,200 2,074 1,929 2,406
Tubeside 1,402 1,402 1,402 1,402 1,402
Overall 396.5 391.4 387.3 381.9 397.4
Overdesign, % 6.21 4.86 3.76 2.33 6.43
Table 8. Effects of varying baffle cut for a constant 400-mmbaffle spacing for Example 3.
one-eighth the pressure drop in an oth-
erwise identical single-pass exchanger.
The advantage of a divided-ow
shell over double-segmental baffles is
that it offers an even larger reduction
in pressure drop, since not only cross-
ow velocity but even window veloc-
ity can be reduced. The disadvantage
is the increase in cost due to the addi-
tional piping required.
Divided-ow shell and double-
segmental baffles. If even a divided-
ow shell with single-segmental baf-
es is unable to meet the allowable
shellside pressure drop limit, it will
be necessary to adopt a combination
of a divided-ow shell and double-
segmental baffles. With such a com-
bination, a very large reduction in
shellside pressure drop is possible
to as low as 4% of the pressure drop
in a single-pass exchanger with the
same baffle spacing and baffle cut. In
sharp contrast, the heat-transfer coef-
cient will reduce to about 40%.
No-tubes-in-window segmental
baffles. As baffle spacing is increased
to reduce the shellside pressure drop,
an exchanger becomes more prone to
tube failure due to ow-induced vi-
bration. Exchangers with double-seg-
mental baffles are less likely to expe-
rience such problems than those with
single-segmental baffles.
However, a vibration problem may
persist even with double-segmental
baffles. In such cases, a no-tubes-in-
window design (Figure 7) should be
adopted. Here, each tube is supported
by every baffle, so that the unsupport-
ed tube span is the baffle spacing. In
exchangers with normal single-seg-
mental baffles, the unsupported tube
span is twice the baffle spacing.
Should it become necessary to use
a very large baffle spacing to restrict
the shellside pressure drop to the per-
mitted value, intermediate supports
may be used to increase the natural
frequency of the tubes, thus produc-
ing a design that is safe against tube
failure due to ow-induced vibration.
The no-tubes-in-window design
requires a larger shell diameter for a
given number of tubes. This esclates
its cost, typically by about 10%. The
higher cost is offset to some extent by
the higher shellside heat-transfer co-
efficient, since pure cross-ow is
more efficient than the combination
of cross-ow and window ow in
conventional designs.
Cross-ow shell. There are some
services where the pressure drop limi-
tation is so severe that none of the
above shell/baffling congurations can
yield a satisfactory design. A steam
ejector condenser operating at a pres-
sure of 50 mm Hg and having an al-
lowable pressure drop of 5 mm Hg is
an example. Such situations require the
use of a cross-ow shell (TEMAX).
Here, pure cross-ow takes place at
a very low velocity, so there is virtually
no pressure drop in the shell. Whatever
pressure drop occurs is almost entirely
in the nozzles. Support plates will be
needed to meet TEMA requirements
and prevent any possible ow-induced
tube vibration. Since the shellside ow
is parallel to these support plates, shell-
side pressure drop is not increased.
Increasing tube pitch
For a given number of tubes, the
smaller the tube pitch, the smaller the
shell diameter, and therefore the
lower the cost. Consequently, design-
ers tend to pack in as many tubes as
mechanically possible.
As noted earlier, designers gener-
ally set the tube pitch at 1.25 times
the tube O.D. For square or rotated
square pitch, a minimum cleaning
lane of in. or 6 mm is recommend-
ed by TEMA.
As far as thermal-hydraulics are
concerned, the optimum tube-pitch-
to-tube-diameter ratio for conversion
of pressure drop to heat transfer is
typically 1.251.35 for turbulent ow
and around 1.4 for laminar ow.
Increasing the tube pitch to re-
duce pressure drop is generally not
recommended for two reasons. First,
it increases the shell diameter and,
thereby, the cost. Second, reducing
pressure drop by modifying the baf-
fle spacing, baffle cut, or shell type
will result in a cheaper design.
However, in the case of X shells, it
may be necessary to increase the tube
pitch above the TEMA minimum to
meet pressure drop limitations, since
there are no other parameters that can
be modied.
Mean temperature difference
Temperature difference is the driv-
ing force for heat transfer.
When two streams flow in op-
posing directions across a tube wall,
there is true countercurrent flow
(Figure 12). In this situation, the
only limitation is that the hot
stream should at all points be hotter
than the cold stream. The outlet
temperature of the cold stream may
be higher than the outlet tempera-
ture of the hot stream, as shown in
Figure 12.
CHEMICAL ENGINEERING PROGRESS FEBRUARY 1998
SHELL- AND- TUBE HEAT EXCHANGERS
s Figure 13. Cocurrent ow.
T
e
m
p
e
r
a
t
u
r
e
Exchanger Length
s Figure 12. Countercurrent ow.
T
e
m
p
e
r
a
t
u
r
e
Exchanger Length
Since the temperature difference
varies along the length of the heat
exchanger, it has to be weighted to
obtain a mean value for single-point
determination of heat-transfer area.
The logarithmic mean temperature
difference (LMTD) represents this
weighted value.
If the hot and cold streams ow in
the same direction, ow is cocurrent
(Figure 13). The mean temperature
difference is still represented by the
LMTD. However, the LMTD for
cocurrent ow is lower than that for
countercurrent ow for the same ter-
minal differences. This is because al-
though one terminal temperature dif-
ference is very high, the other is far
too low that is, the temperature
differences along the path of heat
transfer are not balanced.
What is even more serious with
cocurrent ow is that the outlet tem-
perature of the cold stream must be
somewhat lower than the outlet tem-
perature of the hot stream, which is a
serious limitation. Consequently,
countercurrent ow is always pre-
ferred to cocurrent ow.
These principles apply only to sin-
gle-pass exchangers. However, as
noted earlier, shell-and-tube heat ex-
changers invariably have two or more
tube passes. Since the shellside uid
ows in one direction, half the tube
passes experience countercurrent
ow and the other half experience
cocurrent ow. The MTD for this sit-
uation is neither the LMTD for coun-
tercurrent ow nor that for cocurrent
ow, but a value between the two.
A correction factor, F
t
, which de-
pends on the four terminal tempera-
tures and the shell style can be deter-
mined from charts in the TEMA stan-
dards. The LMTD for countercurrent
ow is multiplied by this factor to ob-
tain the corrected MTD.
An important limitation for 1-2
shells (one shell pass and two or more
tube passes) is that the outlet tempera-
ture of the cold stream cannot exceed
the outlet temperature of the hot
stream. This is because of the presence
of one or more cocurrent passes. In re-
ality, a very small temperature differ-
ence is possible, but this represents an
area of uncertainty and the credit is
very small, so it is usually ignored.
When there is a temperature cross
(that is, the outlet temperature of the
cold stream is higher than the outlet
temperature of the hot stream), and
pure countercurrent ow is not possi-
ble, multiple shells in series must be
used. This will be discussed in detail
in the followup article scheduled to
be published in the next issue.
An F shell has two passes, so if there
are two tube passes, this is a pure coun-
tercurrent situation. However, if an F
shell has four or more tube passes, it is
no longer a true countercurrent situation
and, hence, the F
t
correction has to be
applied. An F shell having four or more
tube passes is represented as a 2-4 shell.
The F
t
factor for a 2-4 shell is identical
to that for two 1-2 shells in series or two
shell passes. The TEMA F
t
factor chart
for three shell passes really represents
three shells in series, that for four shell
passes four shells in series, and so on.
It is important to realize that the
LMTD and F
t
factor concept assumes
that there is no signicant variation in
the overall heat-transfer coefficient
along the length of the shell. Howev-
er, there are some services where this
is not true. An example of this is the
cooling of a viscous liquid as the
liquid is cooled, its viscosity increas-
es, and this results in a progressive
reduction in the shellside heat-trans-
fer coefficient. In this case, the sim-
plistic overall MTD approach will be
inaccurate, and the exchanger must
be broken into several sections and
the calculations performed zone-wise.
Temperature prole distortion
An important issue that has not
been considered so far is the tempera-
ture prole distortion. As noted earli-
er, the leakage and bypass streams are
less efficient for heat transfer than the
main cross-ow stream.
Consider a case where the shellside
stream is the cold uid. Since the
main cross-ow stream encounters a
very large fraction of the total heat-
transfer surface, it has to pick up a
very large part of the total heat duty.
Assume that the cross-ow stream is
58% of the total shellside stream, but
that it comes in contact with 80% of
the tubes. As a result, its temperature
rises more rapidly than if the entire
shellside stream were to pick up the
entire heat duty. Therefore, its temper-
FEBRUARY 1998 CHEMICAL ENGINEERING PROGRESS
s Figure 14. Temperature prole distortion factor due to bypass and leakage.
T
e
m
p
e
r
a
t
u
r
e
Tubeside
M
a
in
C
ro
ss-flo
w
S
tre
a
m
C
S
tream
E Stream
Shellside
L
a
s
t

B
a
f
f
l
e
Exchanger Length
Apparent Temperature Profile
C =Bundle-to-Shell Bypass
E =Baffle-to-Shell Leakage
ature prole will be steeper than that
of the total stream (the apparent tem-
perature prole) without considering
the various ow fractions (Figure 14).
The temperature proles of the
baffle-hole-to-tube leakage, shell-to-
bundle leakage, and pass-partition by-
pass streams will depend on their re-
spective ow fractions and the frac-
tional heat-transfer area encountered.
However, since the shell-to-baffle
leakage stream does not experience
any heat transfer, the remaining four
streams must pick up the entire heat
duty, so that these four streams to-
gether will have a temperature prole
steeper than that of the apparent
stream. Consequently, the temperature
difference between the hot and the
cold streams will be lower all along
the length of the heat exchanger,
thereby resulting in the reduction of
the MTD. This reduction in the MTD
is known as the temperature prole
distortion (or correction) factor.
The temperature prole distortion
factor is more pronounced when the
leakage and bypass streams are high,
especially the shell-to-baffle leakage
stream, and the ratio of shellside tem-
perature difference to the temperature
approach at the shell outlet is high.
The latter is because the closer the
temperature approach at the shell out-
let, the sharper the reduction in MTD. The leakage and bypass streams
tend to be high when the shellside
viscosity is high and when the baffle
spacing is very low. Thus, care has to
be exercised in the design of viscous
liquid coolers such as a vacuum
residue cooler in a crude oil renery.
The minimum recommended tem-
perature prole distortion factor is
0.75. Below this, two or more shells
in series must be employed. By using
multiple shells in series, the ratio of
shellside temperature difference to
the temperature approach at the shell
outlet is reduced. The mixing of the
main cross-ow stream with the by-
pass and leakage streams after each
shell reduces the penalty due to the
distortion of the temperature prole
and hence increases the temperature
prole distortion factor.
In many situations, a temperature
profile distortion factor is unavoid-
able, such as when cooling a viscous
liquid over a large temperature
range, and there is no alternative to
the use of multiple shells in series.
However, in many other situations,
improper baffle spacing unnecessar-
ily imposes such a penalty where it
is easily avoidable. Designers nor-
mally tend to pack baffles as close
as possible to get the maximum
shellside heat-transfer coefficient,
pressure drop permitting. In many
such cases, the use of somewhat
higher baffle spacing will reduce the
shell-to-baffle leakage stream (the
principal culprit) and hence improve
the MTD correction factor appre-
ciably, thereby producing a much
better design.
CHEMICAL ENGINEERING PROGRESS FEBRUARY 1998
SHELL- AND- TUBE HEAT EXCHANGERS
Shell I.D. 500 mm
Tubes 188 tubes, 20 mm O.D. 2 mm thick 6 m long
Number of tube passes 2
Tube pitch 26 mm square (90)
Baffling Single-segmental, 140 mm spacing, 21% cut (diameter)
Connections 75 mm on shellside, 150 mm on tubeside
Heat-transfer area 70 m
2
Table 10. Construction parameters for Example 4.
Nomenclature
c = stream specic heat, kcal/kgC
D = tube inside diameter, m
F
t
= LMTD correction factor,
dimensionless
G = stream mass velocity, kg/m
2
h
h = stream heat-transfer coefficient,
kcal/hm
2
C
k = stream thermal conductivity,
kcal/hmC
Nu = Nusselt number = hD/k,
dimensionless
Pr = Prandtl number = c/k,
dimensionless
Re = Reynolds number = DG/,
dimensionless
Greek Letter
= stream viscosity, kg/mh
Shellside Tubeside
Fluid Naphtha Cooling water
Flow rate, kg/h 9,841 65,570
Temperature in/out, C 114 / 40 33 / 40
Heat duty, MM kcal/h 0.46 0.46
Specic gravity in/out 0.62 / 0.692 1.0 / 1.0
Viscosity in/out, cP 0.254 / 0.484 0.76 / 0.66
Average specic heat, kcal/kgC 0.632 1.0
Thermal conductivity in/out, kcal/hmC 0.092 / 0.101 0.542 / 0.546
Allowable pressure drop, kg/cm
2
0.7 0.7
Fouling resistance, hm
2
C/kcal 0.0002 0.0004
Design pressure, kg/cm
2
(gage) 12.0 6.5
Design temperature, C 150 60
Material of construction Carbon steel Admirality brass
Table 9. Process parameters for Example 4.
Example 4: Temperature
distortion and baffle spacing
Consider an existing naphtha
cooler in a refinery and petrochemi-
cal complex. The process parameters
are listed in Table 9, and the con-
struction parameters in Table 10.
The existing design was undersur-
faced by 21%, mainly because the
temperature profile distortion factor
was 0.6, which is lower than the
minimum recommended value of
0.75. The existing design had a baf-
fle spacing of 140 mm and a baffle
cut of 21% (of the diameter). The
shell-to-baffle leakage stream frac-
tion was 0.24.
To improve the design, the baffle
spacing was progressively increased.
The undersurfacing decreased with
increasing baffle spacing, up to a
spacing of 190 mm; thereafter, per-
formance again started to deteriorate.
Thus, 190 mm is the optimum baffle
spacing.
The detailed results of the vari-
ous iterations are compared in
Table 11.
CEP
FEBRUARY 1998 CHEMICAL ENGINEERING PROGRESS
Existing Design Alternative No. 1 Alternative No. 2 Alternative No. 3 Alternative No. 4
Baffle spacing, mm 140 160 175 190 210
Stream analysis, fraction of stream
Baffle-hole-to-tube leakage (A) 0.189 0.173 0.163 0.154 0.143
Main cross-ow (B) 0.463 0.489 0.506 0.521 0.539
Shell-to-bundle leakage (C) 0.109 0.113 0.116 0.118 0.121
Shell-to-baffle leakage (E) 0.24 0.225 0.215 0.207 0.196
Pass-partition bypass stream (F) 0 0 0 0 0
Overall shellside heat-transfer 62 64.7 66.4 67.9 69.7
efficiency, %
Temperature prole distortion factor 0.6 0.692 0.735 0.766 0.794
Shellside velocity, m/s 0.15 0.14 0.13 0.13 0.12
Shellside heat-transfer coefficient, 614 570 562 550 512
kcal/hm
2
C
Shellside pressure drop, kg/cm
2
0.034 0.029 0.027 0.026 0.023
Overall heat-transfer coefficient,
kcal/hm
2
C 380 362 359 354 338
Mean temperature difference, C 13.73 15.9 16.87 17.58 18.22
Overdesign, % 21.1 12.8 8.26 5.73 6.61
Table 11. Detailed results of Example 4 iterations.
Literature Cited
1. Tubular Exchanger Manufacturers
Association, Standards of the Tubular
Exchanger Manufacturers Associa-
tion, 7th ed., TEMA, New York
(1988).
2. Mukherjee, R., Dont Let Baffling
Baffle You, Chem. Eng. Progress, 92
(4), pp. 7279 (Apr. 1996).
3. Mukherjee, R., Use Double-Segmen-
tal Baffles in Shell-and-Tube Heat Ex-
changers, Chem. Eng. Progress, 88
(11), pp. 4752 (Nov. 1992).
4. Tinker, T., Shellside Characteristics of
Shell-and-tube Heat Exchangers: A
Simplified Rating System for Commer-
cial Heat Exchangers, Trans. ASME,
80, pp. 3652 (1958).
Further Reading
Kakac, S., et al., Heat Exchangers: Ther-
mal-Hydraulic Fundamentals and De-
sign, Hemisphere Publishing Corp.,
New York (1981).
Schlunder, E.V., et al., eds., Heat Ex-
changer Design Handbook, Hemi-
sphere Publishing Corp., New York
(1983).
R. MUKHERJ EE is assistant chief consultant in
the Heat and Mass Transfer Dept. of Engineers
India Ltd., New Delhi (011-91-11-371-6171;
Fax: 011-91-11-371-5059l; e-mail:
shilpi@giasdla.vsnl.net.in), where he has
been employed since 1971. He has over 26
years of experience in the design, revamping,
and troubleshooting of air-cooled and shell-
and-tube heat exchangers (especially for oil
reneries, gas processing plants, and
petrochemical plants), and also has
considerable experience in heat-exchanger-
network synthesis and optimization. He has
written several articles in technical journals
and has presented two papers in the Industrial
Session of the 10th International Heat Transfer
Conference at Brighton in August 1994.
He has served as faculty for several courses in
heat exchanger design, energy conservation,
and heat exchanger network optimization.
He is an honors graduate in chemical
engineering fromJ adavpur Univ., Calcutta,
and is a member of the Indian Institute of
Chemical Engineers and the Indian Society
for Heat and Mass Transfer.
Acknowledgment
The author is grateful to the management of
Engineers India, Ltd., for permission to publish
this article and acknowledges the use of Heat
Transfer Research, Inc.s software for the
worked-out examples and their design
methodology.
HDA Process
Reactor
Flash
compressor
S
t
a
b
i
l
i
z
e
r
P
r
o
d
u
c
t
R
e
c
y
c
l
e
cool heat
Why Energy Integration?
(ICI Experience)
20 case studies
In every case, there was a reduction in energy
In almost every case, the energy savings required
less capital
Up to 60% energy savings, up to 25% capital
savings, up to 15% lower product price
Why spend additional capital to waste energy?
Energy Integration
What are the minimum cooling and heating
requirements?
What is the minimum number of heat
exchangers required?
What are the main trade-offs?
An Example
For the two hot streams being cooled and
the two cold streams being heated shown
below, find the minimum heating and
cooling requirements, as well as the
minimum number of heat exchangers if
T
min
= 10 deg F.
250 F 120 F
200 F 100 F
150 F 90 F
190 F 130 F
FC
p
(Btu/F)
1000
4000
3000
6000
Stream
1
2
3
4
Net Energy Required
(First Law Calculation)
250 F 120 F
200 F 100 F
150 F 90 F
190 F 130 F
FC
p
Q
aval
(Btu/F) (MBtu/hr)
1000 130
4000 400
3000 -180
6000 -360
-10
Thus, 10 x10
3
Btu/hr would have to be
supplied from a hot utility if there were
no requirement of T
min
= 10 deg F.
Question: What are the minimum
heating and cooling requirements?
Net Energy Required at
Temperature Intervals
Define hot and cold temperature scales
shifted by 10 deg F and show streams
250
220
190
160
130
100
240
210
180
150
120
90
T
H
(F)
T
C
(F)
Net Energy Required at
Temperature Intervals
Draw in intervals by breaking at
stream starting and ending points
250
220
190
160
130
100
240
210
180
150
120
90
T
H
(F)
T
C
(F)
Net Energy Required at
Temperature Intervals
Compute net available Q for each interval
250
220
190
160
130
100
240
210
180
150
120
90
T
H
(F)
T
C
(F)
FC
p 1000 4000 3000 6000
Q
aval
(MBtu/hr)
50
-40
-80
40
20
Note: Sum of net heat available =-10 MBtu/hr
Same as first law
Cascade Diagram
We satisfy the net requirement for each
interval from an external utility.
250
220
190
160
130
100
240
210
180
150
120
90
T
H
(F)
T
C
(F)
H
O
T
U
T
I
L
I
T
Y
C
O
L
D
U
T
I
L
I
T
Y
H =50
H =40
H =20
H =-40
H =-80
BUT ...
Cascade Diagram
We can always transfer excess heat from
high temperature intervals to lower
temperature intervals without violating
T
min
=10 F
250
220
190
160
130
100
240
210
180
150
120
90
T
H
(F)
T
C
(F)
H
O
T
U
T
I
L
I
T
Y
C
O
L
D
U
T
I
L
I
T
Y
H = 50
H =40
H =20
H =-40
H =-80
50
10
70
40
60
Minimum Heating
and Cooling
250
220
190
160
130
100
240
210
180
150
120
90
T
H
(F)
T
C
(F)
H
O
T
U
T
I
L
I
T
Y
C
O
L
D
U
T
I
L
I
T
Y
H = 50
H =40
H =20
H =-40
H =-80
50
10
70
40
60
Minimum heating = 70 MBtu/hr
Minimum cooling = 60 MBtu/hr
Net = 10 MBtu/hr of heating
Same as first law
Pinch Temperature
250
220
190
160
130
100
240
210
180
150
120
90
T
H
(F)
T
C
(F)
H
O
T
U
T
I
L
I
T
Y
C
O
L
D
U
T
I
L
I
T
Y
H = 50
H =40
H =20
H =-40
H =-80
50
10
70
40
60
The design problem can be decomposed
into two separate problems:
1. A high temperature region where only
heating is requred
2. A low temperature region where only
cooling is required
Relationship to First Law
Minimum Heat in = 70
HOT
COLD
Minimum Heat out = 60
}
Net = -10
Heat in = 70 + Q
E
HOT
Heat out = 60 + Q
E
}
Net = -10
If we put excess heat into the process,
we must also remove this heat!
Excess Heating and Cooling
250
220
190
160
130
100
240
210
180
150
120
90
T
H
(F)
T
C
(F)
H
O
T
U
T
I
L
I
T
Y
C
O
L
D
U
T
I
L
I
T
Y
H = 50
H =40
H =20
H =-40
H =-80
50
10+Q
E
70
40+Q
E
60+Q
E
Q
E
Q
E
Excess steam or furnace capacity
requires excess cooling water.
Dont transfer heat across the pinch!
Multiple Utilities
250
220
190
160
130
100
240
210
180
150
120
90
T
H
(F)
T
C
(F)
H
O
T
U
T
I
L
I
T
Y
C
O
L
D
U
T
I
L
I
T
Y
H = 50
H =40
H =20
H =-40
H =-80
50
10
70
20
40
cooling
water
hot
water
We prefer to add heat at the lowest
possible temperature, and to remove
heat at the highest possible temperature!
Multiple utilities correspond to
multiple pinches.
T-H Diagram
Enthalpy, MBtu/hr
E
n
t
h
a
l
p
y
,

M
B
t
u
/
h
r
250
230
210
190
170
150
130
110
90
0 200 400 600
Hot
T-H Diagram
Enthalpy, MBtu/hr
E
n
t
h
a
l
p
y
,

M
B
t
u
/
h
r
250
230
210
190
170
150
130
110
90
0 200 400 600
Cold
T-H Diagram
Enthalpy, MBtu/hr
E
n
t
h
a
l
p
y
,

M
B
t
u
/
h
r
250
230
210
190
170
150
130
110
90
0 200 400 600
Cold
T-H Diagram
Enthalpy, MBtu/hr
E
n
t
h
a
l
p
y
,

M
B
t
u
/
h
r
250
230
210
190
170
150
130
110
90
0 200 400 600
Hot
T
min
= 10 F
Q
C,min
Q
H,min
Cold
Limitation of the
Procedure
We need to know
FC
p
values of all streams
Inlet and outlet temperatures of all
streams
BUT the design variables that
fix the process flows must be
determined from optimization
which in turn depends on heat-
exchanger network
Solution: Heat-exchanger
network as function of the flows
Some iterations are needed
An Exercise
Given the following stream data
below, find (assume T
min
= 10
deg C)
the minimum amount heat added
the minimum amount of cooling
the pinch temperature
180 C 60 C
150 C 30 C
135 C 20 C
140 C 80 C
FC
p
(kW/C)
3
1
2
5
Stream
1
2
3
4
Minimum Number of
Exchangers
From results of minimum
heating and cooling estimates
Hot Utility
70 MBtu/hr
Stream 1
130 MBtu/hr
Stream 2
400 MBtu/hr
Sources
Stream 3
180 MBtu/hr
Stream 4
360 MBtu/hr
Cold Utility
60 MBtu/hr
Sinks
70
110
20
340
60
Heat loads always balance as
result of first law analysis
Number of exchangers (N
E
) =
Number of streams (N
S
) +
Number of Utilities (N
U
) - 1
N
E
= 4 + 2 - 1 = 5
Minimum Number of
Exchangers
The previous solution is not
always correct. Consider
Hot Utility
230 MBtu/hr
Stream 1
130 MBtu/hr
Stream 2
400 MBtu/hr
Sources
Stream 3
180 MBtu/hr
Stream 4
360 MBtu/hr
Cold Utility
220 MBtu/hr
Sinks
230
180
130
220
We see that if we have an exact
matching between some
streams, we need less exchanger
N
E
= N
S
+ N
U
- N
P
N
P
= number of independent
subproblems
Minimum Number of
Exchangers
The previous solution is still not
always correct. Consider
Hot Utility
70
Stream 1
130 MBtu/hr
Stream 2
400 MBtu/hr
Sources
Stream 3
180 MBtu/hr
Stream 4
360 MBtu/hr
Cold Utility
220 MBtu/hr
Sinks
70-Q
E
340
110+Q
E
60
We see that there are now six
exchangers and there is now a
loop (i.e., HU -> S3 -> S1 -> S4
-> HU)
N
E
= N
S
+ N
U
+ N
L
- N
P
N
L
= number of loops
Q
E
20-Q
E
Effect of Pinch Analysis
The pinch analysis indicates that we only
use heat above the pinch and cooling below
the pinch, so that the design problem can
be decomposed into two subproblems.
250
220
190
160
130
100
240
210
180
150
120
90
T
H
(F)
T
C
(F)
Assume no loops
& no exact matches
N
E
= N
S
+ N
U
- 1
= 4 + 1 - 1 = 4
N
E
= N
S
+ N
U
- 1
= 3 + 1 - 1 = 3
N
TOTAL
= 7
Effect of Pinch Analysis
Minimum Exchangers:
N = 5
Pinch Analysis:
N = 7
(minimum energy)
A trade-off between captial cost
(minimum exchangers) and
utility costs (minimum energy)
We can decrease the number of
exchangers if we transfer some
heat across the pinch, but the
energy usage increases
Design of Minimum-
Energy HENs
250
220
190
160
130
100
240
210
180
150
120
90
T
H
(F)
T
C
(F)
Q = 110
Q = 20
Q = 240
Q = 160
Q = 120
Q = 60
Q = 360
Stream 1 2 3 4
FC
p
1000 4000 3000 6000
Pinch
Above the pinch: Q
tot
= 70 MBtu/hr, heat added
Above the pinch: Q
tot
= 60 MBtu/hr, heat removed
Design above the Pinch
250
220
190
160
130
100
240
210
180
150
120
90
T
H
(F)
T
C
(F)
Q = 110
Q = 240
Q = 60
Q = 360
Pinch
Feasible Pinch Matches:
(FC
p
)
HOT
< (FC
p
)
COLD
We can match stream 1 with either 3 or 4,
and we can only match stream 2 with 4.
Design above the Pinch
250
220
190
160
130
100
240
210
180
150
120
90
T
H
(F)
T
C
(F)
Q = 60
Q=240
Pinch
Transfer the maximum amount of heat
possible for each match to attemp to
eliminate streams from the problem.
Remaining Heat Loads
250
220
190
160
130
100
240
210
180
150
120
90
T
H
(F)
T
C
(F)
Pinch
Q = 110-60
= 50
Q = 360-240
= 120
170 F
200 F
Transfer the remaining heat from
stream 1 to stream 4.
Remaining Heat Loads
250
220
190
160
130
100
240
210
180
150
120
90
T
H
(F)
T
C
(F)
Pinch
Q = 50
Heat from Hot Utility
250
220
190
160
130
100
240
210
180
150
120
90
T
H
(F)
T
C
(F)
Pinch
178 F
H = 70
Design above the Pinch
(Summary)
250
220
190
160
130
100
240
210
180
150
120
90
T
H
(F)
T
C
(F)
Q = 60
Q=240
Pinch
1. Put in the matches at the pinch
2. Maximize the heat loads to eliminate
streams
3. See what is left
Q=50
H=70
Design above the Pinch
(Alternatives)
250
220
190
160
130
100
240
210
180
150
120
90
T
H
(F)
T
C
(F)
Q = 60
Q=240
Pinch
Q=50
H=70
Design below the Pinch
250
220
190
160
130
100
240
210
180
150
120
90
T
H
(F)
T
C
(F)
Q = 20
Q = 160
Q = 120
Pinch
Design below the Pinch
(One Alternative)
250
220
190
160
130
100
240
210
180
150
120
90
T
H
(F)
T
C
(F)
Pinch
Q = 120
C = 20
C = 40
T=120
A Complete Design
250
220
190
160
130
100
240
210
180
150
120
90
T
H
(F)
T
C
(F)
Q = 60
Q=240
Q=50
H=70
Q = 120 C = 20
C = 40
Total Number of Exchangers: 7
Remarks
Minimum approach temperature
So far, we have assumed a value
of 10 deg F
Trade-off: Larger minimum
approach temperature, smaller
heat-exchanger area but larger
minimum heating and cooling
Additional complexities
The design problem is not always
as simple as the example
considered
Stream splitting
Alternatives
Reducing the Number of
Exchangers
The number of exchangers required
for the overall process is always
less than or equal to that for the
minimum energy network
The minimum energy network
normally contains loops across the
pinch
These loops can be broken by
transferring heat across the pinch,
but we will introduce at least one
violation of the specified T
min
T
min
can be restored by shifting
heat along a path, which increases
energy consumption of the process
Loops
250
220
190
160
130
100
240
210
180
150
120
90
T
H
(F)
T
C
(F)
A set of connections that starts from one
stream and returns to the same stream.
Loops
250
220
190
160
130
100
240
210
180
150
120
90
T
H
(F)
T
C
(F)
A set of connections that starts from one
stream and returns to the same stream.
Loops
250
220
190
160
130
100
240
210
180
150
120
90
T
H
(F)
T
C
(F)
A set of connections that starts from one
stream and returns to the same stream.
Loops
250
220
190
160
130
100
240
210
180
150
120
90
T
H
(F)
T
C
(F)
A set of connections that starts from one
stream and returns to the same stream.
Breaking Loops
250
220
190
160
130
100
240
210
180
150
120
90
T
H
(F)
T
C
(F)
Q = 60
Q=240
Q=50
H=70
Q = 120 C = 20
C = 40
Break the loop with exchanger with the
small load
Remove the smallest heat load from loop
Breaking a loop across the pinch
normally violates the 2nd law
250
220
190
160
130
100
240
210
180
150
120
90
T
H
(F)
T
C
(F)
Q = 60
Q=240-20
Q=50+20
H=70
Q = 120
C = 40+20
BUT ...
250
220
190
160
130
100
240
210
180
150
120
90
T
H
(F)
T
C
(F)
Q = 60
Q=220
Q=70
H=70
Q = 120
C = 60
Hot Stream: 200 F to 120 F
Cold Stream: 150 F to 130 F
Impossible according to 2nd Law!
Restoring T
min
Q=60+Q
E
Q=220
Q=70-Q
E
H=70+Q
E
C = 60+Q
E
Shift heat along a path, which is a
connection between a cooler and a heater
Thus, 1000(60+Q
E
) = 3000(150-110)
Q
E
= 60 MBtu/hr
< 110 F
T=120
Q = 120- Q
E
Reducing Number of
Exchangers
Q=120
Q=220
Q=10
H=130
C = 120
T=110
T=250
T=240
T=120
T=90
T=150
T=200
T=145
T=115
T=100
T=190
T=168.4
T=166.6
T=130
Total number of Exchangers: 6
Q = 60
Restoring T
min
(An Alternative Path)
Q=60
Q=220-Q
E
Q=70
H=70+Q
E
C = 60+Q
E
< 110 F
T=120
But this path cant restore T
min
!
Q=120
Breaking Another Loop
Q=120
Q=220
Q=10
H=130
C = 120
Q=60
Heat Engines
Heat
Engine
Q
in
+W
Q
in
W
Energy
Cascade
Q
out
Energy
Cascade
Q
in
Energy
Cascade
Q
out
Energy
Cascade
Pinch
Heat
Engine
Q
out
-W
Q
in
W
Energy
Cascade
Q
out
Energy
Cascade
Heat
Engine
Q
E
-W
Q
in
W
Energy
Cascade
Q
out
+Q
E
-W
Energy
Cascade
Q
E
Efficiency: 100% 100% Sandard Alone
Place Heat Engine either above
or below the pinch (not across)!
Heat Pumps
Heat
Pump
Q
in
- W
Q
E
W
Energy
Cascade
Q
out
Energy
Cascade
Q
in
Energy
Cascade
Q
out
Energy
Cascade
Place heat pump across the pinch!
Q
E
+W
Pinch
Heat
Pump
Q
in
Q
E
W
Energy
Cascade
Q
out
+ W
Energy
Cascade
Q
E
- W
Heat
Pump
Q
in
- (Q
E
+W)
Q
E
W
Energy
Cascade
Q
out
- Q
E
Energy
Cascade
Q
E
+W
Distillation
Column as a heat engine
Column
Heat In (Q
Reb
)
Heat Out (Q
Cond
)
Column either above or below
the pinch
Q
in
Energy
Cascade
Q
out
Energy
Cascade
Column
Q
in
Energy
Cascade
Q
out
Energy
Cascade
Column
HDA Process
Reactor
Flash
compressor
S
t
a
b
i
l
i
z
e
r
P
r
o
d
u
c
t
R
e
c
y
c
l
e
cool heat
Why Energy Integration?
(ICI Experience)
20 case studies
In every case, there was a reduction in energy
In almost every case, the energy savings required
less capital
Up to 60% energy savings, up to 25% capital
savings, up to 15% lower product price
Why spend additional capital to waste energy?
Energy Integration
What are the minimum cooling and heating
requirements?
What is the minimum number of heat
exchangers required?
What are the main trade-offs?
An Example
For the two hot streams being cooled and
the two cold streams being heated shown
below, find the minimum heating and
cooling requirements, as well as the
minimum number of heat exchangers if
T
min
= 10 deg F.
250 F 120 F
200 F 100 F
150 F 90 F
190 F 130 F
FC
p
(Btu/F)
1000
4000
3000
6000
Stream
1
2
3
4
Net Energy Required
(First Law Calculation)
250 F 120 F
200 F 100 F
150 F 90 F
190 F 130 F
FC
p
Q
aval
(Btu/F) (MBtu/hr)
1000 130
4000 400
3000 -180
6000 -360
-10
Thus, 10 x10
3
Btu/hr would have to be
supplied from a hot utility if there were
no requirement of T
min
= 10 deg F.
Question: What are the minimum
heating and cooling requirements?
Net Energy Required at
Temperature Intervals
Define hot and cold temperature scales
shifted by 10 deg F and show streams
250
220
190
160
130
100
240
210
180
150
120
90
T
H
(F)
T
C
(F)
Net Energy Required at
Temperature Intervals
Draw in intervals by breaking at
stream starting and ending points
250
220
190
160
130
100
240
210
180
150
120
90
T
H
(F)
T
C
(F)
Net Energy Required at
Temperature Intervals
Compute net available Q for each interval
250
220
190
160
130
100
240
210
180
150
120
90
T
H
(F)
T
C
(F)
FC
p 1000 4000 3000 6000
Q
aval
(MBtu/hr)
50
-40
-80
40
20
Note: Sum of net heat available =-10 MBtu/hr
Same as first law
Cascade Diagram
We satisfy the net requirement for each
interval from an external utility.
250
220
190
160
130
100
240
210
180
150
120
90
T
H
(F)
T
C
(F)
H
O
T
U
T
I
L
I
T
Y
C
O
L
D
U
T
I
L
I
T
Y
H =50
H =40
H =20
H =-40
H =-80
BUT ...
Cascade Diagram
We can always transfer excess heat from
high temperature intervals to lower
temperature intervals without violating
T
min
=10 F
250
220
190
160
130
100
240
210
180
150
120
90
T
H
(F)
T
C
(F)
H
O
T
U
T
I
L
I
T
Y
C
O
L
D
U
T
I
L
I
T
Y
H = 50
H =40
H =20
H =-40
H =-80
50
10
70
40
60
Minimum Heating
and Cooling
250
220
190
160
130
100
240
210
180
150
120
90
T
H
(F)
T
C
(F)
H
O
T
U
T
I
L
I
T
Y
C
O
L
D
U
T
I
L
I
T
Y
H = 50
H =40
H =20
H =-40
H =-80
50
10
70
40
60
Minimum heating = 70 MBtu/hr
Minimum cooling = 60 MBtu/hr
Net = 10 MBtu/hr of heating
Same as first law
Pinch Temperature
250
220
190
160
130
100
240
210
180
150
120
90
T
H
(F)
T
C
(F)
H
O
T
U
T
I
L
I
T
Y
C
O
L
D
U
T
I
L
I
T
Y
H = 50
H =40
H =20
H =-40
H =-80
50
10
70
40
60
The design problem can be decomposed
into two separate problems:
1. A high temperature region where only
heating is requred
2. A low temperature region where only
cooling is required
Relationship to First Law
Minimum Heat in = 70
HOT
COLD
Minimum Heat out = 60
}
Net = -10
Heat in = 70 + Q
E
HOT
Heat out = 60 + Q
E
}
Net = -10
If we put excess heat into the process,
we must also remove this heat!
Excess Heating and Cooling
250
220
190
160
130
100
240
210
180
150
120
90
T
H
(F)
T
C
(F)
H
O
T
U
T
I
L
I
T
Y
C
O
L
D
U
T
I
L
I
T
Y
H = 50
H =40
H =20
H =-40
H =-80
50
10+Q
E
70
40+Q
E
60+Q
E
Q
E
Q
E
Excess steam or furnace capacity
requires excess cooling water.
Dont transfer heat across the pinch!
Multiple Utilities
250
220
190
160
130
100
240
210
180
150
120
90
T
H
(F)
T
C
(F)
H
O
T
U
T
I
L
I
T
Y
C
O
L
D
U
T
I
L
I
T
Y
H = 50
H =40
H =20
H =-40
H =-80
50
10
70
20
40
cooling
water
hot
water
We prefer to add heat at the lowest
possible temperature, and to remove
heat at the highest possible temperature!
Multiple utilities correspond to
multiple pinches.
T-H Diagram
Enthalpy, MBtu/hr
E
n
t
h
a
l
p
y
,

M
B
t
u
/
h
r
250
230
210
190
170
150
130
110
90
0 200 400 600
Hot
T-H Diagram
Enthalpy, MBtu/hr
E
n
t
h
a
l
p
y
,

M
B
t
u
/
h
r
250
230
210
190
170
150
130
110
90
0 200 400 600
Cold
T-H Diagram
Enthalpy, MBtu/hr
E
n
t
h
a
l
p
y
,

M
B
t
u
/
h
r
250
230
210
190
170
150
130
110
90
0 200 400 600
Cold
T-H Diagram
Enthalpy, MBtu/hr
E
n
t
h
a
l
p
y
,

M
B
t
u
/
h
r
250
230
210
190
170
150
130
110
90
0 200 400 600
Hot
T
min
= 10 F
Q
C,min
Q
H,min
Cold
Limitation of the
Procedure
We need to know
FC
p
values of all streams
Inlet and outlet temperatures of all
streams
BUT the design variables that
fix the process flows must be
determined from optimization
which in turn depends on heat-
exchanger network
Solution: Heat-exchanger
network as function of the flows
Some iterations are needed
An Exercise
Given the following stream data
below, find (assume T
min
= 10
deg C)
the minimum amount heat added
the minimum amount of cooling
the pinch temperature
180 C 60 C
150 C 30 C
135 C 20 C
140 C 80 C
FC
p
(kW/C)
3
1
2
5
Stream
1
2
3
4
Minimum Number of
Exchangers
From results of minimum
heating and cooling estimates
Hot Utility
70 MBtu/hr
Stream 1
130 MBtu/hr
Stream 2
400 MBtu/hr
Sources
Stream 3
180 MBtu/hr
Stream 4
360 MBtu/hr
Cold Utility
60 MBtu/hr
Sinks
70
110
20
340
60
Heat loads always balance as
result of first law analysis
Number of exchangers (N
E
) =
Number of streams (N
S
) +
Number of Utilities (N
U
) - 1
N
E
= 4 + 2 - 1 = 5
Minimum Number of
Exchangers
The previous solution is not
always correct. Consider
Hot Utility
230 MBtu/hr
Stream 1
130 MBtu/hr
Stream 2
400 MBtu/hr
Sources
Stream 3
180 MBtu/hr
Stream 4
360 MBtu/hr
Cold Utility
220 MBtu/hr
Sinks
230
180
130
220
We see that if we have an exact
matching between some
streams, we need less exchanger
N
E
= N
S
+ N
U
- N
P
N
P
= number of independent
subproblems
Minimum Number of
Exchangers
The previous solution is still not
always correct. Consider
Hot Utility
70
Stream 1
130 MBtu/hr
Stream 2
400 MBtu/hr
Sources
Stream 3
180 MBtu/hr
Stream 4
360 MBtu/hr
Cold Utility
220 MBtu/hr
Sinks
70-Q
E
340
110+Q
E
60
We see that there are now six
exchangers and there is now a
loop (i.e., HU -> S3 -> S1 -> S4
-> HU)
N
E
= N
S
+ N
U
+ N
L
- N
P
N
L
= number of loops
Q
E
20-Q
E
Effect of Pinch Analysis
The pinch analysis indicates that we only
use heat above the pinch and cooling below
the pinch, so that the design problem can
be decomposed into two subproblems.
250
220
190
160
130
100
240
210
180
150
120
90
T
H
(F)
T
C
(F)
Assume no loops
& no exact matches
N
E
= N
S
+ N
U
- 1
= 4 + 1 - 1 = 4
N
E
= N
S
+ N
U
- 1
= 3 + 1 - 1 = 3
N
TOTAL
= 7
Effect of Pinch Analysis
Minimum Exchangers:
N = 5
Pinch Analysis:
N = 7
(minimum energy)
A trade-off between captial cost
(minimum exchangers) and
utility costs (minimum energy)
We can decrease the number of
exchangers if we transfer some
heat across the pinch, but the
energy usage increases
Design of Minimum-
Energy HENs
250
220
190
160
130
100
240
210
180
150
120
90
T
H
(F)
T
C
(F)
Q = 110
Q = 20
Q = 240
Q = 160
Q = 120
Q = 60
Q = 360
Stream 1 2 3 4
FC
p
1000 4000 3000 6000
Pinch
Above the pinch: Q
tot
= 70 MBtu/hr, heat added
Above the pinch: Q
tot
= 60 MBtu/hr, heat removed
Design above the Pinch
250
220
190
160
130
100
240
210
180
150
120
90
T
H
(F)
T
C
(F)
Q = 110
Q = 240
Q = 60
Q = 360
Pinch
Feasible Pinch Matches:
(FC
p
)
HOT
< (FC
p
)
COLD
We can match stream 1 with either 3 or 4,
and we can only match stream 2 with 4.
Design above the Pinch
250
220
190
160
130
100
240
210
180
150
120
90
T
H
(F)
T
C
(F)
Q = 60
Q=240
Pinch
Transfer the maximum amount of heat
possible for each match to attemp to
eliminate streams from the problem.
Remaining Heat Loads
250
220
190
160
130
100
240
210
180
150
120
90
T
H
(F)
T
C
(F)
Pinch
Q = 110-60
= 50
Q = 360-240
= 120
170 F
200 F
Transfer the remaining heat from
stream 1 to stream 4.
Remaining Heat Loads
250
220
190
160
130
100
240
210
180
150
120
90
T
H
(F)
T
C
(F)
Pinch
Q = 50
Heat from Hot Utility
250
220
190
160
130
100
240
210
180
150
120
90
T
H
(F)
T
C
(F)
Pinch
178 F
H = 70
Design above the Pinch
(Summary)
250
220
190
160
130
100
240
210
180
150
120
90
T
H
(F)
T
C
(F)
Q = 60
Q=240
Pinch
1. Put in the matches at the pinch
2. Maximize the heat loads to eliminate
streams
3. See what is left
Q=50
H=70
Design above the Pinch
(Alternatives)
250
220
190
160
130
100
240
210
180
150
120
90
T
H
(F)
T
C
(F)
Q = 60
Q=240
Pinch
Q=50
H=70
Design below the Pinch
250
220
190
160
130
100
240
210
180
150
120
90
T
H
(F)
T
C
(F)
Q = 20
Q = 160
Q = 120
Pinch
Design below the Pinch
(One Alternative)
250
220
190
160
130
100
240
210
180
150
120
90
T
H
(F)
T
C
(F)
Pinch
Q = 120
C = 20
C = 40
T=120
A Complete Design
250
220
190
160
130
100
240
210
180
150
120
90
T
H
(F)
T
C
(F)
Q = 60
Q=240
Q=50
H=70
Q = 120 C = 20
C = 40
Total Number of Exchangers: 7
Remarks
Minimum approach temperature
So far, we have assumed a value
of 10 deg F
Trade-off: Larger minimum
approach temperature, smaller
heat-exchanger area but larger
minimum heating and cooling
Additional complexities
The design problem is not always
as simple as the example
considered
Stream splitting
Alternatives
Reducing the Number of
Exchangers
The number of exchangers required
for the overall process is always
less than or equal to that for the
minimum energy network
The minimum energy network
normally contains loops across the
pinch
These loops can be broken by
transferring heat across the pinch,
but we will introduce at least one
violation of the specified T
min
T
min
can be restored by shifting
heat along a path, which increases
energy consumption of the process
Loops
250
220
190
160
130
100
240
210
180
150
120
90
T
H
(F)
T
C
(F)
A set of connections that starts from one
stream and returns to the same stream.
Loops
250
220
190
160
130
100
240
210
180
150
120
90
T
H
(F)
T
C
(F)
A set of connections that starts from one
stream and returns to the same stream.
Loops
250
220
190
160
130
100
240
210
180
150
120
90
T
H
(F)
T
C
(F)
A set of connections that starts from one
stream and returns to the same stream.
Loops
250
220
190
160
130
100
240
210
180
150
120
90
T
H
(F)
T
C
(F)
A set of connections that starts from one
stream and returns to the same stream.
Breaking Loops
250
220
190
160
130
100
240
210
180
150
120
90
T
H
(F)
T
C
(F)
Q = 60
Q=240
Q=50
H=70
Q = 120 C = 20
C = 40
Break the loop with exchanger with the
small load
Remove the smallest heat load from loop
Breaking a loop across the pinch
normally violates the 2nd law
250
220
190
160
130
100
240
210
180
150
120
90
T
H
(F)
T
C
(F)
Q = 60
Q=240-20
Q=50+20
H=70
Q = 120
C = 40+20
BUT ...
250
220
190
160
130
100
240
210
180
150
120
90
T
H
(F)
T
C
(F)
Q = 60
Q=220
Q=70
H=70
Q = 120
C = 60
Hot Stream: 200 F to 120 F
Cold Stream: 150 F to 130 F
Impossible according to 2nd Law!
Restoring T
min
Q=60+Q
E
Q=220
Q=70-Q
E
H=70+Q
E
C = 60+Q
E
Shift heat along a path, which is a
connection between a cooler and a heater
Thus, 1000(60+Q
E
) = 3000(150-110)
Q
E
= 60 MBtu/hr
< 110 F
T=120
Q = 120- Q
E
Reducing Number of
Exchangers
Q=120
Q=220
Q=10
H=130
C = 120
T=110
T=250
T=240
T=120
T=90
T=150
T=200
T=145
T=115
T=100
T=190
T=168.4
T=166.6
T=130
Total number of Exchangers: 6
Q = 60
Restoring T
min
(An Alternative Path)
Q=60
Q=220-Q
E
Q=70
H=70+Q
E
C = 60+Q
E
< 110 F
T=120
But this path cant restore T
min
!
Q=120
Breaking Another Loop
Q=120
Q=220
Q=10
H=130
C = 120
Q=60
Heat Engines
Heat
Engine
Q
in
+W
Q
in
W
Energy
Cascade
Q
out
Energy
Cascade
Q
in
Energy
Cascade
Q
out
Energy
Cascade
Pinch
Heat
Engine
Q
out
-W
Q
in
W
Energy
Cascade
Q
out
Energy
Cascade
Heat
Engine
Q
E
-W
Q
in
W
Energy
Cascade
Q
out
+Q
E
-W
Energy
Cascade
Q
E
Efficiency: 100% 100% Sandard Alone
Place Heat Engine either above
or below the pinch (not across)!
Heat Pumps
Heat
Pump
Q
in
- W
Q
E
W
Energy
Cascade
Q
out
Energy
Cascade
Q
in
Energy
Cascade
Q
out
Energy
Cascade
Place heat pump across the pinch!
Q
E
+W
Pinch
Heat
Pump
Q
in
Q
E
W
Energy
Cascade
Q
out
+ W
Energy
Cascade
Q
E
- W
Heat
Pump
Q
in
- (Q
E
+W)
Q
E
W
Energy
Cascade
Q
out
- Q
E
Energy
Cascade
Q
E
+W
Distillation
Column as a heat engine
Column
Heat In (Q
Reb
)
Heat Out (Q
Cond
)
Column either above or below
the pinch
Q
in
Energy
Cascade
Q
out
Energy
Cascade
Column
Q
in
Energy
Cascade
Q
out
Energy
Cascade
Column
HDA Process
Reactor
Flash
compressor
S
t
a
b
i
l
i
z
e
r
P
r
o
d
u
c
t
R
e
c
y
c
l
e
cool heat
Why Energy Integration?
(ICI Experience)
20 case studies
In every case, there was a reduction in energy
In almost every case, the energy savings required
less capital
Up to 60% energy savings, up to 25% capital
savings, up to 15% lower product price
Why spend additional capital to waste energy?
Energy Integration
What are the minimum cooling and heating
requirements?
What is the minimum number of heat
exchangers required?
What are the main trade-offs?
An Example
For the two hot streams being cooled and
the two cold streams being heated shown
below, find the minimum heating and
cooling requirements, as well as the
minimum number of heat exchangers if
T
min
= 10 deg F.
250 F 120 F
200 F 100 F
150 F 90 F
190 F 130 F
FC
p
(Btu/F)
1000
4000
3000
6000
Stream
1
2
3
4
Net Energy Required
(First Law Calculation)
250 F 120 F
200 F 100 F
150 F 90 F
190 F 130 F
FC
p
Q
aval
(Btu/F) (MBtu/hr)
1000 130
4000 400
3000 -180
6000 -360
-10
Thus, 10 x10
3
Btu/hr would have to be
supplied from a hot utility if there were
no requirement of T
min
= 10 deg F.
Question: What are the minimum
heating and cooling requirements?
Net Energy Required at
Temperature Intervals
Define hot and cold temperature scales
shifted by 10 deg F and show streams
250
220
190
160
130
100
240
210
180
150
120
90
T
H
(F)
T
C
(F)
Net Energy Required at
Temperature Intervals
Draw in intervals by breaking at
stream starting and ending points
250
220
190
160
130
100
240
210
180
150
120
90
T
H
(F)
T
C
(F)
Net Energy Required at
Temperature Intervals
Compute net available Q for each interval
250
220
190
160
130
100
240
210
180
150
120
90
T
H
(F)
T
C
(F)
FC
p 1000 4000 3000 6000
Q
aval
(MBtu/hr)
50
-40
-80
40
20
Note: Sum of net heat available =-10 MBtu/hr
Same as first law
Cascade Diagram
We satisfy the net requirement for each
interval from an external utility.
250
220
190
160
130
100
240
210
180
150
120
90
T
H
(F)
T
C
(F)
H
O
T
U
T
I
L
I
T
Y
C
O
L
D
U
T
I
L
I
T
Y
H =50
H =40
H =20
H =-40
H =-80
BUT ...
Cascade Diagram
We can always transfer excess heat from
high temperature intervals to lower
temperature intervals without violating
T
min
=10 F
250
220
190
160
130
100
240
210
180
150
120
90
T
H
(F)
T
C
(F)
H
O
T
U
T
I
L
I
T
Y
C
O
L
D
U
T
I
L
I
T
Y
H = 50
H =40
H =20
H =-40
H =-80
50
10
70
40
60
Minimum Heating
and Cooling
250
220
190
160
130
100
240
210
180
150
120
90
T
H
(F)
T
C
(F)
H
O
T
U
T
I
L
I
T
Y
C
O
L
D
U
T
I
L
I
T
Y
H = 50
H =40
H =20
H =-40
H =-80
50
10
70
40
60
Minimum heating = 70 MBtu/hr
Minimum cooling = 60 MBtu/hr
Net = 10 MBtu/hr of heating
Same as first law
Pinch Temperature
250
220
190
160
130
100
240
210
180
150
120
90
T
H
(F)
T
C
(F)
H
O
T
U
T
I
L
I
T
Y
C
O
L
D
U
T
I
L
I
T
Y
H = 50
H =40
H =20
H =-40
H =-80
50
10
70
40
60
The design problem can be decomposed
into two separate problems:
1. A high temperature region where only
heating is requred
2. A low temperature region where only
cooling is required
Relationship to First Law
Minimum Heat in = 70
HOT
COLD
Minimum Heat out = 60
}
Net = -10
Heat in = 70 + Q
E
HOT
Heat out = 60 + Q
E
}
Net = -10
If we put excess heat into the process,
we must also remove this heat!
Excess Heating and Cooling
250
220
190
160
130
100
240
210
180
150
120
90
T
H
(F)
T
C
(F)
H
O
T
U
T
I
L
I
T
Y
C
O
L
D
U
T
I
L
I
T
Y
H = 50
H =40
H =20
H =-40
H =-80
50
10+Q
E
70
40+Q
E
60+Q
E
Q
E
Q
E
Excess steam or furnace capacity
requires excess cooling water.
Dont transfer heat across the pinch!
Multiple Utilities
250
220
190
160
130
100
240
210
180
150
120
90
T
H
(F)
T
C
(F)
H
O
T
U
T
I
L
I
T
Y
C
O
L
D
U
T
I
L
I
T
Y
H = 50
H =40
H =20
H =-40
H =-80
50
10
70
20
40
cooling
water
hot
water
We prefer to add heat at the lowest
possible temperature, and to remove
heat at the highest possible temperature!
Multiple utilities correspond to
multiple pinches.
T-H Diagram
Enthalpy, MBtu/hr
E
n
t
h
a
l
p
y
,

M
B
t
u
/
h
r
250
230
210
190
170
150
130
110
90
0 200 400 600
Hot
T-H Diagram
Enthalpy, MBtu/hr
E
n
t
h
a
l
p
y
,

M
B
t
u
/
h
r
250
230
210
190
170
150
130
110
90
0 200 400 600
Cold
T-H Diagram
Enthalpy, MBtu/hr
E
n
t
h
a
l
p
y
,

M
B
t
u
/
h
r
250
230
210
190
170
150
130
110
90
0 200 400 600
Cold
T-H Diagram
Enthalpy, MBtu/hr
E
n
t
h
a
l
p
y
,

M
B
t
u
/
h
r
250
230
210
190
170
150
130
110
90
0 200 400 600
Hot
T
min
= 10 F
Q
C,min
Q
H,min
Cold
Limitation of the
Procedure
We need to know
FC
p
values of all streams
Inlet and outlet temperatures of all
streams
BUT the design variables that
fix the process flows must be
determined from optimization
which in turn depends on heat-
exchanger network
Solution: Heat-exchanger
network as function of the flows
Some iterations are needed
An Exercise
Given the following stream data
below, find (assume T
min
= 10
deg C)
the minimum amount heat added
the minimum amount of cooling
the pinch temperature
180 C 60 C
150 C 30 C
135 C 20 C
140 C 80 C
FC
p
(kW/C)
3
1
2
5
Stream
1
2
3
4
Minimum Number of
Exchangers
From results of minimum
heating and cooling estimates
Hot Utility
70 MBtu/hr
Stream 1
130 MBtu/hr
Stream 2
400 MBtu/hr
Sources
Stream 3
180 MBtu/hr
Stream 4
360 MBtu/hr
Cold Utility
60 MBtu/hr
Sinks
70
110
20
340
60
Heat loads always balance as
result of first law analysis
Number of exchangers (N
E
) =
Number of streams (N
S
) +
Number of Utilities (N
U
) - 1
N
E
= 4 + 2 - 1 = 5
Minimum Number of
Exchangers
The previous solution is not
always correct. Consider
Hot Utility
230 MBtu/hr
Stream 1
130 MBtu/hr
Stream 2
400 MBtu/hr
Sources
Stream 3
180 MBtu/hr
Stream 4
360 MBtu/hr
Cold Utility
220 MBtu/hr
Sinks
230
180
130
220
We see that if we have an exact
matching between some
streams, we need less exchanger
N
E
= N
S
+ N
U
- N
P
N
P
= number of independent
subproblems
Minimum Number of
Exchangers
The previous solution is still not
always correct. Consider
Hot Utility
70
Stream 1
130 MBtu/hr
Stream 2
400 MBtu/hr
Sources
Stream 3
180 MBtu/hr
Stream 4
360 MBtu/hr
Cold Utility
220 MBtu/hr
Sinks
70-Q
E
340
110+Q
E
60
We see that there are now six
exchangers and there is now a
loop (i.e., HU -> S3 -> S1 -> S4
-> HU)
N
E
= N
S
+ N
U
+ N
L
- N
P
N
L
= number of loops
Q
E
20-Q
E
Effect of Pinch Analysis
The pinch analysis indicates that we only
use heat above the pinch and cooling below
the pinch, so that the design problem can
be decomposed into two subproblems.
250
220
190
160
130
100
240
210
180
150
120
90
T
H
(F)
T
C
(F)
Assume no loops
& no exact matches
N
E
= N
S
+ N
U
- 1
= 4 + 1 - 1 = 4
N
E
= N
S
+ N
U
- 1
= 3 + 1 - 1 = 3
N
TOTAL
= 7
Effect of Pinch Analysis
Minimum Exchangers:
N = 5
Pinch Analysis:
N = 7
(minimum energy)
A trade-off between captial cost
(minimum exchangers) and
utility costs (minimum energy)
We can decrease the number of
exchangers if we transfer some
heat across the pinch, but the
energy usage increases
Design of Minimum-
Energy HENs
250
220
190
160
130
100
240
210
180
150
120
90
T
H
(F)
T
C
(F)
Q = 110
Q = 20
Q = 240
Q = 160
Q = 120
Q = 60
Q = 360
Stream 1 2 3 4
FC
p
1000 4000 3000 6000
Pinch
Above the pinch: Q
tot
= 70 MBtu/hr, heat added
Above the pinch: Q
tot
= 60 MBtu/hr, heat removed
Design above the Pinch
250
220
190
160
130
100
240
210
180
150
120
90
T
H
(F)
T
C
(F)
Q = 110
Q = 240
Q = 60
Q = 360
Pinch
Feasible Pinch Matches:
(FC
p
)
HOT
< (FC
p
)
COLD
We can match stream 1 with either 3 or 4,
and we can only match stream 2 with 4.
Design above the Pinch
250
220
190
160
130
100
240
210
180
150
120
90
T
H
(F)
T
C
(F)
Q = 60
Q=240
Pinch
Transfer the maximum amount of heat
possible for each match to attemp to
eliminate streams from the problem.
Remaining Heat Loads
250
220
190
160
130
100
240
210
180
150
120
90
T
H
(F)
T
C
(F)
Pinch
Q = 110-60
= 50
Q = 360-240
= 120
170 F
200 F
Transfer the remaining heat from
stream 1 to stream 4.
Remaining Heat Loads
250
220
190
160
130
100
240
210
180
150
120
90
T
H
(F)
T
C
(F)
Pinch
Q = 50
Heat from Hot Utility
250
220
190
160
130
100
240
210
180
150
120
90
T
H
(F)
T
C
(F)
Pinch
178 F
H = 70
Design above the Pinch
(Summary)
250
220
190
160
130
100
240
210
180
150
120
90
T
H
(F)
T
C
(F)
Q = 60
Q=240
Pinch
1. Put in the matches at the pinch
2. Maximize the heat loads to eliminate
streams
3. See what is left
Q=50
H=70
Design above the Pinch
(Alternatives)
250
220
190
160
130
100
240
210
180
150
120
90
T
H
(F)
T
C
(F)
Q = 60
Q=240
Pinch
Q=50
H=70
Design below the Pinch
250
220
190
160
130
100
240
210
180
150
120
90
T
H
(F)
T
C
(F)
Q = 20
Q = 160
Q = 120
Pinch
Design below the Pinch
(One Alternative)
250
220
190
160
130
100
240
210
180
150
120
90
T
H
(F)
T
C
(F)
Pinch
Q = 120
C = 20
C = 40
T=120
A Complete Design
250
220
190
160
130
100
240
210
180
150
120
90
T
H
(F)
T
C
(F)
Q = 60
Q=240
Q=50
H=70
Q = 120 C = 20
C = 40
Total Number of Exchangers: 7
Remarks
Minimum approach temperature
So far, we have assumed a value
of 10 deg F
Trade-off: Larger minimum
approach temperature, smaller
heat-exchanger area but larger
minimum heating and cooling
Additional complexities
The design problem is not always
as simple as the example
considered
Stream splitting
Alternatives
Reducing the Number of
Exchangers
The number of exchangers required
for the overall process is always
less than or equal to that for the
minimum energy network
The minimum energy network
normally contains loops across the
pinch
These loops can be broken by
transferring heat across the pinch,
but we will introduce at least one
violation of the specified T
min
T
min
can be restored by shifting
heat along a path, which increases
energy consumption of the process
Loops
250
220
190
160
130
100
240
210
180
150
120
90
T
H
(F)
T
C
(F)
A set of connections that starts from one
stream and returns to the same stream.
Loops
250
220
190
160
130
100
240
210
180
150
120
90
T
H
(F)
T
C
(F)
A set of connections that starts from one
stream and returns to the same stream.
Loops
250
220
190
160
130
100
240
210
180
150
120
90
T
H
(F)
T
C
(F)
A set of connections that starts from one
stream and returns to the same stream.
Loops
250
220
190
160
130
100
240
210
180
150
120
90
T
H
(F)
T
C
(F)
A set of connections that starts from one
stream and returns to the same stream.
Breaking Loops
250
220
190
160
130
100
240
210
180
150
120
90
T
H
(F)
T
C
(F)
Q = 60
Q=240
Q=50
H=70
Q = 120 C = 20
C = 40
Break the loop with exchanger with the
small load
Remove the smallest heat load from loop
Breaking a loop across the pinch
normally violates the 2nd law
250
220
190
160
130
100
240
210
180
150
120
90
T
H
(F)
T
C
(F)
Q = 60
Q=240-20
Q=50+20
H=70
Q = 120
C = 40+20
BUT ...
250
220
190
160
130
100
240
210
180
150
120
90
T
H
(F)
T
C
(F)
Q = 60
Q=220
Q=70
H=70
Q = 120
C = 60
Hot Stream: 200 F to 120 F
Cold Stream: 150 F to 130 F
Impossible according to 2nd Law!
Restoring T
min
Q=60+Q
E
Q=220
Q=70-Q
E
H=70+Q
E
C = 60+Q
E
Shift heat along a path, which is a
connection between a cooler and a heater
Thus, 1000(60+Q
E
) = 3000(150-110)
Q
E
= 60 MBtu/hr
< 110 F
T=120
Q = 120- Q
E
Reducing Number of
Exchangers
Q=120
Q=220
Q=10
H=130
C = 120
T=110
T=250
T=240
T=120
T=90
T=150
T=200
T=145
T=115
T=100
T=190
T=168.4
T=166.6
T=130
Total number of Exchangers: 6
Q = 60
Restoring T
min
(An Alternative Path)
Q=60
Q=220-Q
E
Q=70
H=70+Q
E
C = 60+Q
E
< 110 F
T=120
But this path cant restore T
min
!
Q=120
Breaking Another Loop
Q=120
Q=220
Q=10
H=130
C = 120
Q=60
Heat Engines
Heat
Engine
Q
in
+W
Q
in
W
Energy
Cascade
Q
out
Energy
Cascade
Q
in
Energy
Cascade
Q
out
Energy
Cascade
Pinch
Heat
Engine
Q
out
-W
Q
in
W
Energy
Cascade
Q
out
Energy
Cascade
Heat
Engine
Q
E
-W
Q
in
W
Energy
Cascade
Q
out
+Q
E
-W
Energy
Cascade
Q
E
Efficiency: 100% 100% Sandard Alone
Place Heat Engine either above
or below the pinch (not across)!
Heat Pumps
Heat
Pump
Q
in
- W
Q
E
W
Energy
Cascade
Q
out
Energy
Cascade
Q
in
Energy
Cascade
Q
out
Energy
Cascade
Place heat pump across the pinch!
Q
E
+W
Pinch
Heat
Pump
Q
in
Q
E
W
Energy
Cascade
Q
out
+ W
Energy
Cascade
Q
E
- W
Heat
Pump
Q
in
- (Q
E
+W)
Q
E
W
Energy
Cascade
Q
out
- Q
E
Energy
Cascade
Q
E
+W
Distillation
Column as a heat engine
Column
Heat In (Q
Reb
)
Heat Out (Q
Cond
)
Column either above or below
the pinch
Q
in
Energy
Cascade
Q
out
Energy
Cascade
Column
Q
in
Energy
Cascade
Q
out
Energy
Cascade
Column
Radiation Heat Transfer
4 Electromagnetic radiation occurs between all
bo2.4dies
4 The spectrum ranges from 10
-4
to 10
+4
m
4 Our concern is over a narrow spectrum
10
-1
to 10
+2
m
the visible through the infrared
4 Radiation in gases and transparent materials is a
volumetric phenomenon.
4 Our interest is in radiation as a surface phenomenon
4 Radiation originates due to emission by matter |
Transport does not require matter
4 Radiation can be viewed as propagation of photons or
quamta or of electromagnetic waves
Properties of Radiation
Wave properties
- frequency (radians)
- wave length (micron)
c - speed of light (2.998 x 10
8
m/s
= c/
Energy spectrum
spectral distribution
distribution of wace lengths or energy
directionality
directional distribution
Radiation Intensity
Radiation emission from a body
Consider emission in a particular direction from an
area dA
1
Direction is specified in spherical coordinates
by a zenith angle, , and
by an azimuthal angle,
The differential area, dA
n,
subtends a solid angle d
when viewed from a point on dA
1
The area dA
n
is normal to (,) directions, so
Then d = sin d d

d =
dA
n

2

dA
n
= r
2
sin d d
Spectral Intensity
The rate at which emission passes
from dA
1
through dA
n
Formal definition
The spectral intensity, I
,e,
is the rate at which radiant
energy is emitted at wave length, , in the (,)
direction, per unit area of the emitting surface normal
to this direction, per unit solid angle about this
direction, per unit wavelength interval d about
Or in other forms

I
,e
,, =
dq
dA
1
cos d

dq

= I
,e
,, dA
1
cos sin d d

dq

= I
,e
,, dA
1
cos d
Spectral Heat Flux and Emission
Spectral Heat Flux
Total Heat Flux
Spectral Hemispherical Emissive Power
Total Emissive Power

q

"
= I
,e
,, dA
1
cos sin d d
0
/ 2
0
2

q" = q

"
d
0


E = I
,e
,, dA
1
cos sin d d
0
/ 2
0
2

E = E

d
0

Incident radiation and radiosity


Spectral irradiation
All incident radiation from all directions
Radiosity
all the radiant energy leaving a surface
including the reflected radiation

G = I
,e
,, dA
1
cos sin d d
0
/ 2
0
2

G = G

d
0


J = I
,e + r
,, dA
1
cos sin d d
0
/ 2
0
2

J = J

d
0

Black Body Radiation


Definitions
A Black Body is an ideal surface having the
following properties :
A black body absorbs all incident
radiation regardless of wavelength or
direction
For a prescribed temperature and wave
length, no surface can emit more energy
than a black body.
Although radiation emitted by a black
body is a function of wave length and
temperature, itr is independent of
direction; that is, a bl;ack body is a
diffuse emitter
The Planck Distribution
Spectral distribution of black body emission
Planck Distribution (Emissive Power)
Features
Emission is continuous in wave length
Magnitude increases with temperature
More radiation at shorter wave lengths with T+

I
,b
,T =
2hc
0
2

2
e
hc
0
kT
hc
0
kT
1

h is the universal Planck's constant (6.6256 x 10
3 4
Js)

k is Boltzmanns constant (1.3805 x 10
2 3
J / K

E
, b
,T = I
,b
,T =
C
1

2
e
C
2
T
C
2
T
1
Wiens Displacement Law
Black body spectral distribution has a maximum.
The corresponding wavelength
m
= f(T)
If we find the maximum, we obtain
Wiens Displacement Law

m
T = C
3
= 2897.8 m/K
The Stefan-Boltzmann Law
The Planck distribution introduced into the relation for
the total emissive power of a black body becomes
On integration, it becomes the Stefan-Boltzmann Law
where = 5.670 x 10
-8
W/m
2
-K
4
Remember that black body radiation is diffuse so:

E
b
=
C
1

2
e
C2
T
C2
T
1
d
0


E
b
= T
4

I
b
=
E
b

Band Emission
It is often important to know the fraction of energy
emitted from a black body over a certain range of
wavelengths.
For a range from 0 to , the fraction is F
(0-)
Over a specific interval, we might write:

F
0
=
E
,b
d
0

E
,b
d
0

=
E
,b
d
0

T
4
=
E
,b
T
4
d T
0

1

2
=
E
,b
d

2
E
,b
d
0

=
E
,b
d

2
T
4

which is : F

1

2
= F
0
\1
F
0
2
Surface Emission Emission
The black body is an ideal emitting surface. How
can we describe the behavior of real surfaces. For
this we use the black body as a reference!!
The emissivity is defined as the ratio of the radiation
emitted by a real surface to that emitted by a black body
at the same temperature
A real surface may not be a diffuse emitter, so the
emissivity may have different values depending on
wavelength and direction
Spectral directional emissivity
Total directional emissivity
Spectral hemispherical emissivity

,
, , , T =
I
,e
, , , T
I
,b

, , T =
I
e
, , T
I
b

, T =
E

, T
E
,b
Emissivities
We can make the following observations
regarding the values of emissivities
Surface Absorption, Reflection, and
Transmission
4 The Spectral Irradiation G

is the radiation flux of


energy at wavelength per unit d about . The flux
may originate from different sources
4 The source may be reflected, absorbed, or transmitted.
4 Reflection and absorption of radiant energy is what we
might view a s color
Absorptivity

,
=
I
,abs
,,
I
,abis
,,

,
I
,i
,, dA
1
cos sin d d
0
/ 2
0
2
I
,i
,, dA
1
cos sin d d
0
/ 2
0
2

G = I
,i
,, dA
1
cos sin d d
0
/ 2
0
2

=
G
,abs
G

Spectral directional absorptivity


Spectral hemispherical absorptivity
On Absorptivity

=
G
abs
G
Total hemispherical absorptivity

=

d
0

d
0

Reflectivity

,
,, =
I
, i , r e f
,,
I
,i
,,

=
G
,ref


=
G
ref
G
Spectral directional reflectivity
Spectral hemispherical reflectivity
Total hemispherical reflectivity
Transmissivity
Spectral hemispherical transmissivity
Hemispherical transmissivity

=
G
,tr

G

=
G
tr
G
Radiation balances
For semitransparent media
For opaque media

= 1

+ + = 1

= 1

+ = 1
Kirchhoffs Law
Consider a large isothermal enclosure of surface
temperature T
s
, In the enclosure are several small
bodies. These have a negligible influence on the
radiation field,
The surface forms a black body cavity, that is,
G = E
b
(T
s
)
Energy balance on body 1
GA
1
- E
1
(T
s
) A
1
Or
Applied to each body, the relation yields Kirchhoffs
Law

E
1
T
s

= E
b
T
s

E
1
T
s

1
=
E
1
T
s

2
= ..... = E
b
T
s
More on Kirchhoffs Law
From the definition of total hemispherical
emissivity, we obtain an equivalent representation
of Kirchhoffs Law

1

1
=

2
=

3
= .... = 1

=

,
=
,
ChE 333 - First Examination 1 February 28, 2000
Chemical Engineering 333
Heat Transfer
First Examination 28 February , 2000
Part A. (Closed Book)
(25 points)
a) Describe the three mechanisms for Heat Transfer (10)
b) What is Fouriers Law of heat conduction? (5)
c) What is an Local Heat Transfer Coefficient ? (5)
d) What is difference between the Nusselt Number and the Biot Number? (5)
I. (25 points)
A composite structure that is 2 meters wide
is composed of four materials, each of
differing thickness and height.
Estimate the heat flow through the structure if surfaces at constant x are isothermal and
there id no heat transfer normal to the x-direction.
T
1
= 100C ; T
0
= 0C ;
Material A is 5 cm thick and 10 cm high k
A
= 1 W/m-K
Material B is 12 cm thick and 4 cm high k
B
= 0.3 W/m-K
Material C is 12 cm thick and 6 cm high k
C
= 0.6 W/m-K
Material D is 4 cm thick and 10 cm high k
D
= 10 W/m-K
Hint: Develop the equivalent electrical circuit.
x
A
B
C
D
QQ
QQ
T
1
T
0
ChE 333 - First Examination 2 February 28, 2000
ChE 333 Heat Transfer
First Examination
Part B (Open Book)
3. (25 points)
On the flight of Apollo 12, plutonium oxide (PuO
2
) was used to generate electrical power.
Heat was generated uniformly through the loss of kinetic energy from alpha particles
emitted by the Pu
238
. Consider that the sphere of plutonium oxide is 3 cm in diameter and
is covered with thermo-electric elements for converting heat to electricity. The physical
properties of these telluride elements and heat rejection considerations suggest that the
surface temperature must be 200C. The ceramic nature of the PuO
2
allows a maximum
temperature of 1750C, With these constraints, determine the maximum allowable
volumetric heating rate.
4. (25 points)
A cylindrical steel rod is connected to two
steel plates, each at a different temperature,
T
1
and T
2
respectively, The rod is cooled by
air passing past the rod. The convective heat
transfer coefficient is h.
How does the average temperature vary
along the rod ?
The rod is 4 cm in diameter and 50 cm long,
T
1
= 100C ; T
0
= 0C ;
h = 11 W/m
2-
K k
s
= 13 W/m-K
Note and hints
:
If you solve the problem analytically, you will get full credit
If you calculate the numbers in addition, you get extra credit
Make a shell balance for heat flow through and from a differential length of rod x.
Find the corresponding differential equation
State the boundary conditions
Express the equation boundary conditions in dimensionless form
Solve the equation (Incropera & DeWitt section 3.6)
T
a
T
1
T
0
ChE 333 - First Examination 3 February 28, 2000
ChE 333 - First Examination 1 February 28, 2000
Chemical Engineering 333
Heat Transfer
Solution to the First Examination 28 February , 2000
Part A. (Closed Book)
Problem 1 (25 points)
a) Describe the three mechanisms for Heat Transfer (15)
Heat transfer can occur by three distinct mechanisms, conduction, convection, and
radiation, Conduction occurs down a temperature gradient in a material,
Conduction in a gas is occurs by molecular collisions. In solids or liquids, heat is
transferred by vibrations or phonon transport. The process is described best by
Fouriers Law. Convection is process of energy being carried by a fluid. The
details of the convection process are governed by the energy equation and by the
conservation equation for linear momentum. In lieu of knowledge of detailed
temperature field, convection from a surface is governed by Newtons law of
cooling, q
I
n
I
= h(T
s
T
a
). Radiation is the mechanism by which electromagnetic
radiation is transported to a surface. The heat flux from a surface follows
Boltzmanns Law and is a function of the surface temperature to the 4
th
power.
b) What is Fouriers Law of heat conduction?
Fourierss Law states that the heat flux vector is a linear function of the temperature
gradient. The proportionality constant is termed the thermal conductivity.

q = k T
c) What is a Local Heat Transfer Coefficient?
As described above the local heat transfer coefficient is the proportionality constant
in Newtons Law of Cooling.

q n = h T T
b
ChE 333 - First Examination 2 February 28, 2000
Typical Values of Heat Transfer Coefficients
d) What is difference between the Nusselt Number and the Biot Number?
The Nusselt and Biot Numbers are the ratio of resistances of a conductive process to that of
a convective process.
The Nusselt number measures the resistance to convection and conduction in the same
phase. The Biot number is the ratio of the conductive resistance in a solid phase to
convective resistance in the adjacent fluid phase.
h (watts/m
2
-K)
Free Convection 5 - 25
Forced Convection
gases 5-250
liquids 50 - 20,000
Phase Change
(boiling or condensation) 2500-100,000

Nu =
hL
k
; Bi =
hL
k
s
ChE 333 - First Examination 3 February 28, 2000
Solution to Problem 2 (25 points)
A composite structure that is 2 meters wide
is composed of four materials, each of
differing thickness and height.
Estimate the heat flow through the structure if surfaces at constant x are isothermal and
there id no heat transfer normal to the x-direction.
T
1
= 100C ; T
0
= 0C ;
Material A is 5 cm thick and 10 cm high k
A
= 1 W/m-K
Material B is 12 cm thick and 4 cm high k
B
= 0.3 W/m-K
Material C is 12 cm thick and 6 cm high k
C
= 0.6 W/m-K
Material D is 4 cm thick and 10 cm high k
D
= 10 W/m-K
Hint: Develop the equivalent electrical circuit.
Conduction through each element of the composite is described by
These can be rewritten to solve for the overall temperature difference (T
1
T
0
).
The relations are as below:
x
A
B
C
D
QQ
QQ
T
1
T
0

Q
A
=
k
A
L
A
S
A
T
1
T
2

Q
B
=
k
B
L
B
S
B
T
2
T
3
; Q
C
=
k
C
L
C
S
C
T
2
T
3

Q
D
=
k
D
L
D
S
D
T
3
T
0

Q
D
= Q
A
= Q
B
+ Q
C

T
1
T
2
=
Q
A
k
A
L
A
S
A

Q
A
= Q
B
+ Q
C
=
k
B
L
B
S
B
+
k
C
L
C
S
C
T
2
T
3

T
2
T
3
=
Q
A
k
B
L
B
S
B
+
k
C
L
C
S
C

T
3
T
0
=
Q
A
k
D
L
D
S
D
ChE 333 - First Examination 4 February 28, 2000
So that we have
and finally
R
I
= L
i
/(k
i
S
I
) so that R
A
= 0.5 K/W ; R
D
= 0.02 K/W
R
B
= 5.0 K/W ; R
C
= 1.67 K/W ;
R
BC
= (1/R
B
+ 1/R
C
) = 2.6 K/W
R = R
A
+ R
BC
+ R
D
= 3.12 K/W
Then Q = (T
1
T
0
)/R = 312 Watts

T
1
T
0
=
Q
A
k
A
L
A
S
A
+
Q
A
k
B
L
B
S
B
+
k
C
L
C
S
C
+
Q
A
k
D
L
D
S
D

Q
A
=
T
1
T
0
1
k
A
L
A
S
A
+
1
k
B
L
B
S
B
+
k
C
L
C
S
C
+
1
k
D
L
D
S
D
ChE 333 - First Examination 5 February 28, 2000
ChE 333 Heat Transfer
First Examination
Part B (Open Book)
Solution to Problem 3 (25 points)
On the flight of Apollo 12, plutonium oxide (PuO
2
) was used to generate electrical power.
Heat was generated uniformly through the loss of kinetic energy from alpha particles
emitted by the Pu
238
. Consider that the sphere of plutonium oxide is 3 cm in diameter and
is covered with thermo-electric elements for converting heat to electricity. The physical
properties of these telluride elements and heat rejection considerations suggest that the
surface temperature must be 200C. The ceramic nature of the PuO
2
allows a maximum
temperature of 1750C, With these constraints, determine the maximum allowable
volumetric heating rate.
The shell balance on a spherical shell is :
The equation is obtained by taking the limit of r -> 0
\
At r = 0, T is finite , so C
1
= 0 and at r = R, T = T
0
so
The maximum volumetric energy production must
be as below:
For this system, k = 4 W/M-K, then the power maximum is:

4r
2
q
r
r
4r
2
q
r
r + r
+ 4r
2
r R
v
= 0

q
r
= k
dT
dr

d r
2
q
r
dr
+ r
2
R
v
= 0

d
dr
r
2dT
dr
+ r
2
R
v
k
= 0

T T
0
=
R
2
6

r
2
6
R
v
k

R
v
= 6 k
T
1
T
0
R
2
=
6 4 W/ m K 1550 K
0.03m
2
ChE 333 - First Examination 6 February 28, 2000
R
v
= 41.3 MW/m
3
ChE 333 - First Examination 7 February 28, 2000
Solution to Problem 4 (25 points)
A cylindrical steel rod is connected to two
steel plates each at a different temperature, T
1
and T
2
respectively. The rod is cooled by air
at T
a
passing past the rod. The convective
heat transfer coefficient is h.
How does the average temperature vary
along the rod?
The rod is 4 cm in diameter and 50 cm long,
T
1
= 100C ; T
0
= 0C ; T
a
= 20C ;
h = 11 W/m
2-
K k
s
= 13 W/m-K
Note and hints
:
If you solve the problem analytically, you will get full credit
If you calculate the numbers in addition,
You get extra credit
Make a shell balance for heat flow through and from a differential length of rod x.
Find the corresponding differential equation
State the boundary conditions
Express the equation boundary conditions in dimensionless form
Solve the equation (Incropera & DeWitt section 3.6)
The shell balance yields
And the limiting process provides
We apply Fouriers Law
And use the following dimensionless temperature and position :
T
a
T
1
T
0

R
2
q
x
x
Rq
x
x + x
Rx h T T
a
= 0

D
2
4
d q
x
dx
D h T T
a
= 0

=
T T
a
T
0
T
a
; =
x
L

q
x
= k
dT
dx
ChE 333 - First Examination 8 February 28, 2000
The resulting differential equation is as in the fin problems:
The Boundary conditions are
And the modulus m is defined as:
The solution is of the form
So that when we use the boundary conditions, we obtain
The modulus is
m = 2.30

D
2
4
d
2

d
2
+
h D
k
L
D
2
= 0

=
0
at = 0 ; =
1
at = 1

m
2
=
h D
k
L
D
2

=
1
sinh(m)
sinh (m)
+
0
cosh(m)sinh(m) cosh(m)sinh(m)
sinh (m)

= A sinh (m) + B cosh (m)
(0) = A sinh (0) + B cosh (0) =
0
(1) = A sinh (m) + B cosh (m) =
1

m
2
=
h D
k
L
D
2
=
11 (.04)
13
0.5
.04
2
= 5.288
ChE 333 - First Examination 9 February 28, 2000
m = 2.30 cosh(m)= 5.03722
sinhh(m)= 4.93696
sinh(m cosh(m
0 1 0 1
0.1 0.60182 0.23203 1.02657
0.2 0.23563 0.4764 1.10768
0.3 -0.11805 0.74607 1.24765
0.4 -0.478 1.05539 1.4539
0.5 -0.86335 1.42078 1.73741
0.6 -1.29457 1.86166 2.11324
0.7 -1.79458 2.40146 2.60135
0.8 -2.38994 3.06886 3.22768
0.9 -3.11229 3.89932 4.0255
1 -4 4.93696 5.03722

0
= 1 at = 0 ;
1
= 4 at = 1
Temperature Profile
-5
-4
-3
-2
-1
0
1
0 0.2 0.4 0.6 0.8 1
Position
Series1
ChE 333 - Second Examination 1 April 3, 2000
Chemical Engineering 333
Heat Transfer
Second Examination 3 April, 2000
Part A. (Closed Book)
Problem 1 (20 points)
Define and describe the following:
a) Forced Convection
b) Free Convection
c) Rayleigh Number
d) Peclet Number
Problem 2 (20 points)
Your new job at Armour Foods is in the design of the cooking and sterilization system
for canned hams. The current process uses steam at 120C to cook canned hams to a
centerline temperature of 70C in 1 hour. The process heats a can containing a1.5 kg.
Ham. The marketing department believes that 2.5 kg. hams would be a best seller. In
order to estimate the production capacity of the system, you have to estimate the cooking
time for the 2.5 kg ham.
Note:
Condensing steam yield a very high heat transfer coefficient so you can assume that the
Biot number is infinite.
Problem 3 (10 points)
a) In a detailed analysis of conduction problem, what do we mean by short times and
long times. What significance does that have in the physics of the problem?
b) In convection, what does one mean by a developing temperature field?
ChE 333 - Second Examination 2 April 3, 2000
ChE 333 Heat Transfer
Second Examination
Part B (Open Book)
Problem 3 (30 points)
I lived in Texas in the winter of 1962-63. It was a horrible winter, particularly on the
Gulf Coast where the temperature did not go above freezing for 45 days. My house
was a cottage built on piers so that the water piping was exposed under the house to
air at 5C. My wife and I discovered that we had to keep water flowing in the pipes
in order to prevent freezing. The principal exposed copper tubing was not insulated,
2.25 cm in diameter (OD) with and ID of 2,0 cm. In order to estimate the minimum
flow rate, one has to estimate the Overall Heat Transfer Coefficient between the
water and the air.
a) Estimate the mean heat transfer coefficient from the water to the inside
tubing wall for laminar flow in the tubing for a flow of 5 cc/min
b) Estimate the mean transfer coefficient for cross-flow of air past the tubing at
10 km/hr
c) Evaluate the Overall heat transfer coefficient between the water and the air.
In our analysis of heat transfer to a fluid flowing in a pipe, we derived that the
equation for the mixing cup temperature (the average temperature) is :

wC
p
d T
dz
= Dh T
R
T
This equation can be integrated to yield

T T
R
T
1
T
R
= exp
h Dz
wC
p
= exp 4 St
z
D
d) What assumptions were necessary to derive this equation
e) For what flow regimes is it applicable?
f) How can this relation be used to evaluate whether the pipes described above
would freeze? Describe the procedure..
Data
(water) (air)
Prandtl No 12.9 0.69
k 0.556 0.025 W/m-K
1.7 13.9 x10
-6
m
2
/sec
C
p
4217 1009 J/kg-K
1.311 1000 kg/m
3
ChE 333 - Second Examination 3 April 3, 2000
Problem 4 (20 points)
An experiment that was once part of the Senior Laboratory was the measurement of the
temperature history of a point in an acrylic disk. The purpose of the experiment is to
determine the Fourier diffusivity , , of the material. The experiment consisted of the
following steps:
1. Place a uniformly cold (0C) disk in a stirred water bath at 60C
2 Measure the temperature at some point in the disk at 1 minute intervals.
The data they obtained were the following:
Time (minutes) 0 6 10 15 20 25 35 40
Temperature (C) 0 2.1 8.0 16.1 23.3 29.5 38.9 42.4
Time (minutes) 50 60 70 90 100 120
Temperature (C) 47.8 51.6 54.1 57.1 58.0 59.0
The disk is sufficiently thin that it can be treated as an infinitely wide slab of 1.5 cm. thick.
What is the Fourier diffusivity?
The solution for the temperature field is
Nota Bene
If you do not understand the question, ask!!!
If you believe that there is missing information, ask!!!!

T - T
1
T
0
- T
1
=
4 -1
n
2n + 1

n =0

e-
2n+1
2
4t
b
2 cos 2n + 1
x
b
Examination #3 April 28, 2000
ChE 333 1
Chemical Engineering 333 - Heat Transfer
28 April 2000
Open Book and Notes
1. (40 points)
A water-cooled countercurrent heat exchanger is used to condense ammonia in a
refrigeration plant. Superheated ammonia enters the condenser at 50C, cools to 25C,
condenses at 25C, and the condensed liquid ammonia is subcooled to 6C before leaving
the exchanger. Cooling water enters at 5C and leaves at 15C. The water flow rate is 25
kg/min.
a) Plot the ammonia temperature, T
A
, , versus the heat transferred from ammonia,
Q
A
. Also on the same sketch, plot the water temperature, T
W
, versus the heat
transferred to water, Q
W
. (15 points)
b) Describe how you would determine the minimum water rate ? (5 points)
c) What is the condensation rate of the ammonia? (5 points)
d) Determine the condenser area required. (15 points)
Data
Overall Heat Transfer Coefficients
NH
3
vapor to water U = 60 W/m
2
-C
Condensing

NH
3
to water U = 360 W/m
2
-C
NH
3
liquid to water U = 120 W/m
2
-C
Enthalpy data
NH
3
vapor at 50C H = 1537.7 kJ/kg
NH
3
saturated vapor at 25C H = 1465.0 kJ/kg
NH
3
saturated liquid at 25C H = 298.8 kJ/kg
NH
3
subcooled liquid at 6C H = 208.9 kJ/kg
H
2
O liquid at 5C H = 21.0 kJ/kg
H
2
O liquid at 15C H = 63.0 kJ/kg
Hint - Break the condenser up into three parts and calculate the area for each and then
sum for the total area.
Examination #3 April 28, 2000
ChE 333 2
2. (30 points)
Water at a temperature of 195F is used to heat kerosene. The exchanger has to heat 12000
lbm/hr of the kerosene. The temperature of the kerosene is 65 F. The double pipe heat
exchanger consists of 6 feet of 1 1/4 inch Type M copper tubing inside and a 2 inch Type
M copper tubing outside. The water is fed at the rate of 5000 lbm/hr.
a) Determine the minimum water rate which can be used to cool the kerosene.
b) Determine the overall heat transfer coefficient.
c) Determine the outlet temperature of both streams in countercurrent flow.
d) Determine the outlet temperature of both streams in co-current flow.
Data:
Inside diameter of outer pipe - 0.1076 ft.
Outside diameter of inner pipe - 0.1176 ft.
Inside diameter of inner pipe - 0.1674 ft.
water kerosene copper units
conductivity 0.376 0.355 231.0 BTU/hr-ft-F
density 61.464 55.85 557.5 lb
m
/ft
3
heat capacity 1.00 0.555 0.915 BTU/lb
m
-F
kinematic viscosity 0.514 0.421 10
-5
ft
2
/hr
Hint: Firstly, determine the flow regime is the shell and in the tube. Select the appropriate
relations for the Nusselt number. Then determine the Overall Heat Transfer coefficient.
4. (30 points)
A shell-and-tube heat exchanger is used to heat 85,000 lb./hr of water from 187F to
255F with 40 psia steam condensing on the tube side. The exchanger has one shell-side
pass and two tube-side passes. There are 120 tubes with an inside diameter of 1.0 in. and
an outside diameter of 1.125 in. If the overall coefficient based on the inside area is 572
BTU/hr-ft
2
-F, how long must the tubes be?
CHE 333
Solutions to Problem Set 1
Problem 1.2
The conductivity, surface area and thickness of a wall separating a room from the ambient, are known.
Given the temperature in the room, the heat loss can be described by 1-D conduction through the wall.
The given values are used to set up the heat loss rate Q by Fourier's law (see below).
k .. , 0.75 1 1.25 Ti 25 To .. , 15 10 40
S 20 L 0.3
Q , To k
. .
k S
To Ti
L
Q , To 0.75
.
2 10
3
.
1.75 10
3
.
1.5 10
3
.
1.25 10
3
.
1 10
3
750
500
250
0
250
500
750
Q , To 1
.
2.667 10
3
.
2.333 10
3
.
2 10
3
.
1.667 10
3
.
1.333 10
3
.
1 10
3
666.667
333.333
0
333.333
666.667
.
1 10
3
Q , To 1.25
.
3.333 10
3
.
2.917 10
3
.
2.5 10
3
.
2.083 10
3
.
1.667 10
3
.
1.25 10
3
833.333
416.667
0
416.667
833.333
.
1.25 10
3
The heat loss rate is always higher for a wall of higher conductivity.All the curves meet at To=25 C
where there is no heat loss (the room temperature =ambient temperature). The sign of Q changes
where there is no heat loss (the room temperature =ambient temperature). The sign of Q changes
as one crosses 25 C, since below this temperature the room loses heat, whereas it gains heat if the ambient temp
is higher than 25 C.
Problem 1.6
This problem involves comparing the properties of two different materials for the same temperature
difference. For the composite wall:
qc =-kc T/Lc
For the masonry wall:
qm=-km T/Lm (for the same temperature difference)
It is required that qm =0.8 qc
Hence -0.8 kc T/Lc =-km T/Lm

Or
kc 0.25 Lc 0.1 km 0.75
Lm
.
kmLc
.
0.8 kc
= Lm 0.375 m
Obviously, since the masonry wall is more conducting, it will have to be thicker than the composite
wall if one expects it to conduct only 80% of the heat conducted by the composite wall for the same
temperature difference across them.
Problem 1.7
Another problem of 1-D conduction, this time across a chip that is 1 mm thick and with a surface
area S =5 mm square. The heat generated by circuits in the chip is given and must be dissipated at the same
rate to maintain steady-state.
k 150 t 0.001
S 0.005
2
or = S 2.5 10
5
Q 4
Heat loss rate Q =kS T/ t => DT
.
Q
t
.
k S
So = DT 1.067 K
Problem 1.13
This time the heat loss is described by a heat transfer coefficient (i.e.,"Newton's law of cooling"). The heat loss
rate Q =h S T where h is the heat transfer coefficient, S =surface area available for heat transfer, and T is
the temperature difference. The maximum chip power allowed is determined by the heat transfer coefficient,
since all this generated heat must be removed by the coolant.
So Qmax =h T S
For air cooling:
h1 200
DT 85 15 or = DT 70
S 0.005
2
or = S 2.5 10
5
Qmax
. .
h1 DT S
= Qmax 0.35W
Similarly for the dielectric coolant:
h2 3000
Qmax
. .
h2 DT S or
= Qmax 5.25 W
The higher the value of h, the higher the maximum allowed chip power.
Problem 1.29
Water is used for heating a house. The daily water consumption (in volume units) is given, as also the
temperature difference to be achieved (55-15 =40 deg. C). So the energy requirement is
simply E =mCp T where Cp is the specific heat of water.
m
. .
1000 100
365
264.17
or = m 1.382 10
5 kg
year
Cp 4180
J
kgK
DT 40
J
year

E
. .
mCp DT or = E 2.31 10
10
But 1 kWh =3.6 * 10^6 J ==>E =6417 kWh/ year
If an electrical heater is used to directly heat the water, then assuming 100% efficiency and given the cost (C) of
electrical power, the heating cost H will be:
C 0.08
H
.
6417 C or = H 513.36
dollars
year
Alternatively, a heat pump may be used. This device comprises a compressor that takes W units of work from an
external source (eg. electrical power) to circulate compressed refrigerant in the system. The refrigerant takes an
amount of heat Qc from the ground and releases heat Qh in the house to heat the water. The energy balance on
the heat pump gives
W = Qh - Qc (1)
Qh is known since it is the heat requirement (calculated earlier, 6417 kWh). Also the COP (coefficient of
performance) of the heat pump is given.
COP =Qc/W , i.e., the amount of heat it can take in per unit external work provided. In this case, the COP =3
==>Qc =3W (2)
(1) and (2) give W =Qh/4 =1604.25 kWh
However this work requires electrical power and the efficiency with which the electrical power is converted into
work, is 85%. So the electrical power requirement will be W/0.85 =1887.35 kWh
Hence the cost H =1887.35 * 0.08 =151 dollars/year
The heat pump is more economical since it takes part of the energy requirements from the surroundings.
Problem 1.35
Liquid oxygen is stored in a spherical tank of known size. Because of convective and radiative heat transfer from
the surroundings, the oxygen evaporates; and the rate of evaporation is to be evaluated.
h 10 To 298
Ti 263
Emissivity e 0.2
s
.
5.67 10
8
.
Stefan constant
Flux q
.
h To Ti
. .
e s To
4
Ti
4
or = q 385.174
d 0.5
.
tank diameter
The heat loss rate is
Q
. .
q 3.1416 d
2
or = Q 302.516
J
s
The latent heat of vaporization (H) is given , so the evaporation rate is:
H
.
214 10
3
m
Q
H
or = m 1.414 10
3 kg
s
The expression for the flux shows that the evaporation rate is a linear function of the emissivity.
e .. , 0.2 0.3 1
m e
. . .
h To Ti
. .
es To
4
Ti
4
3.1416
d
2
H
kg
s
0.2 0.4 0.6 0.8 1
0.0014
0.0016
0.0018
0.002
m e
e
End
Problem 2.7 Incopera and DeWitt
In this problem we have one-dimensional heat conduction through a wall at steady state. The
physical properties of the material are assumed to be constant. We will use Fourier's law to
calculate the unknown quantities.
"Thermal conductivity of the wall
k
.
25
W
.
mK
W
"Wall thickness
L
.
0.5 m
dT T2 T1
Case 1: We know the temperatures at the two ends of the wall and we have to calculate the
temperature gradient and the thermal flux
T1
.
400 K T2
.
300 K
Temperature gradient calculation: dT_dx
T1 T2
L
= dT_dx 200 m
1
K
Calculation of Heat Flux: q
.
k dT_dx k = q
W
m
2
q
To sketch the temperature distribution we must first solve the Fourier's equation. to do that
we need a boundary condition. We see from the sketch in the book that at x=0 the
Temperature is T=T2. Notice that at x=0 the temperature is T2 and at x=L the temperature is
T1 (see sketch in the book)
q
.
k
dT
dx
T x
.
q x
k
T2
q
k
Distance[m]
T
e
m
p
e
r
a
t
u
r
e

[
K
]
T x
L 0 x
T
Case 2: We know the temperature at one end of the wall and the temperature gradient we have to
calculate the temperature at the other end and the heat flux
1
T1
.
273.15 100 K dT_dx
.
250
K
m
Calculation of T2
T2 T1
.
dT_dx L = T2 498.15 K
Calculation of Heat Flux: q
.
k dT_dx k = q
W
m
2
q
T x
.
q x
k
T2
q
k
Distance[m]
T
e
m
p
e
r
a
t
u
r
e

[
K
]
T x
L 0 x
T
Case 3: We know the temperature at one end of the wall and the temperature gradient we have to
calculate the temperature at the other end and the heat flux
T1
.
273.15 80 K dT_dx
.
200
K
m
T2 T1
.
dT_dx L
Calculation of T2 = T2 253.15 K
Calculation of Heat Flux: q
.
k dT_dx k = q
W
m
2
q
T x
.
q x
k
T2
q
k
Distance[m]
T
e
m
p
e
r
a
t
u
r
e

[
K
]
T x
L 0 x
T
Case 4: We know the temperature at one end of the wall and the heat flux. We have to calculate
the temperature at the other end and the temperature gradient
q
.
4000
W
m
2
W
T2
.
273.15 5 K
2
Calculation of temperature gradient: dT_dx
q
k
q
k
= dT_dx 200 m
1
K
Calculation of T1: T1 T2
.
dT_dx L = T1 368.15 K
T x
.
q x
k
T2
q
k
Distance[m]
T
e
m
p
e
r
a
t
u
r
e

[
K
]
T x
L 0 x
T
Case 5: We know the temperature at one end of the wall and the heat flux. We have to calculate
the temperature at the other end and the temperature gradient
q
.
3000
W
m
2
W
T1
.
273.15 30 K
Calculation of temperature gradient: dT_dx
q
k
q
k
= dT_dx 200 m
1
K
Calculation of T2: T2 T1
.
dT_dx L
= T2 203.15 K
T x
.
q x
k
T2
q
k
Distance[m]
T
e
m
p
e
r
a
t
u
r
e

[
K
]
T x
L 0 x
T
3
Problem 2.11 Incopera and DeWit
In order to solve this problem we have to use Fourier's law in two dimensions. We have to keep in mind
that the direction of the heat flux is always normal to a surface of constant temperature that is called
isothermal temperature. In our problem surfaces A, and B are isothermal so the heat flux will be normal
to them. The normal to surface A is in the y direction, and the normal to surface B is in the x direction.
We further have to assume no heat generation and that the thermal conductivity does not depend on
direction.
q
. .
k A
dT
dx
dT
dy
Area_A
.
2 m
k
.
10
W
.
mK
W
Area_B
.
1 m
dT_dy_A
.
30
K
m
dT_dx_A
.
0
K
m
dT_dy_B
.
0
K
m
qA
. .
k Area_A dT_dy_A dT_dy_B k
Heat rate on Surface A :
= qA
W
m
qA
Heat rate on Surface A = Heat Rate on Surface B
dT_dx_B
qA
.
k Area_B
qA
k
= dT_dx_B dT_dx_B
Problem 2.18 Incopera and DeWit
(a) In order to estimate the thermal conductivity of the test materials we need to know the thermal
conductivity of the reference material. We'll initially assume that the temperature at the point
where the differential thermocouple of the reference material is 350 K. We can make a plot of the
thermal conductivity of the Armco iron from the values given in table A2 of the book.
T
.
100 K
.
200 K
.
300 K
.
400 K
.
600 K
.
800 K
.
1000 K
.
1200 K
.
1500 K
k
.
95.6
W
.
m K
.
80.6
W
.
mK
.
80.2
W
.
mK
.
65.7
W
.
mK
.
53.1
W
.
mK
.
42.2
W
.
mK
.
32.3
W
.
mK
.
28.7
W
.
m K
.
31.4
W
.
mK
W W W W W W W W W
4
Temperature[K]
T
h
e
r
m
a
l

C
o
n
d
u
c
t
i
v
i
t
y

[
W
/
m
/
k
]
k
800 0 T
k
From the graph above we see that at 350 K the thermal conductivity of Armco is ~70 W/m/K
The flux between the three different materials will be equal
L
.
10 mm dTr
.
2.49 K kr
.
70
W
.
mK
W
T_heat_source
.
400 K
dT1
.
3.32 K dT2
.
3.32 K
T_cold_sink
.
300 K
Flux of reference material q
.
kr
dTr
L
kr = q
W
m
2
q
k1
q
dT1
L
q
Thermal conductivity of Sample 1: = k1
W
.
mK
k1
k2
q
dT1
L
q
Thermal conductivity of Sample 2: = k2
W
.
mK
k2
Each value of thermal conductivity can be assigned to the average temperature of the
sample. Knowing the temperatures at the heat source and the cold sink, and the thermal
conductivities of all the materials we can calculate the length of the cylindrical samples.It is
possible then to calculate the average temperature of each cylinder.
.
dT1
L
cylinder_length
.
dTr cylinder_length
L
.
dT2 cylinder_length
L
.
100 K
cylinder_length
. .
100
K
dT1 dTr dT2
L
= cylinder_length 0.11 m
The temperature at the interface between the Test Sample 1 and the Reference material is
T_1_r T_heat_source
.
dT1 cylinder_length
L
= T_1_r 363.636 K
5
The average temperature of sample one:
T_1_ave
T_heat_source T_1_r
2
= T_1_ave 381.818 K
The temperature at the interface between the Test Sample 2 and the reference material is
T_2_r T_1_r
.
dTr
cylinder_length
L
= T_2_r 336.364 K
The average temperature of reference material : T_r_ave
T_2_r T_1_r
2
= T_r_ave 350 K
This justifies our initial selection of the thermal conductivity for the reference material
The average temperature of sample two:
T_2_ave
T_cold_sink T_2_r
2
= T_2_ave 318.182 K
(b) We assume that the differential thermocouples imbedded in the samples have the same
spacing and the three cylinders have the same length.
Then dT1 and dT2 will depend on the thermal conductivity of the samples.
One case where dT1 and dT2 are expected to be different is when the test samples are different
materials. In such a case they will have significantly different thermal conductivities.
When the materials of the test samples are the same:
dT1 is not expected to be equal with dT2 if the thermal conductivity of the material is temperature
sensitive in the temperature range of the experiment.
Problem 2.37 Incopera and DeWit
(a) We use cylindrical coordinates to write the heat diffusion equations for the cable and the
insulation. The system is symmetric the and z directions so the corresponding terms will be
equal to zero.
Cable
.
1
r
d
.
k
r
dT
dr
dr
q_gen 0 (1)
The boundary conditions to eq. 1 are
1.T=Ts1 when r=r1. Outer surface temperature
2. dT/dr=0 at r=0. Symmetry of the problem
6
d
.
k
c
dT
dr
. .
q_gen r dr
dT
dr
.
q_gen
r
.
2 kc
C1
.
kc r
Applying the second boundary condition we find that
C1=0
Applying the first boundary condition we find that
T
.
q_gen r
2
.
2 kc
C2
C2 Ts1
.
q_gen r1
2
.
4 kc
So the temperature profile for the cable is
T r Ts1
.
.
q_gen r1
2
.
4 kc
1
r
2
r1
2
Ts1
q_gen r1
kc
r1
Insulation
Insulation behavior can also be described with the heat diffusion equation in
cylindrical coordinates. The difference with the cable is that there is no heat
generation term.
.
1
r
d
.
k
s
dT
dr
dr
0 (2)
The boundary conditions to eq. 2 are
1.T=Ts1 when r=r1. Inner surface temperature
2.T=Ts1 when r=r2. Outer surface temperature
By solving Eq. 2 we get T
.
C1
ln r
ks
C2
By applying the B.C. we can calculate the constants C1 and C2. After substituting them we get
Ti r Ts1
.
Ts1 Ts2
ln
r
r1
ln
r1
r2
Ts1 Ts1 Ts2
r1
r1
r2
7
Ts1
.
343.15 K Ts2
.
303.15 K r1
.
15 mm
kc
.
200
W
m
W
r2
.
15.5 mm
q_gen
.
100
W
s
W
s
c1 .. ,
.
0 mm
.
1 mm
.
15 mm c2 .. ,
.
15 mm
.
15.05 mm
.
15.5 mm
T r Ts1
.
.
q_gen r1
2
.
4 kc
1
r
2
r1
2
q_gen
kc
Ti r Ts1
.
Ts1 Ts2
ln
r
r1
ln
r1
r2
T c1
15.5 c1
mm
T c1
15 15.2 15.4 15.6
300
320
340
360
Ti c2
c2
mm
Temperature Drop Profile in the cable Temperature Drop Profile in insulation
(b)
The heat transfer is given by Fourier's Law q
. .
k A
dT
dx
The outside area A is equal to A
. .
2 pi r2
dT
dr
. .
Ts1 Ts2
ln
r1
r2
k
1
.
k r
From part (a) we see that for the insulation
Substituting in Fourier's Law we get
8
q
. . .
2 pi k
Ts1 Ts2
ln
r1
r2
We will apply an energy balance to a control surface around the cable, and more
specifically at the interface between the cable and the insulation. The energy balance
at steady state can be written as
Rate of energy in - Rate of energy out +Rate of energy generation=0
There is no energy that is entering the system, so the rate of heat conduction per unit length
out will be equal to the rate of heat generation
q
.
V q_gen
q
. . . .
Unit_Length 2 pi r1 q_gen
q
Unit_Length
. . .
2 pi r1 q_gen
(c) We apply a control volume in the outer surface of the insulation. There is no heat generation in
the insulation layer of the cable. The heat per unit entering is equal to the amount of heat
generated, which was calculated in part b. The heat loss from the outer surface of the insulation
layer is due to convection and radiation. In such a case
q
Unit_Length
. . . .
h 2 pi r1 Ts2 T_inf
. . . . .
esigma Ts2
4
Tsur
4
2 pi r2
Substituting from part b we get
Ts2
4
.
.
h r2
.
esigma
Ts2
.
1
.
e sigma
.
r1
r2
q_gen
. .
r2 h T_inf
. . .
r2 esigmaTsur
4
0
The above equation is 4th order, and it has four possible solutions.
Let B=hr2/e*sigma and C=
.
1
.
esigma
.
r1
r2
q_gen
. .
r2 h T_inf
. . .
r2 e sigmaTsur
4
(3)
Then the equation (3) becomes
x
4
.
B x C 0
The above equations has four solutions. From all the four only one has physical meaning.
9
(d)
R
.
0.005
ohm
m
I
.
250 AA
Energy per unit length generated Power
.
I
2
R I = Power
W
m
Power
Radius of cable: r1
.
15 mm
Radius of cable and insulation: r2
.
15.5 mm
kc
.
200
W
.
mK
W
Thermal conductivity of cable:
Thermal conductivity of insulation: ks
.
0.15
W
.
mK
W
h
.
25
W
.
m
2
K
W
Convective heat transfer coefficient:
Air Temperature: T_inf
.
273.15 25 K
Surface Temperature Tsur
.
273.15 35 K
Emissivity : e 0.9
sigma
. .
5.67 10
8 W
.
m
2
K
4
W
Stefan Boltzmann constant :
C
.
1
.
esigma
.
1
. .
2 3.14 r2
Power
.
h T_inf
. .
esigmaTsur
4
Power T_inf
sigma
Power h
B
h
.
esigma
h
sigma
f x x
4
.
B x C B C
To find the solution we will first plot f(x) over a range of temperatures
temp ..
.
300 K
.
500 K
10
f temp
temp
f
temp
.
400 K
Ts2 root , f temp temp f
So the temperature outside of the insulation is = Ts2 303.15 K
q
. . .
2 pi k
Ts1 Ts2
ln
r1
r2
From (b) we have
solving for Ts1 we get
Ts1 Ts2
.
Power ln
r2
r1
. .
2 3.1415 ks
Ts2
. .
2 3.1415 ks
Power
ks
= Ts1 343.15 K
From part (a) T r Ts1
.
.
Power
.
3.14 r1
2
r1
2
.
4 kc
1
r
2
r1
2
Power
kc
Tcenter Ts1
Power
3.1415
.
4 kc
Ts1
.
4 kc
Power
kc
= Tcenter Tcenter
11
Problem 3.15 Incopera and DeWitt
When we have one-dimensional, steady state conduction in a composite plane wall with no heat
generation the heat flux is constant. In our example we have five thermal resistances in series.
One thermal resistance for convection in the outer wall, three resistances for conduction for each
component of the wall and one resistance for convection for the inner wall.
Ti
.
273.15 20 K
Inside Temperature = Ti 293.15 K
Outside Temperature To
.
273.15 15 K = To 258.15 K
Wall Area A
.
350 m
2
kp
.
0.17
W
.
m K
W
Plaster Board thermal conductivity:
Plaster Board Length
Lp
.
10 mm
Fiber Glass thermal conductivity
(Estimated value from table A3)
kb
.
0.033
W
.
mK
W
Lb
.
100 mm
Fiber Glass Length
ks
.
0.12
W
.
mK
W
Plywood siding thermal conductivity:
Plywood siding Length Ls
.
20 mm
hi
.
30
W
.
m
2
K
W
ho
.
60
W
.
m
2
K
W
The total heat loss through the wall is given by
q
Ti To
1
.
hi A
Lp
.
kp A
Lb
.
kb A
Ls
.
ks A
1
.
ho A
.
hi A
.
kp A hi kp
So the total thermal resistance of the wall is
Rwall
1
.
hi A
Lp
.
kp A
Lb
.
kb A
Ls
.
ks A
1
.
ho A
.
hi A
.
kp A hi kp
12
= Rwall
K
W
Rwall
(b) The total heat loss through the wall is
= q W q ho1 ho ho
(c) I a windy day the convective heat loss increases
ho
.
300
W
.
m
2
K
W
qwind
Ti To
1
.
hi A
Lp
.
kp A
Lb
.
kb A
Ls
.
ks A
1
.
ho A
.
hi A
.
kp A hi kp
The new heat loss is
= qwind qwind
Percentage increase
of Heat Loss
q_percent
.
qwind q
100
q
qwind q
q
= q_percent q_percent %
Percentage increase
of convective heat transfer
coefficient
h_percent
.
ho ho1
100
ho1
ho ho1
ho1
= h_percent h_percent %
We observe that even though the resistance in convective heat loss constant decreased 400%, the
total heat loss increased only by 0.405%.
(d) In the wall of our example the thermal resistances of the different parts are in series. In such a case
the controlling resistnace will be the larger one. In this example the larger resistance is due to the layer
of the glass fiber blanket
Problem 3.102 Incopera and DeWitt
In order to solve this problem we will make the following assumptions:
1. Steady state conditions
13
2. One dimensional conduction along the rod
3. Constant physical properties
4. No heat loss due to radiation
5. There is no heat loss through the insulation
6. The convective heat transfer coefficient isuniform along the exposed surface of the rod
7. There is no heat loss from the end of the rod (rod is adiabatic), since it is insulated
Data given by the problem statement
k
.
60
W
.
mK
W
Diameter of the rod: D
.
25 mm Thermal conductivity of the rod:
Area of the rod:
A
.
3.1415
D
2
4
= A 4.909 10
4
m
2
Temperature of furnace wall: Tw
.
273.15 200 K
Thickness of furnce insulation: Lins
.
200 mm
Maximum operating temperature: Tmax
.
273.15 100 K
Ambient air temperature: Tair
.
273.15 25 K
h
.
15
W
.
m
2
K
W
Convective transfer coefficient:
In the insulated part of the rod we have one dimensional heat conduction. The temparature To
must satisfy the following relation:
To Tw
.
Lins
q
.
k A
Tw
q
.
k A
q
k
1
where q is the heat that is removed from the oven through the rod. This amount will be equal
with the amount of heat that is removed from the rod.
Fin Perimeter: P
.
3.1415 D
m1
.
h P
.
k A
h
k
For an adiabatic fin the heat transfer rate will be given by equation (3.76) of the book:
q
. . . . .
h P k A To Tair tanh
.
m Lo
.
m Lo h k Lo (2)
We can substitue (2) in (1) and solve for To
To Tw
.
Lins
. . . . .
h P k A To Tair tanh
.
m1 Lo
.
k A
To
. .
Tw k A
. . .
Lins
. . .
h P k A tanh
.
m1 Lo Tair
.
k A
. .
Lins
. . .
h P k A tanh
.
m1 Lo
A Tair k h k m1 Lo
14
(b) Lo
.
200 mm
To
. .
Tw k A
. . .
Lins
. . .
h P k A tanh
.
m1 Lo Tair
.
k A
. .
Lins
. . .
h P k A tanh
.
m1 Lo
.
m1 Lo k h k m1
= To 258.15 K
We see that the temperature To is larger than the maximum operating limit. We will examine several
design alternatives to see which parameter has the largest influence on the temperature.
Change Insulation length
L .. ,
.
10 mm
.
100 mm
.
400 mm
T Lins
. .
Tw k A
. . .
Lins
. . .
h P k A tanh
.
m1 Lo Tair
.
k A
. .
Lins
. . .
h P k A tanh
.
m1 Lo
k h k m1
k h k m1
Insulation thickness [mm]
T
o

[
K
]
450
300
T L
.300 .100 L
T L
From the graph above we see that increasing the insulation thickness of the oven makes the
temperature To to decrease. To make To~373K, the insulation thickness has to be increased to
~250mm (about 25%)
= T
.
250 mm T
.
250 mm
Change length of the rod
Lins
.
200 mm
L .. ,
.
10 mm
.
50 mm
.
500 mm
15
T Lo
. .
Tw k A
. . .
Lins
. . .
h P k A tanh
.
.
h P
.
k A
Lo Tair
.
k A
. .
Lins
. . .
h P k A tanh
.
.
h P
.
k A
Lo
k h k
h
k
k h k
h
k
Insulation thickness [mm]
T
o

[
K
]
450
300
T L
.400 .050 L
T L
Increasing the length of the rod will make To decrease but not enough. Even if the exposed part
of the rod is ~1m the temperature will not be below the specified operating limit
= T
.
1000 mm T
.
1000 mm
Changing Material
Lins
.
200 mm
Lo
.
200 mm
k1 .. ,
.
1
W
.
1
.
mK
.
10
W
.
mK
.
1000
W
.
mK
W W W
T k
. .
Tw k A
. . .
Lins
. . .
h P k A tanh
.
.
h P
.
k A
Lo Tair
.
k A
. .
Lins
. . .
h P k A tanh
.
.
h P
.
k A
Lo
h
h
h
h
Thermal conductivity of Material [W/m/K]
T
o

[
K
]
400
300
T k1
100 0 k1
T k1
16
T
o

[
K
]
Another way to decrease To below the 373K is to change the material of the rod. A material
with smaller thermal conductivity can be used. For example different Stailness Steel alloys
have thermal conductivities in the range of ~15 W/m/K and can be used instead.
Problem 3.119 Incopera and DeWitt
In order to calculate the maximum allowable chip power dissipation we have to calculate first
the total heat resistance of the fin and the contact interface with the chip.
Assumptions :
1. Steady-state conditions
2. One-dimension radial conduction fins
3. Constant properties
4. No heat exchange due to radiation
5. Uniform convection coefficient over outer surface
6. The entire fin surface, as well as the exposed base are maintained at the same
temperature
Below you can see a schematic of the thermal circuit:
Characteristic dimensions of fin Physical Properties
Lb
.
3 mm
k
.
400
W
.
mK
W
Lf
.
6 mm
S
.
0.50 mm
w1
.
0.25 mm Wc
.
16 mm
17
Lc Lf
w1
2
Area of one fin Af
. .
2 w1 Lc
Bulk Area Ab
.
Wc Wc
Total Number of fins N
.
Wc Wc
.
S S
= N 1.024 10
3
Total Area At
.
N Af Ab
Interface Contact Resistance : R_tc
. . .
5 10
6
m
2 K
WW
Conductive Resistance of the bulk: R_b
Lb
.
k Wc
2
k
Resistance for covective heat loss from bulk surface and fin
h
.
1500
W
.
m
2
K
W
m1
.
2 h
.
k w1
h
k
nf
tanh
.
m1 Lc
.
m1 Lc
.
m1 Lc m1
no 1
.
.
N Af
At
1 nf nf
R_fin
1
. .
no h At no h
T_hot
.
273.15 85 K T_cold
.
273.15 25 K
q_total
T_hot T_cold
R_fin R_b
R_tc
Ab
R_b
Ab
R_fin R_b
R_tc
= q_total W q_total
18
(b) Substituting the analytical quantities for each parameter in the equation of heat dissipation we
can get
q_t , w1 Lf
T_hot T_cold
R_b
R_tc
Ab
1
. .
h
.
N
. .
2 w1 Lf w1
2
Ab 1
.
.
N
. .
2 w1 Lf w1
2
.
N
. .
2 w1 Lf w1
2
Ab
1
tanh
. .
2
h
.
k w1
Lf
w1
2
. .
2
h
.
k w1
Lf
w1
2
R_b
R_tc
Ab
1
. .
h
.
N
. .
2 w1 Lf w1
2
Ab 1
.
.
N
. .
2 w1 Lf w1
2
.
N
. .
2 w1 Lf w1
2
Ab
1
tanh
. .
2
h
.
k w1
Lf
w1
2
. .
2
h
.
k w1
Lf
w1
2
It is not possible to increase w without increasing S. This is because for we already have (S-w)=0.25
We'll examine the influence of w by trying different values smaller than 0.25 mm
L_fin .. ,
.
6 mm
.
7 mm
.
10 mm
q_t ,
.
0.25 mm L_fin
q_t ,
.
0.20 mm L_fin
q_t ,
.
0.15 mm L_fin
q_t ,
.
0.10 mm L_fin
10 L_fin
mm
q_t
So we see that the dissipated heat is maximum for the maximum allowed value of w, for a specified
value of S. Similarly an increased value of the fin length increase the amount of heat that can be
dissipated.
19
T (x, 0. 5 )
3 0
40
5 0
60
7 0
80
90
1 00
0. 0 0. 5 1 . 0 1 . 5 2 . 0
T (1 , y )
0
20
40
60
80
1 00
1 20
1 40
1 60
0 0. 2 0. 4 0. 6 0. 8 1 1 . 2
100 100 100 100 100
50 85.31857 103.9098 114.9796 121.3088 121.3918 200
50 87.36451 115.3409 134.6999 148.8636 164.2584 200
50 98.79856 135.3895 159.6154 175.1874 186.7784 200
50 122.4403 167.803 193.1847 205.4924 207.6675 200
50 173.1595 220.1976 239.8281 245.93 238.3994 200
300 300 300 300 300
1
2
3
4
5
6
7
S 1
S 5
0
50
100
150
200
250
300
250-300
200-250
150-200
100-150
50-100
0-50
Heat Transfer Coefficients
0
5000
10000
15000
20000
25000
0.2 0.3 0.4 0.5 0.6 0.7 0.8
mass flow rate
H
T
C
hi
ho
Uo
Tco versus mass flow rate
310
315
320
325
330
335
340
345
350
355
5 7 9 11 13 15 17 19
mass flow rate (kg/s)
T
c
o

(
K
)
Rf''=0
Rf''=0.0002
Rf''=0.0005

S-ar putea să vă placă și